Tải bản đầy đủ (.pdf) (51 trang)

Đề thi thử THPT quốc gia

Bạn đang xem bản rút gọn của tài liệu. Xem và tải ngay bản đầy đủ của tài liệu tại đây (1.95 MB, 51 trang )

<span class='text_page_counter'>(1)</span><div class='page_container' data-page=1>

1 DAYHOCTOAN.VN
<b>Bài 1. </b> Cho ba số thực dương thỏa mãn 1 1 1 1


<i>x</i>  <i>y</i>  <i>z</i>  <i>xyz</i>

 

1 . Tìm giá trị lớn nhất của
2


2 1


1 1 1


<i>y</i>


<i>x</i> <i>z</i>


<i>P</i>


<i>x</i> <i>y</i> <i>z</i>




  


  


<b>Lời giải: </b>


Ta có <i>A B C</i>, , (0; ), <i>A</i>  <i>B</i> <i>C</i>  thì tan tan tan tan tan tan 1


2 2 2 2 2 2


<i>A</i> <i>B</i> <i>B</i> <i>C</i> <i>C</i> <i>A</i>



  


Theo giả thiết

 

1  <i>x</i>. <i>y</i> <i>y</i>. <i>z</i>  <i>z</i>. <i>x</i> 1
Ta có tan


2


<i>A</i>


<i>x</i>  , tan


2


<i>B</i>


<i>y</i>  , tan


2


<i>C</i>


<i>z</i>  với <i>A B C</i>, , (0; ), <i>A</i>  <i>B</i> <i>C</i> 
Ta có <i>P</i>sin<i>A</i>sin<i>B</i>cos<i>C</i> 2 cos cos 2 cos2 1


2 2 2


<i>C</i> <i>A</i><i>B</i> <i>C</i>


  



2 2


1 1 3


2(cos cos ) 1 cos


2 2 2 2 2 2


<i>C</i> <i>A</i><i>B</i> <i>A</i><i>B</i>


     


Vậy max 3


2


<i>P</i> Khi


2
3


6
<i>C</i>


<i>A</i> <i>B</i>




 





  





2 2 3


tan


12 2 3
3


<i>x</i> <i>y</i>
<i>z</i>




 


  




  


 



<b>Bài 2. </b> Cho a, b, c là 3 số thực dương thỏa mãn <i>abc</i>1 . Tìm giá trị nhỏ nhất của biểu thức:


2 2 2


( 2)(2 1) ( 2)(2 1) ( 2)(2 1)


<i>a</i> <i>b</i> <i>c</i>


<i>S</i>


<i>ab</i> <i>ab</i> <i>bc</i> <i>bc</i> <i>ac</i> <i>ac</i>


  


      .


<b>Lời giải: </b>


2


2 2


1 4 4 1


2 1 2 1 1


( 2)(2 1) 9


( )(2 ) ( 2 ) ( )



a


<i>a</i>


<i>ab</i> <i>ab</i>


<i>b</i> <i>b</i> <i>b</i> <i>b</i> <i>b</i>


<i>a</i> <i>a</i> <i>a</i> <i>a</i>


  


  <sub></sub> <sub></sub> <sub> </sub> <sub></sub> <sub></sub>


đáp số : Min 1


3


<i>S</i>  . Dấu "" xảy ra khi và chỉ khi <i>a</i>  <i>b</i> <i>c</i> 1


<b>Bài 3. </b> Tìm giá trị lớn nhất của biểu thức:


3 3


2


( )( )( )


<i>x y</i>


<i>P</i>


<i>x</i> <i>yz y</i> <i>zx z</i> <i>xy</i>


   , trong đó x, y, z là các số dương
thỏa mãn: <i>x</i>  <i>y</i> 1 <i>z</i> .


<b>Lời giải: </b>


Ta có : <i>x</i><i>yz</i>   <i>z</i> <i>y</i> 1 <i>yz</i>

<i>y</i>1



<i>z</i>1






1 1 1


<i>y</i><i>zx</i>   <i>z</i> <i>x</i> <i>zx</i> <i>x</i> <i>z</i>





1 1 1


<i>z</i><i>xy</i>   <i>x</i> <i>y</i> <i>xy</i> <i>x</i> <i>y</i>


3 3 3 3


3 3 2 3 3 2


x y x y


P = =



(x+1) (y+1) (z-1) (x+1) (y+1) (x+ y)
Theo CơSi ta có :


2
3


x x x


+ +1 3


2 2  4 ;


2
3


y y y


+ +1 3


2 2  4 ;


2


</div>
<span class='text_page_counter'>(2)</span><div class='page_container' data-page=2>

2 DAYHOCTOAN.VN
Vậy


3 3


2 2



x y 4


P


27 27 <sub>729</sub>


( x )( y )4 xy


4 4


 


Dấu "" xảy ra khi và chỉ khi <i>x</i> <i>y</i> 2; <i>z</i>5. Vậy MaxP = 4
729


<b>Bài 4. </b> Cho 3 số dương a, b, c thoả 3.
2


<i>a</i>  <i>b</i> <i>c</i> CMR: <i>B</i> 1 1<sub>3</sub> 1 1<sub>3</sub> 1 1<sub>3</sub> 729


<i>a</i> <i>b</i> <i>c</i>


   


 <sub></sub> <sub></sub>  <sub></sub>  <sub></sub>


    .


<b>Lời giải: </b>



Ta có: B 1 1<sub>3</sub> 1<sub>3</sub> 1<sub>3</sub> <sub>3</sub>1<sub>3</sub> <sub>3 3</sub>1 <sub>3 3</sub>1 <sub>3</sub>1<sub>3 3</sub>


<i>a</i> <i>b</i> <i>c</i> <i>a b</i> <i>a c</i> <i>b c</i> <i>a b c</i>


       


B  3 3


3 3 3 3 3 3 3 3 3 3 3 3
1


1 3 3

1

1



<i>a b c</i>

<i>a b c a b c</i>

<i>a b c</i>



   = 1 3 1 3 <sub>2 2 2</sub>

1

<sub>3 3 3</sub>

1


<i>abc</i>

<i>a b c</i>

<i>a b c</i>



   =
3
1 1
<i>abc</i>
 <sub></sub> 
 
 


Mặt khác: <sub></sub> <sub></sub> 


 



  


3


ab 1


8


c a b c


3 


3
3
1


1 9 729


1
8
<i>B</i>
 
 
 <sub></sub> <sub></sub>  
 
 


Vậy 1 1<sub>3</sub> 1 1<sub>3</sub> 1 1<sub>3</sub> 729


<i>a</i> <i>b</i> <i>c</i>



 <sub></sub>  <sub></sub>  <sub></sub> <sub></sub>


   


   


Dấu "" xảy ra khi và chỉ khi 1


2


<i>a</i>  <i>b</i> <i>c</i>
<b>Bài 5. </b> Cho









<i>xyz</i>


<i>z</i>


<i>y</i>


<i>x</i>


<i>z</i>


<i>y</i>



<i>x</i>

,

,

0




CMR:


2 2 2


1 1 1 3


2


1 1 1


<i>x</i> <i>y</i> <i>z</i>


  
  
<b>Lời giải: </b>
)
(
2
)
(
2
)
)(
(
)
(
1
1
2



2 <i><sub>x</sub></i> <i><sub>z</sub></i>


<i>z</i>
<i>y</i>
<i>x</i>
<i>y</i>
<i>z</i>
<i>x</i>
<i>y</i>
<i>x</i>
<i>yz</i>
<i>xyz</i>
<i>z</i>
<i>y</i>
<i>x</i>
<i>x</i>
<i>xyz</i>
<i>x</i> 













Tương tự VT


)
(
2
)
(
2
1
1
;
)
(
2
)
(
2
1
1
2


2 <i><sub>y</sub></i> <i><sub>z</sub></i>


<i>y</i>
<i>z</i>
<i>x</i>
<i>z</i>
<i>z</i>
<i>z</i>
<i>y</i>


<i>z</i>
<i>y</i>
<i>x</i>
<i>x</i>
<i>y</i> 










Dấu "" xảy ra khi và chỉ khi <i>x</i>  <i>y</i> <i>z</i> 3<sub> </sub>


<b>Bài 6. </b> Cho <i>ABC</i> nhọn thoả mãn hệ thức: tan<sub>3</sub> tan<sub>3</sub> tan<sub>3</sub> 1
tan tan tan


<i>A</i> <i>B</i> <i>C</i>


<i>B</i> <i>C</i> <i>A</i> <b>. </b>Chứng minh tam giác ABC đều.


<b>Lời giải: </b>


Do tam giác ABC nhọn nên <i>tanA</i>  0 ,<i>tanB</i>  0 , <i>tanC</i>  0 . Viết lại bất đẳng thức :


3 3 3



cot cot cot


1


cot cot cot


<i>B</i> <i>C</i> <i>A</i>


<i>A</i>  <i>B</i>  <i>C</i> 


Áp dụng bất đẳng thức Côsi :
3


2


cot


cot .cot 2 cot
cot


<i>B</i>


<i>A</i> <i>B</i> <i>B</i>


<i>A</i>  


3


2



cot


cot .cot 2 cot
cot


<i>C</i>


<i>B</i> <i>C</i> <i>C</i>


<i>B</i>  


3


2


cot


cot .cot 2 cot
cot


<i>A</i>


<i>C</i> <i>A</i> <i>A</i>


</div>
<span class='text_page_counter'>(3)</span><div class='page_container' data-page=3>

3 DAYHOCTOAN.VN
Suy ra<b> : </b>


3 3 3


2 2 2



cot cot cot


2( cot cot cot ) 1


cot cot cot


<i>B</i> <i>C</i> <i>A</i>


<i>A</i> <i>B</i> <i>C</i>


<i>A</i>  <i>B</i>  <i>C</i>    


vì <i>cotAcotB</i>  <i>cotBcotC</i> <i>cotCcotA</i>  1
Ta lại có 2 2 2


cot <i>A</i>cot <i>B</i>cot <i>C</i>cotAcotB  cotBcotC cotCcotA 1  
Từ đó suy ra :


3 3 3


cot cot cot


1


cot cot cot


<i>B</i> <i>C</i> <i>A</i>


<i>A</i>  <i>B</i>  <i>C</i> 



<b>Bài 7. </b> Cho <i>ABC</i> thỏa mãn


12
2
cot
2
cot
2
cot
2
sin
2
sin
2
sin


2  2  2  2 








 <i>A</i><sub></sub> <i>B</i><sub></sub> <i>C</i> <i>A</i> <i>B</i> <i>C</i>


. Chứng minh <i>ABC cân. </i>


<b>Lời giải: </b>



Ta có 12


2
cot
2
cot
2
cot
2
sin
2
sin
2
sin


2  2  2  2 








 <i>A</i><sub></sub> <i>B</i><sub></sub> <i>C</i> <i>A</i> <i>B</i> <i>C</i>


(1)
 15
2
B


sin
1
2
B
sin
1
2
A
sin
1
2
C
sin
2
B
sin
2
A
sin
2
2
2
2










 <sub></sub> <sub></sub>
(2)


Theo bất đăng thức cô si ta có :


12
2
A
sin
1
2
A
sin
8
2
A
sin
8
3
2
A
sin
1
2
A
sin
8
2


A
sin
8
3 <sub>2</sub>
2




  12


2
A
sin
1
2
A
sin
16
2

 .


Dấu "" xảy ra khi và chỉ khi


3
A
2
1
2


A
sin
2
A
sin
1
2
A
sin
16
2







Tương tự ta cũng có. 12
2
B
sin
1
2
B
sin
16
2



 ; 12


2
C
sin
1
2
C
sin
16
2


 36
2
B
sin
1
2
B
sin
1
2
A
sin
1
2
C
sin
2


B
sin
2
A
sin
16
2
2
2









 <sub></sub> <sub></sub>
(3)


Mặt khác ta có


2
B
A
cos
4
B
A


cos
4
B
A
sin
2
2
C
sin
2
B
sin
2
A


sin       


2
3
2
1
4
B
A
sin
2
2
3
4
B


A
sin
2
1
4
B
A
sin
2
2


2 <sub></sub> 






  <sub></sub>







Do đó:
2
3
2
C


sin
2
B
sin
2
A


sin    (4). Dấu "" xảy ra khi và chỉ khi


3


<i>B</i>


<i>A</i>


Từ (3) và (4) ta có:


2
14
.
3
36
2
C
sin
2
B
sin
2


A
sin
14
2
B
sin
1
2
B
sin
1
2
A
sin
1
2
C
sin
2
B
sin
2
A
sin
16
2
2
2








 <sub></sub> <sub></sub>









 <sub></sub> <sub></sub>
 15
2
B
sin
1
2
B
sin
1
2
A
sin
1
2

C
sin
2
B
sin
2
A
sin
2
2
2
2









 <sub></sub> <sub></sub>
(5)


</div>
<span class='text_page_counter'>(4)</span><div class='page_container' data-page=4>

4 DAYHOCTOAN.VN
Như vậy tam giác ABC đều.


<b>Bài 8. </b> Với ; ;<i>x y z</i>0 thoả mãn: <i>x</i>4 <i>y</i>4<i>z</i>4 3. Tìm giá trị lớn nhất của biểu thức: <i>P</i>  <i>x</i> <i>y</i> 2<i>z</i>


<b>Lời giải: </b>



Ta có: với <i>a</i>0. Áp dụng BĐT Bunhiakopsky




2


2 1 1 2 2 2 2


. 2 1 ax 4


a


<i>P</i> <i>a x</i> <i>a y</i> <i>z</i> <i>ay</i> <i>z</i>


<i>a</i> <i>a</i>


   


<sub></sub>   <sub></sub> <sub></sub>  <sub></sub>  


 


 






2


4 2 2 4 4 4



1 2 16


<i>P</i> <i>a</i> <i>x</i> <i>y</i> <i>z</i>


<i>a</i>


 


 <sub></sub>  <sub></sub>   


 




2
2
4<sub>3</sub> 2 <sub>1</sub> <sub>2</sub> <sub>16</sub>


<i>P</i> <i>a</i>


<i>a</i>


 


  <sub></sub>  <sub></sub> 


 


Dấu "=" xảy ra  2 2 2



2 2 2 2


2 4


4
4


<i>x</i> <i>y</i>


<i>x</i> <i>y</i>


<i>ax</i> <i>z</i> <i>a x</i> <i>z</i>


<i>x</i> <i>z</i> <i>x</i> <i>az</i>


<i>a</i>


   


 <sub></sub>


   


 


  <sub></sub>





 <sub></sub>





3 2 2 2 2
16
<i>a x z</i> <i>x z</i>


   <i><sub>a</sub></i> 3<sub>16</sub><sub> </sub>


Vậy dấu "=" xảy ra khi 4 3 4


4 4 4


16
3
<i>x</i> <i>y</i>


<i>z</i> <i>x</i>


<i>x</i> <i>y</i> <i>z</i>










   


3


4 4


3 3


3 3


; 2.


2 16 2 16


<i>x</i> <i>y</i> <i>z</i>


   


 


Do đó



2
3
4


max <sub>3</sub>


2



3 1 8 4 16


16


<i>P</i>  <sub></sub>  <sub></sub> 


 


<b>Bài 9. </b> Cho các số dương , ,<i>a b c</i> thỏa mãn điều kiện <i>abc</i>1. Tìm giá trị nhỏ nhất của biểu thức


2 2 2 2 2 2


<i>bc</i> <i>ca</i> <i>ab</i>


<i>P</i>


<i>a b</i> <i>a c</i> <i>b a</i> <i>b c</i> <i>c a</i> <i>c b</i>


  


  


<b>Lời giải: </b>


2 2 2


( ) ( ) ( )


<i>bc</i> <i>ca</i> <i>ab</i>



<i>P</i>


<i>a b c</i> <i>b a</i> <i>c</i> <i>c a b</i>


  


  


2 2 2


1 1 1


<i>a</i> <i>b</i> <i>c</i>


<i>b c</i> <i>a</i> <i>c</i> <i>a b</i>


<i>bc</i> <i>ac</i> <i>ab</i>


 <sub></sub>  <sub></sub>  <sub></sub> 2 2 2


1 1 1


1 1 1 1 1 1


<i>a</i> <i>b</i> <i>c</i>


<i>b</i> <i>c</i> <i>c</i> <i>a</i> <i>a</i> <i>b</i>


  



  


Đặt <i>x</i> 1
<i>a</i>


 , <i>y</i> 1
<i>b</i>


 , <i>z</i> 1
<i>c</i>


 . Do <i>abc</i> 1 <i>xyz</i>1 và , ,<i>a b c</i> dương suy ra <i>x y z</i>, , dương.
Ta có


2 2 2


<i>x</i> <i>y</i> <i>z</i>


<i>P</i>


<i>y</i> <i>z</i> <i>z</i> <i>x</i> <i>x</i> <i>y</i>


  


  


Áp dụng bất đẳng thức Cơsi, ta có
2



4


<i>x</i> <i>y</i> <i>z</i>


<i>x</i>
<i>y</i> <i>z</i>




 


 ,


2


4


<i>y</i> <i>z</i> <i>x</i>


<i>y</i>
<i>z</i> <i>x</i>




 


 ,


2



4


<i>z</i> <i>x</i> <i>y</i>


<i>z</i>


<i>x</i> <i>y</i>




 


 2


<i>x</i> <i>y</i> <i>z</i>


<i>P</i>   <i>x</i> <i>y</i> <i>z</i>


    


3


3 3


2 2 2


<i>x</i> <i>y</i> <i>z</i>


<i>P</i>   <i>xyz</i>



   


Dấu “=” xảy ra khi <i>x</i>  <i>y</i> <i>z</i> 1 hay <i>a</i>  <i>b</i> <i>c</i> 1. Vậy min


3
2


</div>
<span class='text_page_counter'>(5)</span><div class='page_container' data-page=5>

5 DAYHOCTOAN.VN
<b>Bài 10. </b> Cho 3 số dơng <i>a b c</i>, , thoả mãn : 3


4


<i>a b c</i>   . Tìm giá trị nhỏ nhất của biểu thức


3 3 3


1 1 1


3 3 3


<i>P</i>


<i>a</i> <i>b</i> <i>b</i> <i>c</i> <i>c</i> <i>a</i>


  


  


<b>Lời giải: </b>



áp dụng BĐT Cơsi cho 3 số dương ta có


3


3


1 1 1 3


3 . 9


<i>x</i> <i>y</i> <i>z</i> <i>xyz</i>


<i>x</i> <i>y</i> <i>z</i> <i>xyz</i>


 


  <sub></sub>   <sub></sub> 


 


1 1 1 9


<i>x</i> <i>y</i> <i>z</i> <i>x</i> <i>y</i> <i>z</i>


   


  (*)
áp dụng (*) ta có


3 3 3



1 1 1


3 3 3


<i>P</i>


<i>a</i> <i>b</i> <i>b</i> <i>c</i> <i>c</i> <i>a</i>


  


   3 3 3


9


3 3 3


<i>a</i> <i>b</i> <i>b</i> <i>c</i> <i>c</i> <i>a</i>




    


áp dụng BĐT Cô si cho 3 số dương ta có
3<sub>(</sub> <sub>3 ).1.1</sub> 3 1 1 1<sub>(</sub> <sub>3</sub> <sub>2)</sub>


3 3


<i>a</i> <i>b</i>



<i>a</i> <i>b</i>      <i>a</i> <i>b</i>


3<sub>(</sub> <sub>3 ).1.1</sub> 3 1 1 1<sub>(</sub> <sub>3</sub> <sub>2)</sub>


3 3


<i>b</i> <i>c</i>


<i>b</i> <i>c</i>      <i>b</i> <i>c</i>


3<sub>(</sub> <sub>3 ).1.1</sub> 3 1 1 1<sub>(</sub> <sub>3</sub> <sub>2)</sub>


3 3


<i>c</i> <i>a</i>


<i>c</i> <i>a</i>      <i>c</i> <i>a</i>


3 3 3 1


3 3 3 [4( ) 6] 3


3


<i>a</i> <i>b</i> <i>b</i> <i>c</i> <i>c</i> <i>a</i> <i>a</i> <i>b</i> <i>c</i>


          


3
<i>P</i>


 


Dấu “=” xảy ra khi và chỉ khi


3
4


3 3 3


<i>a b c</i>


<i>a</i> <i>b</i> <i>b</i> <i>c</i> <i>c</i> <i>a</i>


   




     


1
4


<i>a</i> <i>b</i> <i>c</i>


   
Vậy minP = 3


<b>Bài 11. </b> Cho các số dương , ,<i>a b c</i> thoả mãn <i>a b c</i>  3. Chứng minh rằng:



2 2 2


1 1 1


3


1 1 1


<i>a</i> <i>b</i> <i>c</i>


<i>b</i> <i>c</i> <i>a</i>


 <sub></sub>  <sub></sub>  <sub></sub>


  


<b>Lời giải: </b>
<b>: </b>


Bất đẳng thức trên tương đương với:


2 2 2


1 1 1


1 1 1 3 3 3


1 1 1



<i>a</i> <i>b</i> <i>c</i>


<i>a</i> <i>b</i> <i>c</i> <i>a b c</i>


<i>b</i> <i>c</i> <i>a</i>


  


             


  


Hay



2 2 2


2 2 2


1 . 1 . 1 .


3


1 1 1


<i>a</i> <i>b</i> <i>b</i> <i>c</i> <i>c</i> <i>a</i>


<i>b</i> <i>c</i> <i>a</i>


  



  


   .


Bây giờ ta dùng bất đẳng thức AM – GM cho các mẫu thức:


2

2

2

2

2

2


2 2 2


1 1 1 1 1 1


1 1 1 2 2 2


<i>a</i> <i>b</i> <i>b</i> <i>c</i> <i>c</i> <i>a</i> <i>a</i> <i>b</i> <i>b</i> <i>c</i> <i>c</i> <i>a</i>


<i>b</i> <i>c</i> <i>a</i> <i>b</i> <i>c</i> <i>a</i>


     


    


  


1

1

 

1



2 2 2


<i>a</i> <i>b</i> <i>b</i> <i>c</i> <i>c</i> <i>a</i>



</div>
<span class='text_page_counter'>(6)</span><div class='page_container' data-page=6>

6 DAYHOCTOAN.VN


3


3
2


<i>ab bc</i> <i>ca</i>


  


  Vì



2
3
3


<i>a b c</i>
<i>ab bc ca</i>     


<b>Bài 12. </b> Chứng minh rằng điều kiện cần và đủ để tam giác ABC là tam giác đều là


2 2 2 1


2


2 2 2 4 2 2 2


<i>A</i> <i>B</i> <i>C</i> <i>A B</i> <i>B C</i> <i>C</i> <i>A</i>



<i>cos</i> <i>cos</i> <i>cos</i>   <i>cos</i>  <i>cos</i>  <i>cos</i>  (*)


<b>Lời giải: </b>


2 2 2


(*) 4 4 4 8


2 2 2 2 2 2


<i>A</i> <i>B</i> <i>C</i> <i>A B</i> <i>B C</i> <i>C</i> <i>A</i>


<i>cos</i> <i>cos</i> <i>cos</i> <i>cos</i>  <i>cos</i>  <i>cos</i> 


    




2 cos cos cos 1


2 2 2


<i>A B</i> <i>B C</i> <i>C</i> <i>A</i>


<i>A</i> <i>A</i> <i>A</i> <i>cos</i>  <i>cos</i>  <i>cos</i> 


    


8sin sin sin



2 2 2 2 2 2


<i>A</i> <i>B</i> <i>C</i> <i>A B</i> <i>B C</i> <i>C</i> <i>A</i>


<i>cos</i>  <i>cos</i>  <i>cos</i> 


 


(Nhân 2 vế với 8


2 2 2


<i>A</i> <i>B</i> <i>C</i>


<i>cos</i> <i>cos</i> <i>cos</i> )








8sin .sin .sin<i>A</i> <i>B</i> <i>C</i> sin<i>A</i> sin<i>B</i> sin<i>B</i> sin<i>C</i> sin<i>C</i> sin<i>A</i>


    


sin<i>A</i> sin<i>B</i> sin<i>C</i>


   (áp dụng BĐT Côsi)


<i>A</i> <i>B</i> <i>C</i>


   (ĐPCM).



<b>Bài 13. </b> Chứng minh rằng nếu <i>x</i>, <i>y</i>, <i>z</i> là ba số dương tuỳ ý thì với mọi tam giác <i>ABC</i>, ta đều có:
<b>a) </b>1cos 1cos 1cos


2 2 2


<i>x</i> <i>y</i> <i>z</i>


<i>A</i> <i>B</i> <i>C</i>


<i>x</i>  <i>y</i>  <i>z</i>  <i>yz</i> <i>xz</i> <i>xy</i> .


<b>b) Tam giác </b><i>ABC</i> có đặc điểm gì nếu thoả mãn điều kiện: cos cos cos 2
2


<i>C</i>


<i>A</i> <i>B</i>  .


<b>Lời giải </b>


<b>a) </b>1cos 1cos 1cos


2 2 2


<i>x</i> <i>y</i> <i>z</i>


<i>A</i> <i>B</i> <i>C</i>


<i>x</i>  <i>y</i>  <i>z</i>  <i>yz</i> <i>xz</i> <i>xy</i> .



Bất đẳng thức tương đương với:


2 2 2


2 cos 2 cos 2 cos 0


<i>x</i> <i>y</i> <i>z</i>  <i>yz</i> <i>A</i> <i>xz</i> <i>B</i> <i>xy</i> <i>C</i>




2 2 2 2 2 2 2


cos sin cos sin 2 cos 2 cos 2 cos 0


<i>x</i> <i>B</i> <i>B</i> <i>y</i> <i>A</i> <i>A</i> <i>z</i> <i>yz</i> <i>A</i> <i>xz</i> <i>B</i> <i>xy</i> <i>A B</i>


         


 





2 2 2 2 2 2 2 2


2


cos cos 2 cos cos sin sin 2 sin sin


2 cos cos 0



<i>x</i> <i>B</i> <i>y</i> <i>A</i> <i>xy</i> <i>A</i> <i>B</i> <i>x</i> <i>B</i> <i>y</i> <i>A</i> <i>xy</i> <i>A</i> <i>B</i>


<i>z</i> <i>z y</i> <i>A</i> <i>x</i> <i>B</i>


      


    .


 

2

2


cos cos sin sin 0


<i>x</i> <i>B</i> <i>A z</i> <i>x</i> <i>B</i> <i>y</i> <i>A</i>


      (luôn đúng).


Dấu bằng xảy ra khi và chỉ khi hệ sau đây thoả mãn:


cos cos 0


sin sin 0


<i>x</i> <i>B</i> <i>y</i> <i>A z</i>


<i>x</i> <i>B</i> <i>y</i> <i>A</i>


  





 <sub></sub> <sub></sub>




sin


cos cos


sin
sin sin


<i>x</i> <i>B</i>


<i>x</i> <i>B</i> <i>A</i> <i>z</i>


<i>A</i>


<i>x</i> <i>y</i>


<i>A</i> <i>B</i>


 <sub></sub> <sub></sub>



 


 <sub></sub>








sin sin


sin sin


<i>x</i> <i>A B</i> <i>z</i> <i>A</i>


<i>x</i> <i>y</i>


<i>A</i> <i>B</i>


 




 






sin sin


sin sin


<i>x</i> <i>C</i> <i>z</i> <i>A</i>


<i>x</i> <i>y</i>


<i>A</i> <i>B</i>







 




 sin sin sin


<i>x</i> <i>y</i> <i>z</i>


<i>A</i> <i>B</i> <i>C</i>


</div>
<span class='text_page_counter'>(7)</span><div class='page_container' data-page=7>

7 DAYHOCTOAN.VN


Tức là tam giác <i>ABC</i> đồng dạng với tam giác có ba cạnh là <i>x</i>, <i>y</i>, <i>z</i>.


<b>b) Tam giác </b><i>ABC</i> có đặc điểm gì nếu thoả mãn điều kiện: cos cos cos 2
2


<i>C</i>


<i>A</i> <i>B</i>  .


Áp dụng phần a với <i>x</i> <i>y</i> 1, <i>z</i> 2.


Nhận thấy 2



2 2 2


<i>x</i> <i>y</i> <i>z</i>


<i>yz</i> <i>zx</i> <i>xy</i>  . Theo kết quả câu a thì <i>ABC</i> đồng dạng với tam giác có ba
cạnh là 1, 1, 2. Nghĩa là tam giác <i>ABC</i> vuông cân tại <i>C</i>.


<b>Bài 14. </b> Cho ba số dương <i>a</i><sub>, </sub><i>b</i><sub>, </sub><i>c</i><sub> thoả mãn </sub><i>abc</i>1<sub>. Chứng minh rằng: </sub>


1 1 1


1


1 1 1


<i>a</i> <i>b</i> <i>b</i> <i>c</i> <i>c</i> <i>a</i>  .


<b>Lời giải </b>


3 3

3 2 3 3

3

3 3



<i>a b</i>  <i>a</i> <i>b</i> <i>a</i>  <i>ab</i> <i>b</i>  <i>ab</i> <i>a</i> <i>b</i>




3 3 3


1 1


<i>a b</i> <i>ab</i> <i>a</i> <i>b</i>



      3

3 3

3


<i>ab</i> <i>a</i> <i>b</i> <i>abc</i>


   3

3 3 3



<i>ab</i> <i>a</i> <i>b</i> <i>c</i>


  




3 3


3 3 3


3 3 3 3 3 3 3 3


1 1


1


<i>abc</i> <i>c</i>


<i>a b</i> <i>ab</i> <i>a</i> <i>b</i> <i>c</i> <i>ab</i> <i>a</i> <i>b</i> <i>c</i> <i>a</i> <i>b</i> <i>c</i>


   


        .



Tương tự 3


3 3 3


1
1


<i>a</i>


<i>b c</i>   <i>a</i> <i>b</i> <i>c</i> ,


3


3 3 3


1
1


<i>b</i>


<i>c a</i>   <i>a</i> <i>b</i> <i>c</i> .


Vậy 1 1 1 1


1 1 1


<i>a</i> <i>b</i> <i>b</i> <i>c</i> <i>c</i> <i>a</i>  .
Đẳng thức xảy ra khi <i>a</i>  <i>b</i> <i>c</i> 1.



<b>Bài 15. </b> Cho <i>ABC</i> không tù. Chứng minh rằng: tan tan tan tan .tan .tan 10 3


2 2 2 2 2 2 9


<i>A</i><sub></sub> <i>B</i><sub></sub> <i>C</i><sub></sub> <i>A</i> <i>B</i> <i>C</i> <sub></sub>


. Dấu
bằng xảy ra khi nào?


<b>Lời giải </b>


Đặt tan
2


<i>A</i>


<i>x</i> , tan


2


<i>B</i>


<i>y</i> , tan


2


<i>C</i>
<i>z</i>





, , 0;1
1
3
<i>x y z</i>


<i>xy</i> <i>yz</i> <i>zx</i>
<i>x</i> <i>y</i> <i>z</i>


 




<sub></sub>   


   


.


Áp dụng BĐT Cơsi cho ba só không âm 1<i>x</i>, 1<i>y</i>, 1<i>z</i> ta được:

1

 

1

 

1

<sub>3</sub>

<sub></sub>

<sub></sub>

<sub></sub>

<sub></sub>



1 1 1


3


<i>x</i> <i>y</i> <i>z</i>


<i>x</i> <i>y</i> <i>z</i>



    


     3<sub>1</sub>   

<i><sub>x</sub></i> <i><sub>y</sub></i> <i><sub>z</sub></i>

<i><sub>xy</sub></i><i><sub>yz</sub></i><i><sub>zx</sub></i><i><sub>xyz</sub></i>




3<sub>2</sub> <i><sub>x</sub></i> <i><sub>y</sub></i> <i><sub>z</sub></i> <i><sub>xyz</sub></i>


     .


3
3


3 10 3


2 1 2 1


3 3 9


<i>x</i> <i>y</i> <i>z</i>


<i>x</i> <i>y</i> <i>z</i> <i>xyz</i>      


     <sub></sub>  <sub></sub>  <sub></sub><sub></sub>  <sub></sub><sub></sub> 


  <sub></sub> <sub></sub> (ĐPCM).


Dấu bằng xảy ra khi 3
3


<i>x</i>  <i>y</i> <i>z</i> .


<b>Bài 16. </b> Cho các số thực dương <i>x</i>, <i>y</i>, <i>z</i> thoả mãn <i>x</i>  <i>y</i> <i>z</i> <i>xyz</i>. Chứng minh rằng:


2 2 2


3 1 1 1


<i>xy</i><i>yz</i><i>zx</i>  <i>x</i>   <i>y</i>   <i>z</i>  .


</div>
<span class='text_page_counter'>(8)</span><div class='page_container' data-page=8>

8 DAYHOCTOAN.VN
Ta có <i>xyz</i>   <i>x</i> <i>y</i> <i>z</i> 2 <i>xy</i><i>z</i>

 



2


2 0


<i>z</i> <i>xy</i> <i>xy</i> <i>z</i>


    <i>xy</i> 1 1 <i>z</i>2


<i>z</i>
 


  (ta loại trường


hợp


2
1 1 <i>z z</i>



<i>xy</i>    vì ,<i>x y</i>0).


Với


2
1 1 <i>z</i>
<i>xy</i>


<i>z</i>
 


 , ta có:



2


2
1 1


2 2 . <i>z</i> 2 1 1


<i>z x</i> <i>y</i> <i>z xy</i> <i>z</i> <i>z</i>


<i>z</i>
 


      .


Nên <i>z x</i>

<i>y</i>

2 1

 1<i>z</i>2

.



Tương tự, ta có

2


2 1 1


<i>x z</i><i>y</i>   <i>x</i> , <i>y x</i>

<i>z</i>

2 1

 1<i>y</i>2

.


Cộng theo vế ba bất đẳng thức trên ta được 2 2 2


3 1 1 1


<i>xy</i><i>yz</i><i>zx</i>  <i>x</i>  <i>y</i>  <i>z</i> (ĐPCM)
<b>Bài 17. </b> Tính các góc của tam giác <i>ABC</i>, biết các cạnh <i>a</i>, <i>b</i>, <i>c</i> và các góc <i>A</i>, <i>B</i>, <i>C</i> thoả mãn hệ thức:




4


2 3 3
sin sin sin


2 2 2 8


<i>p p</i> <i>a</i> <i>bc</i>


<i>A</i> <i>B</i> <i>C</i>


 






 <sub></sub>







với <i>p</i> là nửa chu vi tam giác <i>ABC</i>.


<b>Lời giải: </b>




4<i>p p a</i> <i>bc</i> 

<i>a b c b c a</i> 



 

<i>bc</i> 

<i>b c</i>

2<i>a</i>2<i>bc</i> <i>b</i>2 <i>c</i>2 <i>a</i>22<i>bc</i><i>bc</i>


1


cos 1


2


<i>A</i>


   2 1


cos


2 4


<i>A</i>



  2 3


sin


2 4


<i>A</i>


  sin 3


2 2
<i>A</i>


  .


1


sin sin sin sin cos cos


2 2 2 2 2 2 2


<i>A</i> <i>B</i> <i>C</i> <i>A</i> <i>B C</i> <i>B C</i> 


 <sub></sub>  <sub></sub>


 


1



sin 1 cos


2 2 2


<i>A</i> <i>B C</i> 


 <sub></sub>  <sub></sub>


 


2


1 1 1


sin


8 2 2 2


<i>A</i>


 


  <sub></sub>  <sub></sub>


 


2


1 1 3 1 2 3 3



8 2 2 2 8


  <sub></sub>


  <sub></sub><sub></sub>  <sub></sub><sub></sub> 


  .


Dấu bằng xảy ra khi


3
sin


2 2


cos 1


2
<i>A</i>


<i>B C</i>











 <sub></sub>





2
3


6
<i>A</i>


<i>B</i> <i>C</i>




 

 


  



.


<b>Bài 18. </b> Cho , ,<i>a b c</i>0 thoả 21<i>ab</i>2<i>bc</i>8<i>ca</i>12. Tìm giá trị nhỏ nhất của biểu thức <i>P</i> 1 2 3


<i>a</i> <i>b</i> <i>c</i>


   .


<b>Lời giải </b>



Đặt <i>x</i> 1
<i>a</i>


 , <i>y</i> 2
<i>b</i>


 , <i>z</i> 3
<i>c</i>


 . Điều kiện bài toán trở thành , , 0


2 2 4 7


<i>x y z</i>


<i>xyz</i> <i>x</i> <i>y</i> <i>z</i>





 <sub></sub> <sub></sub> <sub></sub>


 .


Bài toán quy về tìm giá trị nhỏ nhất của <i>P</i>  <i>x</i> <i>y</i> <i>z</i>.


Từ 2<i>xyz</i>2<i>x</i>4<i>y</i>7<i>z</i> <i>z</i>

2<i>xy</i>7

2<i>x</i>4<i>y</i>


2 7



2 4


2 7


<i>xy</i>


<i>x</i> <i>y</i>


<i>z</i>
<i>xy</i>




<sub> </sub> 


 <sub></sub>




.


Khi đó


14
2


2 4 11 7



2 7 2 2 2 7


<i>x</i>


<i>x</i> <i>y</i> <i><sub>x</sub></i>


<i>P</i> <i>x</i> <i>y</i> <i>x</i> <i>y</i>


<i>xy</i> <i>x</i> <i>x</i> <i>xy</i>





       


  2


11 7


2 1
2


<i>x</i>


<i>x</i> <i>x</i>


</div>
<span class='text_page_counter'>(9)</span><div class='page_container' data-page=9>

9 DAYHOCTOAN.VN
Dễ dàng chứng minh được: <sub>2</sub>


7


3
7
2 1


2
<i>x</i>
<i>x</i>




  .


7
3


11 3 9 3 9 11


2 .


2 2 2 2 2


<i>x</i>


<i>P</i> <i>x</i> <i>x</i> <i>x</i>


<i>x</i> <i>x</i> <i>x</i>




         .



Vậy min 15
2


<i>P</i> khi 1


3


<i>a</i> , 4


5


<i>b</i> , 3


2


<i>c</i> .
Dễ dàng CM được <sub>2</sub>


7
3
7
2 1


2
<i>x</i>
<i>x</i>





  .


<b>Bài 19. </b> Cho hai số <i>x</i>, <i>y</i> thoả <i>x</i>24<i>y</i>2 4. Tìm giá trị nhỏ nhất, giá trị lớn nhất của biểu thức


2 2


4 3 2


<i>M</i>  <i>x</i>  <i>xy</i> <i>y</i> .


<b>Lời giải </b>


Từ


2


2 2 2


4 4 1


2


<i>x</i>


<i>x</i>  <i>y</i>   <sub> </sub> <i>y</i> 


  .


Đặt sin
2



<i>x</i> <sub></sub>


 , <i>y</i>cos.


2 2


16sin 6 sin cos 2 cos 3sin 2 7 cos 2 9


<i>M</i>           .


Áp dụng BĐT Bunhiacopki ta có <i>M</i> đạt giá trị lớn nhất là 9 58 và giá trị nhỏ nhất là 9 58.


Cách 2:



2 2


2 2


4 4 3 2


4


<i>x</i> <i>xy</i> <i>y</i>


<i>M</i>


<i>x</i> <i>y</i>


 





 sử dụng điều kiện có nghiệm của phương tình bậc hai, ta có kết
quả trên.


<b>Bài 20. </b> Cho , ,<i>a b c</i>0 và <i>ab bc ca</i>  1. Chứng minh rằng:


2 2 2 <sub>2</sub> <sub>2</sub> <sub>2</sub>


2 2 2 1 1 1


1 1 1 <sub>1</sub> <sub>1</sub> <sub>1</sub>


<i>a</i> <i>b</i> <i>c</i>


<i>a</i>  <i>b</i>  <i>c</i>   <i><sub>a</sub></i>   <i><sub>b</sub></i>   <i><sub>c</sub></i>  .


<b>Lời giải </b>


Từ điều kiện đầu bài, ta đặt tan
2


<i>A</i>


<i>a</i> , tan


2


<i>B</i>



<i>b</i> , tan


2


<i>C</i>


<i>c</i> và <i>A</i>, <i>B</i>, <i>C</i> là ba góc trong một tam
giác.


Từ <sub>2</sub> <sub>2</sub> <sub>2</sub>


2 2 2


2 2 2 1 1 1


1 1 1 <sub>1</sub> <sub>1</sub> <sub>1</sub>


<i>a</i> <i>b</i> <i>c</i>


<i>a</i>  <i>b</i>  <i>c</i>   <i><sub>a</sub></i> <sub></sub>  <i><sub>b</sub></i> <sub></sub>  <i><sub>c</sub></i> <sub></sub>


sin sin sin cos cos cos


2 2 2


<i>A</i> <i>B</i> <i>C</i>


<i>A</i> <i>B</i> <i>C</i>



     

 

* .


Mặt khác ta có: sin sin 2 sin cos 2 sin 2 cos


2 2 2 2


<i>A</i> <i>B</i> <i>A</i> <i>B</i> <i>A</i> <i>B</i> <i>C</i>


<i>A</i> <i>B</i>      ,


sin sin 2 cos
2


<i>A</i>


<i>B</i> <i>C</i> , sin sin 2 cos


2


<i>B</i>


<i>C</i> <i>A</i> .


Cộng các bất đẳng thức trên, ta chứng minh được

 

* .
<b>Bài 21. </b> Cho tam giác <i>ABC</i> có các góc thoả mãn


2


</div>
<span class='text_page_counter'>(10)</span><div class='page_container' data-page=10>

10 DAYHOCTOAN.VN



<b>Lời giải </b>


Ta có:


2


<i>A</i>  <i>B</i> <i>C</i> 


3 <i>C</i> 2


 


   0 cos 1


2


<i>C</i>


   .


 



cos 2<i>A</i>cos 2<i>B</i>2 cos <i>A B</i> cos <i>A B</i>  2 cos<i>C</i>cos <i>A B</i>  2 cos<i>C</i>(do cos<i>C</i>0 và




cos <i>A B</i> 1)


Dấu bằng xảy ra khi <i>A</i><i>B</i> hoặc



2


<i>C</i> .


Từ đó

2

 

2

2


4 2 cos 1 2 2 cos 1 1 2 cos


<i>P</i> <i>C</i>  <sub></sub> <i>C</i>  <sub></sub> <i>C</i>


  8cos2<i>C</i>

2cos2<i>C</i> 1

2cos<i>C</i>


2



4 2 2


16 cos <i>C</i> 8cos <i>c</i> 1 1 2 cos<i>C</i> 4 4 cos <i>C</i> 1 1 2 cos<i>C</i> 4 4


             .


Dấu bằng xảy ra khi


3


<i>C</i> .
Vậy <i>P</i> đạt giá trị nhỏ nhất khi


3


<i>A</i>  <i>B</i> <i>C</i>  .



<b>Bài 22. </b> Cho ba số dương <i>a</i> , <i>b</i>, <i>c</i> thoả mãn <i>ab</i> 1 <i>c a b</i>

. Tìm giá trị lớn nhất của biểu thức


2 2 2


1 1 1


<i>a</i> <i>b</i> <i>c</i>


<i>P</i>


<i>a</i> <i>b</i> <i>c</i>


  


   .


<b>Lời giải </b>


Đặt <i>a</i>tan, <i>b</i>tan , <i>c</i> tan với , 0;
2

 <sub></sub> <sub></sub>


 , 2;0

 <sub></sub> <sub></sub>


 .
Từ giả thiết, ta có:





1 tan .tan 1 tan tan tan


<i>ab</i> <i>c a b</i>        


1 tan tan
tan 1 tan . tan


 


  




 


 tan  2 tan

 



 


 <sub></sub>  <sub></sub> 


  .


Đặt <i>a</i>tan, <i>b</i>tan, <i>c</i> tan với , 0; , ;0


2 2



 


 <sub></sub> <sub></sub>  <sub></sub> <sub></sub>


   .


2 <i>k</i>




   


     , vì


2


    
     nên


2



     .


Mà <i>P</i>tan .cos 2tan .cos 2tan .cos 2 1

sin 2 sin 2 2 sin2



2   


  



 



2

2



1


2 sin cos 2 sin cos cos cos 1


2


<i>P</i>              


2 <sub>2</sub>



cos cos 5


cos 1


2 4 4


<i>P</i> <sub></sub>    <sub></sub>     


  .


Đẳng thức xảy ra khi và chỉ khi







2


cos 1


cos


cos 0


2


 
 


  




 


 





1
cos


2
 







 






5
12
3



 




  



 
  



.


2 3



<i>a</i> <i>b</i>


</div>
<span class='text_page_counter'>(11)</span><div class='page_container' data-page=11>

11 DAYHOCTOAN.VN
Vậy GTLN của <i>P</i> là 5


4 khi <i>a</i>  <i>b</i> 2 3, <i>c</i> 3.


<b>Bài 23. </b> cho , ,<i>x y z</i> 0. Chứng minh rằng


2



 

2



 

3



53


<i>xy</i> <i>yz</i> <i>zx</i>


<i>P</i>


<i>z</i> <i>x</i> <i>z</i> <i>y</i> <i>x</i> <i>y</i> <i>x</i> <i>z</i> <i>y</i> <i>z</i> <i>y</i> <i>x</i>


   


      .


<b>Lời giải </b>


Đặt <i>a</i> <i>y</i> <i>z</i>, <i>b</i> <i>z</i> <i>x</i>, <i>c</i> <i>x</i> <i>y</i>.


Khi đó <i>a</i>, <i>b</i> , <i>c</i> là ba cạnh của tam giác <i>ABC</i>.


Ta có:









2
2


4 4


<i>b c a c a b</i> <i>c</i> <i>a b</i>


<i>xy</i>


<i>z</i> <i>x</i> <i>z</i> <i>y</i> <i>ab</i> <i>ab</i>


     


 


 


2 2 2


1 1 1


cos


4 2 2 2


<i>c</i> <i>a</i> <i>b</i>


<i>C</i>
<i>ab</i>



 


     .


Tương tự




12cos 12
<i>yz</i>


<i>A</i>


<i>x</i><i>y</i> <i>x</i><i>z</i>    ,

 



1 1


cos


2 2


<i>zx</i>


<i>B</i>
<i>y</i> <i>z</i> <i>y</i><i>x</i>    .
Suy ra cos cos 3cos 7


2 2


<i>P</i> <sub></sub> <i>C</i> <i>A</i> <i>B</i><sub></sub>



  .


Ta có cos cos 3cos 2 cos cos 3cos


2 2 2 2


<i>A C</i> <i>A C</i>


<i>C</i> <i>A</i> <i>B</i>    <i>B</i>


2
3


2sin cos 1 2sin


2 2 2 2


<i>B</i> <i>A C</i>  <i>B</i>


  <sub></sub>  <sub></sub>


 


2 3


3sin 2 sin


2 2 2


<i>B</i> <i>B</i>



   


2


2 11 11


3 sin


2 3 6 6


<i>B</i>


 


  <sub></sub>  <sub></sub>  


  .


Suy ra 7 11 5


2 6 3


<i>P</i>   .


Dấu bằng xảy ra khi


2 4


sin



2 3 3


<i>B</i>


<i>b</i> <i>a</i>


<i>A</i> <i>C</i> <i>a</i> <i>c</i>


 <sub></sub>  <sub></sub>


 <sub></sub>


 


 <sub></sub>  <sub></sub>


 


2


<i>x</i> <i>y</i> <i>z</i>


   .


<b>Bài 24.</b> <b><sub>Câu 1. </sub></b> <sub>Cho </sub> , , 0


6
<i>a b c</i>
<i>a b c</i>






   


 Tìm GTNN của


2 1 2 1 2 1


<i>S</i> <i>a</i> <i>b</i> <i>c</i>


<i>b c</i> <i>a c</i> <i>a b</i>


     


  


<b>Bài giải: </b>


Áp dụng bất đẳng thức Bunhiacơpxki ta có:


2 1 2 2 2 1


( ) (4 1 ) 4


<i>a</i> <i>a</i>


<i>b c</i> <i>b c</i>



 <sub></sub>  <sub></sub> <sub></sub> <sub></sub>


 <sub></sub>  <sub></sub>


 


2 1 2 2 2 1


( ) (4 1 ) 4


<i>b</i> <i>b</i>


<i>a</i> <i>c</i> <i>a</i> <i>c</i>


 <sub></sub>  <sub></sub> <sub></sub> <sub></sub>


 <sub></sub>  <sub></sub>


 


2 1 2 2 2 1


( ) (4 1 ) 4


<i>c</i> <i>c</i>


<i>b</i> <i>a</i> <i>b</i> <i>a</i>


 <sub></sub>  <sub></sub> <sub></sub> <sub></sub>



 <sub></sub>  <sub></sub>


 


Cộng theo vế 3 bất đẳng thức trên ta đợc: <i>S</i>. 17 4(<i>a b c</i>) ( 1 1 1 )


<i>a b</i> <i>b c</i> <i>a c</i>


     


  


3


3


4( )


. .


<i>a b c</i>


<i>a b b c</i> <i>c a</i>


   


   ( theo bất đẳng thức Côsi)
9


4(<i>a b c</i>)



<i>a b</i> <i>b c</i> <i>a c</i>


   


</div>
<span class='text_page_counter'>(12)</span><div class='page_container' data-page=12>

12 DAYHOCTOAN.VN




2 2 2


9


4( )


(1 1 1 ) ( ) ( ) ( )
<i>a b c</i>


<i>a b</i> <i>a c</i> <i>b c</i>


   


      


9


4( )


6( )



<i>a b c</i>


<i>a b c</i>


   


 


31 9 9


( )


8 8 2 6( ) 2 6( )


<i>a b c</i>
<i>a b c</i>


<i>a b c</i> <i>a b c</i>


 


     


   


2


31 3 93 9 51


.6 3.



8 2 4 2




   


51
2 17
<i>S</i>


  3 17


2
 .


Vậy ưMin 3 17
2


<i>S</i>  khi <i>a</i>  <i>b</i> <i>c</i> 2.


<b>Bài 25. </b> <b>Câu 2. </b> Chứng minh rằng tam giác <i>ABC</i> đều nếu thoả mãn:


2
9
tan tan tan


2 2 2 4


<i>A</i> <i>B</i> <i>C</i> <i>R</i>



<i>S</i>


   .


<b>Bài giải: </b>


Trong đó <i>S</i>, <i>R</i> lần lượt là diện tích và bán kính đường trịn ngoại tiếp <i>ABC</i> .
Ta có 1 sin


2


<i>S</i>  <i>ab</i> <i>C</i> 2 sin sin sin<i>R</i> <i>A</i> <i>B</i> <i>C</i> thế vào đẳng thức đã cho ta được:
tan tan tan 8 sin si


(   ) <i>R</i> <i>A</i> n<i>B</i>sin<i>C</i> 9<i>R</i>


 2 2 2 9


4 sin sin sin 4 sin sin sin 4 sin sin sin


2 2 2 4


<i>A</i> <i>B</i> <i>C</i>


<i>B</i> <i>C</i> <i>A</i> <i>C</i> <i>A</i> <i>B</i>


2 sin

<i>B</i>sin<i>C</i>sin<i>A</i>sin<i>C</i>sin<i>A</i>sin<i>B</i>



9


2 sin sin cos sin sin cos sin sin cos


4


<i>B</i> <i>C</i> <i>A</i> <i>A</i> <i>C</i> <i>B</i> <i>A</i> <i>B</i> <i>C</i>


   


Theo BĐT Côsi: 2 2 2


sin s


sin <i>A</i>sin <i>B</i>sin <i>C</i> <i>A</i> in<i>B</i>sin<i>B</i>sin<i>C</i>sin<i>A</i>sin<i>C</i>
Nên


2 2 2

 

2 2 2

2 2 2 9


2 sin sin sin sin sin sin sin sin sin


4


<i>VT</i>  <i>A</i> <i>B</i> <i>C</i>  <i>A</i> <i>B</i> <i>C</i> <i>A</i> <i>B</i> <i>C</i> (*)


Dấu đẳng thức xảy ra khi <i>ABC</i> đều.
Chứng minh BĐT (*):


Ta có sin2 sin2 sin2 1 cos 2 1 cos 2 sin2


2 2



<i>A</i> <i>B</i>


<i>M</i>  <i>A</i> <i>B</i> <i>C</i>     <i>C</i>


 

2


1 cos <i>A B</i> cos <i>A B</i> 1 cos <i>C</i>


     




2


2 cos <i>C</i> cos<i>C</i>cos <i>A B</i>


   


Xét

cos2 c s

1
4
9


4 <i>C</i> o <i>C</i>cos <i>B</i>


<i>M</i>    <i>A</i> 


2 2



1 1



cos cos sin 0


2 4


<i>C</i> <i>A B</i> <i>A B</i>


 


 <sub></sub>   <sub></sub>   


 


Dấu đẳng thức xảy ra khi <i>ABC</i> đều. Suy ra điều phải chứng minh.


<b>Bài 26. </b> <b>Câu 3. </b> Tìm giá trị nhỏ nhất của biểu thức <i>A</i>2 11 2 <i>y</i>4 <i>x</i> <i>y</i> 5<b>, </b>với x, y là các số thực


thoả mãn 2 2


– 2 – 6 6 0


</div>
<span class='text_page_counter'>(13)</span><div class='page_container' data-page=13>

13 DAYHOCTOAN.VN


<b>Bài giải: </b>


Ta thấy 2 2 <sub>– 2 – 6</sub> <sub>6 0</sub>


<i>x</i> <i>y</i> <i>x</i> <i>y</i>  là phương trình của đường trịn

 

<i>C</i> tâm<i>I</i>

 

1;3 , bán kính <i>R</i>2.
Vì <i>x y</i>, thoả mãn <i>x</i>2<i>y</i>2 – 2 – 6<i>x</i> <i>y</i> 6 0 nên ta có



2 11 2 4 5


<i>A</i>  <i>y</i> <i>x</i> <i>y</i> =2( 11 2 <i>y</i>+ 4<i>x</i>4<i>y</i>20)


=2( (<i>x</i>2<i>y</i>22<i>x</i>6<i>y</i> 6) (11 2 ) <i>y</i> + (<i>x</i>2<i>y</i>22<i>x</i>6<i>y</i> 6) (4<i>x</i>4<i>y</i>20)) = = 2(


2 2


(<i>x</i>1) (<i>y</i>4) + 2 2


(<i>x</i>1) (<i>y</i>5) )2

<i>NM</i> <i>PM</i>

, trong đó <i>N</i>

 

1; 4 nằm bên trong

 

<i>C</i> ,

1;5



<i>P</i>  nằm bên ngoài

 

<i>C</i> ,<i>M x y</i>

   

;  <i>C</i> .
Gọi <i>M<sub>o</sub></i> là giao điểm của đoạn thẳng <i>PN</i> với

 

<i>C</i> 


( )
vµ cïng h-íng


<i>o</i>


<i>NM<sub>o</sub></i> <i>NP</i>
<i>M</i>  <i>C</i>



  



 toạ độ của điểm <i>Mo</i> là nghiệm của hệ



2 2


- 2 - 6 6 0


1 4


0


2 1


<i>x</i> <i>y</i> <i>x</i> <i>y</i>


<i>x</i> <i>y</i>


   




  


 


 






1
5


23


5
<i>x</i>


<i>y</i>
  





 


1 23
;
5 5


<i>o</i>


<i>M</i> <sub></sub> <sub></sub>
  .


Với mọi <i>M x y</i>

   

;  <i>C</i> ta thấy<i>NM</i> <i>PM</i> <i>PN</i>  5, dấu “=” xảy ra khi <i>M x y</i>

 

; 1 23;
5 5


<i>o</i>


<i>M</i>  



 <sub></sub> <sub></sub>


 


=

 

<i>PN</i> ( )<i>C</i> .


Vậy min

 

<i>A</i> 2

<i>NMo</i><i>PM</i>

2.<i>PN</i> 2. 5 , đạt được khi


1
5
23


5
<i>x</i>


<i>y</i>
  





 .


<b>Bài 27. </b> <b>Câu 4. </b> Cho các số thực <i>a b c</i>, , 1, <i>a</i>2<i>b</i>2<i>c</i>2 4. Tìm phần nguyên của


1 1 1


2



<i>a</i> <i>b</i> <i>c</i>


<i>B</i>


<i>a</i> <i>b</i> <i>c</i>


 


    (Phần nguyên của số thực <i>x</i> là số ngun lớn nhất khơng vượt q<i>x</i>, và
được kí hiệu là

 

<i>x</i> ).


<b>Bài giải: </b>


Từ giả thiết suy ra 1<i>a b c</i>, , 2
Nh vậy

2 –<i>a a</i>



–1

 0 1


2


<i>a</i>
<i>a</i>
 ≤ 3


2 . Tương tự
1
2


<i>b</i>
<i>b</i>
 ≤ 3



2 ,
1
2


<i>c</i>
<i>c</i>
 ≤ 3


2.


Do đó ta có 1 1 1


2


<i>a</i> <i>b</i> <i>c</i>


<i>B</i>


<i>a</i> <i>b</i> <i>c</i>


 


    ≤ 9 5


2  (1).


Theo BĐT <b>Cauchy</b> ta có 1


2



<i>a</i>
<i>a</i>


 ≥ 2. .1
2
<i>a</i>


<i>a</i>= 2.
Tơng tự 1


2


<i>b</i>
<i>b</i>


 ≥ 2, và 1


2


<i>c</i>
<i>c</i>


 ≥ 2.
Suy ra B = 1 1 1


2


<i>a</i> <i>b</i> <i>c</i>


<i>a</i> <i>b</i> <i>c</i>



  <sub>  </sub>


</div>
<span class='text_page_counter'>(14)</span><div class='page_container' data-page=14>

14 DAYHOCTOAN.VN


<b>Bài 28. </b> <b>Câu 5. </b> Tìm giá trị lớn nhất của hàm số: <i>f x y z</i>

, ,

 <i>yz x</i> 1 <i>xz y</i> 2 <i>xy z</i> 3
<i>xyz</i>


    


trên miền

( , , ) : 1; 2; 3



<i>D</i> <i>x y z</i> <i>x</i> <i>y</i> <i>z</i>


<b>Bài giải: </b>


, ,



<i>f x y z</i>  <i>x</i> 1 <i>y</i> 2 <i>z</i> 3


<i>x</i> <i>y</i> <i>z</i>




 


 


Vận dụng bất đẳng thức Côsi cho từng cặp số ta có: <i>x</i> 1 <i>x</i> 1.1



<i>x</i> <i>x</i>


 


 1


2


 2 2. 2


. 2


<i>y</i> <i>y</i>


<i>y</i> <i>y</i>


 



1


2 2


 ; 3 3. 3


3


<i>z</i> <i>z</i>



<i>z</i> <i>z</i>


 <sub></sub>  1


2 3


Cộng vế vế các bất đẳng thức ta đợc <i>f x y z</i>

, ,

1(1 1 1 )


2 2 3


  


Đẳng thức xảy ra khi <i>x</i>2,<i>y</i>4,<i>z</i>6
Do đó max <i>f x y z</i>

, ,

 1(1 1 1 )


2  2  3 với

<i>x y z</i>, ,

<i>D</i>
<b>Bài 29. </b> <b>Câu 6. </b> Cho <i>x y</i>, là hai số dương thay đổi, có tổng bằng17


4 . Tìm giá trị nhỏ nhất của biểu thức:


2


2


1 2 11 3
11


1



<i>x</i> <i>y</i>


<i>P</i> <i>x</i> <i>x</i>


<i>y</i> <i>y</i> <i>xy</i>




    


 .


<b>Bài giải: </b>


Ta có:


2


1 1 3


11


1


<i>P</i> <i>x</i> <i>x</i>


<i>y</i> <i>y</i>


<i>x</i>
<i>y</i>



   


<sub></sub>  <sub></sub>  <sub></sub>  <sub></sub>


    <sub></sub> .


Đặt: <i>t</i> <i>x</i> 1 0
<i>y</i>
   .


Ta có:


2


2 3 1 3 1 3 1 47


11 12 2 12 .


2 4 4 4


<i>P</i> <i>t</i> <i>t</i> <i>t</i> <i>t</i> <i>t</i>


<i>t</i> <i>t</i> <i>t</i>


   


    <sub></sub> <sub></sub> <sub></sub>  <sub></sub>   


   



Đẳng thức xảy ra khi 1


2


<i>t</i> . Giải hệ:


17
4
1 1
2
<i>x</i> <i>y</i>


<i>x</i>
<i>y</i>
  



  



được: 1


4


<i>x</i> và<i>y</i>4.


Vậy: Min 47
4



<i>P</i> <b> </b>đạt được khi 1


4


<i>x</i> và <i>y</i>4.


<b>Bài 30. </b> <b>Câu 7. </b> Tìm giá trị lớn nhất, nhỏ nhất của


2 2


2 2


3


<i>x</i> <i>xy</i> <i>y</i>


<i>P</i>


<i>x</i> <i>xy</i> <i>y</i>


 




 


<b>Bài giải: </b>


•<i>y</i>0 thì <i>P</i>1.


• <i>y</i>0 thì


2
2


3 1
1


<i>t</i> <i>t</i>


<i>P</i>


<i>t</i> <i>t</i>
 


  với
<i>x</i>
<i>t</i>


</div>
<span class='text_page_counter'>(15)</span><div class='page_container' data-page=15>

15 DAYHOCTOAN.VN


Gọi <i>P</i> là một giá trị bất kỳ của nó khi đó phương trình sau ẩn <i>t</i> phải có nghiệm.

2

2


1 3 1


<i>P t</i> <i>t</i> <i>t</i> <i>t</i>


      

<sub>1</sub>

2

<sub>3</sub>

 

<sub>– 1</sub>

<sub>0</sub>


<i>P t</i> <i>P t</i> <i>P</i>


     có nghiệm


Hay 1 <sub>2</sub> <sub>2</sub>


(3 ) 4(1 ) 0 (*)
<i>P</i>


<i>P</i> <i>P</i>





     


(*) 3<i>P</i>2 – 6<i>P</i>13 0   

1 3

 <i>P</i> 3 1
Vậy giá trị lớn nhất của <i>P</i>1


Vậy giá trị nhỏ nhất của <i>P</i>  

1 3

.


<b>Bài 31. </b> <b>Câu 8. </b> Cho , ,<i>a b c</i> là ba số thực dương.Chứng minh rằng: <i>a</i> <i>b</i> <i>c</i> 2
<i>b c</i>  <i>a c</i>  <i>b a</i> 


<b>Bài giải: </b>


2
( )



<i>a</i> <i>a</i> <i>a</i>


<i>b c</i>  <i>a b c</i>  <i>a b c</i> 
2


( )


<i>b</i> <i>b</i> <i>b</i>


<i>a c</i>  <i>b a c</i>  <i>a b c</i> 
2


( )


<i>c</i> <i>c</i> <i>c</i>


<i>b a</i>  <i>c b a</i> <i>a b c</i> 


Cộng 3 bất đẳng thức trên vế theo vế ta có điều phải chứng minh
<b>Bài 32. </b> <b>Câu 9. </b> Tìm GTNN của 2 2 2 2


6 12 45 10 16 89


<i>P</i> <i>x</i> <i>y</i>  <i>x</i> <i>y</i>  <i>x</i> <i>y</i>  <i>x</i> <i>y</i> .


<b>Bài giải: </b>


2 2 2 2



6 12 45 10 16 89


<i>P</i> <i>x</i> <i>y</i>  <i>x</i> <i>y</i>  <i>x</i> <i>y</i>  <i>x</i> <i>y</i>
Biến đổi;

 

2

2

 

2

2


3 6 5 8


<i>P</i> <i>x</i>  <i>y</i>  <i>x</i>  <i>y</i>


Trong mặt phẳng toạ độ với hệ <i>Oxy</i> ta gọi  là đường thẳng có phương trình:<i>x</i>2<i>y</i> 4 0và các
điểm <i>M x y</i>

 

; ,<i>A</i>

 

3;6 ,<i>B</i>

 

5;8 thì <i>P</i><i>MA</i><i>MB</i>.


Bài tốn trở thành tìm toạ độ điểm <i>M</i> thuộc  sao cho tổng <i>MA</i><i>MB</i> đạt giá trị nhỏ nhất.
Rõ ràng ,<i>A B</i> nằm về cùng một phía với .


Ta tìm được điểm <i>A</i>

 

5; 2 ,đối xứng <i>A</i> qua  .


Với <i>M</i> thuộc  ta có:<i>MA</i><i>MB</i><i>MA</i><i>MB</i><i>A B</i> (không đổi)


Đẳng thức xảy ra khi và chỉ khi ,<i>A M B</i> , thẳng hàng hay <i>M</i> chính là giao điểm của  với đường
thẳng <i>A B</i>


Tìm được PT đường thẳng <i>A B</i> là <i>x</i>– 5 0 .


Giải hệ PT: 5 0
2 4 0
<i>x</i>


<i>x</i> <i>y</i>



 


   


5
9
2
<i>x</i>
<i>y</i>





  <sub></sub>



Kết kuận:<i>MinP</i>6


5
9
2
<i>x</i>
<i>y</i>





 



</div>
<span class='text_page_counter'>(16)</span><div class='page_container' data-page=16>

16 DAYHOCTOAN.VN
<b>Bài 33. </b> <b>Câu 10. </b> Cho ba số thực a,b,c thỏa mãn: 1<sub>2</sub> 1<sub>2</sub> 1<sub>2</sub> 1


<i>a</i> <i>b</i> <i>c</i>  Tìm GTLN của biểu thức:


2 2 2 2 2 2


1 1 1


5 2 2 5 2 2 5 2 2


<i>T</i>


<i>a</i> <i>ab</i> <i>b</i> <i>b</i> <i>bc</i> <i>c</i> <i>c</i> <i>ac</i> <i>a</i>


  


     


<b>Bài giải: </b>


Ta có:


2 2 2 2 2


1 1


5<i>a</i> 2<i>ab</i> 2<i>b</i> 3<i>a</i> 2<i>a</i> 2<i>ab</i> 2<i>b</i>




     2


1
3<i>a</i> 6<i>ab</i>




2
1


3<i>a</i> 6<i>ab</i> 2 3 4 2 3 4 2


1 1 1


3<i>a</i> 3<i>ab</i> 3<i>ab</i> <sub>9</sub> <i><sub>a b</sub></i> <sub>3</sub> <i><sub>a b</sub></i>


  


 


3 4 2 3 2 3 2 3 2


1 1


3 <i>a b</i> 3 <i>a</i> <i>a</i> <i>b</i>



3 2 3 2 3 2


1 1 1 1


. .


3 <i><sub>a</sub></i> <i><sub>a</sub></i> <i><sub>b</sub></i>


 1 1. 1<sub>2</sub> 1<sub>2</sub> 1<sub>2</sub>


3 3 <i>a</i> <i>a</i> <i>b</i>


 


 <sub></sub>   <sub></sub>


 


2 2 2


3 4 2


1 1 1 1 1


3 3


3 <i>a b</i> <i>a</i> <i>a</i> <i>b</i>


   



2 2 2


2 2 2 2 2 2


1 1 1


1


1 1 1 1 1 1 1 1 1 1


2


3 3 3 3 6 3 6 3


<i>a</i> <i>a</i> <i>b</i>


<i>a</i> <i>a</i> <i>b</i> <i>a</i> <i>a</i> <i>b</i>


 <sub></sub> <sub></sub> <sub></sub>


  <sub></sub> <sub></sub>


 


    <sub></sub>   <sub></sub>


 


Suy ra: <sub>2</sub> <sub>2</sub> <sub>2</sub>



2 2


1 1 1 1 1 1


6 3 6 3


5<i>a</i> 2<i>ab</i> 2<i>b</i> <i>a</i> <i>a</i> <i>b</i>


 


 <sub></sub>   <sub></sub>


 


 


Tương tự ta có: <sub>2</sub> <sub>2</sub> <sub>2</sub>


2 2


1 1 1 1 1 1


6 3 6 3


5<i>b</i> 2<i>bc</i> 2<i>c</i> <i>b</i> <i>b</i> <i>c</i>


 


 <sub></sub>   <sub></sub>



 


 


<sub>2</sub> <sub>2</sub> <sub>2</sub>


2 2


1 1 1 1 1 1


6 3 6 3


5<i>c</i> 2<i>ac</i> 2<i>a</i> <i>c</i> <i>c</i> <i>a</i>


 


 <sub></sub>   <sub></sub>


 


 


 3 1<sub>2</sub> 1<sub>2</sub> 1<sub>2</sub> 3 1 1 1


6 3 6 3 2 3 2 3 3


<i>T</i>


<i>a</i> <i>b</i> <i>c</i>



 


 <sub></sub>   <sub></sub>   


 


Vậy 1


3


<i>T</i>


<i>Max</i>  khi và chỉ khi <i>a</i>  <i>b</i> <i>c</i> 3.


<b>Bài 34. </b> <b>Câu 11. </b> Cho <i>x y z</i>, , là các số thực dương thỏa mãn <i>x</i>  <i>y</i> <i>z</i> <i>xyz</i>.Chứng minh rằng:
2


2 2


1 1


1 1 <i>x</i> <i>y</i> 1 1 <i>z</i>


<i>xyz</i>


<i>x</i> <i>y</i> <i>z</i>


 


   



   (I)


<b>Bài giải: </b>


Giả thiết suy ra: 1 1 1 1
<i>xy</i> <i>yz</i><i>zx</i>  .
Ta có:


2


2


1 <i>x</i> 1 1 1 1 1 1 1 1


<i>x</i> <i>x</i> <i>xy</i> <i>yz</i> <i>zx</i> <i>x</i> <i>y</i> <i>x</i> <i>z</i>


 


 <sub></sub> <sub></sub> <sub></sub> <sub></sub> <sub></sub> <sub></sub>  <sub></sub> 


  


 


 


1 2 1 1
2 <i>x</i> <i>y</i> <i>z</i>



 


 <sub></sub>   <sub></sub>


 


Dấu ""xảy ra  <i>y</i> <i>z</i>


</div>
<span class='text_page_counter'>(17)</span><div class='page_container' data-page=17>

17 DAYHOCTOAN.VN
2


2 2


1 1


1 1 <i>x</i> <i>y</i> 1 1 <i>z</i>


<i>x</i> <i>y</i> <i>z</i>


 


   


   3 1 1 1


<i>x</i> <i>y</i> <i>z</i>


 


 



 


 


Dấu ""xảy ra   <i>x</i> <i>y</i> <i>z</i>
Ta chứng minh:3 1 1 1 <i>xyz</i>


<i>x</i> <i>y</i> <i>z</i>


 


  


 


 

   



2 2


3 <i>xy</i> <i>yz</i> <i>zx</i> <i>xyz</i> <i>x</i> <i>y</i> <i>z</i>


      


 

2

 

2

2


0


<i>x</i> <i>y</i> <i>y</i> <i>z</i> <i>z</i> <i>x</i>



       (Điều này lng đúng).
Dấu bằng có khi và chỉ khi <i>x</i> <i>y</i> <i>z</i>.


Vậy

 

<i>I</i> được chứng minh,dấu bằng có khi và chỉ khi <i>x</i>  <i>y</i> <i>z</i> 3.


<b>Bài 35. </b> <b>Câu 12. </b> Cho <i>x y z</i>, , là 3 số dương thỏa mãn điều kiện: <i>x</i>2<i>y</i>2<i>z</i>2 2011. Tìm giá trị nhỏ nhất
của <i>P</i> <i>xy</i> <i>yz</i> <i>zx</i>


<i>z</i> <i>x</i> <i>y</i>


  


<b>Bài giải: </b>




2 2 2 2 2 2


2 2 2 2


2 2 2 2


<i>x y</i> <i>x z</i> <i>z y</i>


<i>P</i> <i>x</i> <i>y</i> <i>z</i>


<i>z</i> <i>y</i> <i>x</i>


      =



2 2 2 2 2 2


2 2 2 2.2011


<i>x y</i> <i>x z</i> <i>z y</i>


<i>z</i>  <i>y</i>  <i>x</i> 


Ta có,theo BĐT Cosi:


2 2 2 2 2 2


2 2 2


2 2 2


<i>x y</i> <i>x z</i> <i>z y</i>


<i>x</i> <i>y</i> <i>z</i>


<i>z</i>  <i>y</i>  <i>x</i>    .


Nên <i>P</i>2 3.2011 <i>P</i> 3.2011.


Vậy GTLN của P bằng 3.2011 đạt đợc khi 2011
3
<i>x</i>  <i>y</i> <i>z</i>


<b>Bài 36. </b> <b>[0D4-4]</b> Cho <i>a,b,c</i> là các số dương thỏa mãn:<i>ab bc ca</i>  3<i>. </i>Chứng minh rằng :





<i>2</i> <i>2</i> <i>2</i>


<i>1</i> <i>1</i> <i>1</i> <i>1</i>


<i>+</i> <i>+</i>


<i>1+ a</i> <i>b+ c</i> <i>1+b</i> <i>c + a</i> <i>1+ c</i> <i>a +b</i> <i>abc</i>.
<b>Lời giải </b>


Từ giả thiết <i><sub>3 = ab+bc+ca</sub></i><i><sub>3 a b c</sub>3</i> <i>2</i> <i>2 2</i>  <i>3</i> <i><sub>a b c</sub>2</i> <i>2</i> <i>2</i> <i><sub>1</sub></i>


<i>abc 1</i>


  .


Nên ta có :




2 2


1 1 1 1


1<i>a</i> <i>b c</i> <i>abc</i><i>a</i> <i>b c</i>  <i>a ab bc ca</i>  3<i>a</i>.
Tương tự:





<i>2</i>


<i>1</i> <i>1</i>


<i>1+b</i> <i>c + a</i> <i>3b</i>; <i>2</i>



<i>1</i> <i>1</i>


<i>1+ c</i> <i>a +b</i> <i>3c</i>.
Cộng vế với vế các BĐT trên ta được:


2 2 2


1 1 1 1 1 1 1


3 3 3 3


1 ( ) 1 ( ) 1 ( )


<i>ab bc ca</i>


<i>a</i> <i>b</i> <i>c</i> <i>abc</i> <i>abc</i>


<i>a b c</i> <i>b a c</i> <i>c b a</i>


 


      


     



Dấu bằng xảy ra    a b c 1.<b> </b>


<b>Bài 37. </b> <b>[0D4-4]</b> G Cho ba số thực không âm <i>a b c</i>, , thỏa mãn điều kiện: <i>a b c</i>  1. Tìm giá trị lớn nhất,
giá trị nhỏ nhất của biểu thức <i>F</i><i>ab bc ca</i>  2<i>abc</i>.


</div>
<span class='text_page_counter'>(18)</span><div class='page_container' data-page=18>

18 DAYHOCTOAN.VN
Điều kiện: <i>x</i> 2.


Từ giả thiết, chỉ ra được a; b;c

 

0;1  0 abc1.


Ta có <sub>ab bc ca</sub>  <sub>3 (abc)</sub>3 2 <sub>3 (abc)</sub>3 3 <sub>3abc</sub><sub>2abc</sub><sub>, suy ra </sub><i><sub>F</sub></i><sub>0</sub><sub>. </sub>
Dấu bằng có xảy ra, chẳng hạn tại <i>a</i> <i>b</i> 0; <i>c</i>1.


Vậy giá trị nhỏ nhất của <i>F</i>là 0 đạt được khi <i>a</i> <i>b</i> 0; <i>c</i>1 và các hoán vị.
Khơng mất tính tổng qt giải sử a b c a 0; 1


3


 


    <sub></sub> <sub></sub>.


Ta có: ab bc ca 2abc a b c

bc 1 2a

 

a 1 a

1

b c

 

2 1 2a


4


            .



1

 

2




a 1 a 1 a 1 2a


4


     .

2

2


1 1


1 a 2a a 1 1 2a a 1


4 4 


     <sub></sub>   <sub></sub>.


3


1 1 2a a a 7


1


4 3 27


<sub></sub> <sub></sub> <sub> </sub> <sub></sub> 
 <sub></sub><sub></sub> <sub></sub>  <sub></sub>


 


 



  .


Vậy giá trị lớn nhất của <i>P</i> là 7


27 đạt được khi:


1


a b c


3


   .


<b>Bài 38. </b> <b> [0D4-4]</b> Cho <i>a</i>,<i>b</i>,<i>c</i><sub> là các số thực khác </sub>0.Chứn minh rằng: <i>a</i> <i>b</i> <i>b</i> <i>c</i> <i>c</i> <i>a</i>


<i>c</i> <i>a</i> <i>b</i>


<b>2</b> <b>2</b> <b>2</b> <b>2</b> <b>2</b> <b>2</b>


<b>2</b> <b>2</b> <b>2</b> <b>6</b>. Dấu


đẳng thức xảy ra khi nào?


<b>Lời giải </b>
2


2 2 1 0



<i>t</i>  <i>t</i> <i>mx m</i>  

 

2 . Ta có:


( ) ( ) ( )


<i>a</i> <i>b</i> <i>b</i> <i>c</i> <i>c</i> <i>a</i> <i>a</i> <i>b</i> <i>a</i> <i>c</i> <i>b</i> <i>c</i>


<i>c</i> <i>a</i> <i>b</i> <i>b</i> <i>a</i> <i>c</i> <i>a</i> <i>c</i> <i>b</i>


<b>2</b> <b>2</b> <b>2</b> <b>2</b> <b>2</b> <b>2</b> <b>2</b> <b>2</b> <b>2</b> <b>2</b> <b>2</b> <b>2</b>


<b>2</b> <b>2</b> <b>2</b> <b>2</b> <b>2</b> <b>2</b> <b>2</b> <b>2</b> <b>2</b> .


Do <i>a</i>,<i>b</i>,<i>c</i> khác0 nên <i>a</i><b>2</b>, <i>b</i>2,<i>c</i>2 là các số dương.Á dụng bất đẳng thức cauchy ta có: <i>a</i> <i>b</i>


<i>b</i> <i>a</i>


<b>2</b> <b>2</b>
<b>2</b> <b>2</b>
<i>a b</i>


<i>b a</i>
<b>2</b> <b>2</b>
<b>2</b> <b>2</b>


<b>2</b> <b>2</b>, <i>a</i> <i>c</i>


<i>c</i> <i>a</i>


<b>2</b> <b>2</b>
<b>2</b> <b>2</b>



<i>a c</i>
<i>c a</i>
<b>2</b> <b>2</b>
<b>2</b> <b>2</b>


<b>2</b> <b>2</b>, <i>b</i> <i>c</i>


<i>c</i> <i>b</i>


<b>2</b> <b>2</b>
<b>2</b> <b>2</b>


<i>b c</i>
<i>c b</i>
<b>2</b> <b>2</b>
<b>2</b> <b>2</b>


<b>2</b> <b>2</b>.


Vậy <i>a</i> <i>b</i> <i>b</i> <i>c</i> <i>c</i> <i>a</i>


<i>c</i> <i>a</i> <i>b</i>


<b>2</b> <b>2</b> <b>2</b> <b>2</b> <b>2</b> <b>2</b>


<b>2</b> <b>2</b> <b>2</b> <b>6</b>.


Dấu “=” xảy ra khi và chỉ khi <i>a</i><b>2</b> <i>b</i><b>2</b> <i>c</i><b>2</b>
.



<b>Bài 39. </b> <b>[2D1-4]</b> Chứng minh:

<i>x</i>2<i>ax b</i>

 

2 <i>x</i>2<i>cx</i><i>d</i>

 

2  2<i>x</i>21

2 với mọi <i>a</i>, b, <i>c</i>, <i>d</i> thỏa mãn điều
kiện: 2 2 2 2


1
<i>a</i> <i>b</i>  <i>c</i> <i>d</i>  .


<b>Lời giải </b>


Áp dụng BĐT Bunhiacopski cho 2 cặp 3 số:

<i>x x</i>; ;1

<i>x a b</i>; ;

ta có:


2 2 2 2 2 2 2 2 2 2 2


) ( 1)( ) (2 1)( )


</div>
<span class='text_page_counter'>(19)</span><div class='page_container' data-page=19>

19 DAYHOCTOAN.VN


Áp dụng BĐT Bunhiacopski cho 2 cặp 3 số:

<i>x x</i>; ;1

<i>x c d</i>; ;

ta có:


)
d
c
x
)(
1
x
2
(
)
d


c
x
)(
1
x
x
(
)
d
cx
x


( 2  2  2 2 2 2 2  2 2 2 2

 

2
Từ

 

1 và

 

2 suy ra:


2 2 2 2 2 2 2 2 2 2 2 2


(<i>x</i> <i>ax b</i> ) (<i>x</i> <i>cx</i><i>d</i>) (2<i>x</i> 1)(<i>x</i> <i>a</i> <i>b</i> ) (2 <i>x</i> 1)(<i>x</i> <i>c</i> <i>d</i> )


)
d
c
b
a
x
2
)(
1
x
2



( 2 2 2 2 2 2


 .
2
2
2
2
)
1
x
2
(
)
1
x
2
)(
1
x
2
(    


 ( Vì <i>a</i>2<i>b</i>2 <i>c</i>2 <i>d</i>2 1).
Vậy:

2

 

2 2

 

2 2

2


2 1


<i>x</i> <i>ax b</i>  <i>x</i> <i>cx</i><i>d</i>  <i>x</i>  .
Đẳng thức xảy ra



2 1 1


<i>x</i> <i>a</i> <i>b</i> <i>c</i> <i>d</i>


<i>x</i> <i>x</i>


       <i>a</i> <i>c</i> <i>bx</i><i>dx</i>.


<b>Bài 40. </b> <b> [0D4-4]</b> Cho a<sub>1</sub>,a<sub>2</sub>,..a<sub>10</sub> là các số thực dương. Tìm giá trị nhỏ nhất của biểu thức:




2 2 2


1 2 10


10 1 2 9


...
...


<i>a</i> <i>a</i> <i>a</i>


<i>P</i>


<i>a</i> <i>a</i> <i>a</i> <i>a</i>


  





   .


<b>Lời giải </b>


Ta có: 2 2 2 2 2 2 2 2 2


1 2 10 1 10 2 10 9 10


1 1 1


... ...


9 9 9


<i>a</i> <i>a</i>  <i>a</i> <i>a</i>  <i>a</i> <i>a</i>  <i>a</i>  <i>a</i>  <i>a</i> .
Áp dụng BĐT cauchy ta có: 2 <sub>1</sub> <sub>10</sub>


10
2


1 a a


3
2
a
9
1



a   .


2<sub>2</sub> <sub>10</sub>2 a<sub>2</sub>a<sub>10</sub>


3
2
a
9
1


a   .


...


2 2


9 10 9 10


1 2


9 3


<i>a</i>  <i>a</i>  <i>a a</i> .


 <sub>1</sub>2 <sub>2</sub>2 <sub>10</sub>2 a<sub>10</sub>

a<sub>1</sub> a<sub>2</sub> ... a<sub>9</sub>


3


2
a
...


a


a        .


<sub></sub>

<sub></sub>


3
2
)
a
...
a
a
(
a
)
a
...
a
a
(
a
.
3
2
a
...
a
a
a
a

...
a
a
P
9
2
1
10
9
2
1
10
9
2
1
10
2
10
2
2
2
1 














 .


<b>Bài 41. </b> <b>[0D4-4]</b> Chứng minh rằng: 1 1 2; , 1


<i>n</i> <i>n</i>


<i>n</i> <i>n</i> <i>n</i> <i>n</i> <i><sub>n</sub></i> <i><sub>n</sub></i>


<i>n</i> <i>n</i>


      .


<b>Lời giải </b>
Từ điều kiện <i>n</i> , <i>n</i>1 ta có: <i>n</i>


<i>n</i> <i>n</i> 1


<i>n</i>


<i>n</i>
<i>n</i>


  1 0


<i>n</i>



<i>n</i>
<i>n</i>
   .


Áp dụng BĐT cauchy cho <i>n</i> số dương: 1 ;1;1;...;1


<i>n</i>


<i>n</i>
<i>n</i>


 ta có:




1 1 .1.1...1


<i>n</i> <i>n</i>


<i>n</i> <i>n</i> <i><sub>n</sub></i> <i>n</i>


<i>n</i> <i>n</i>


 


  <sub></sub><sub></sub>  <sub></sub><sub></sub>


</div>
<span class='text_page_counter'>(20)</span><div class='page_container' data-page=20>

20 DAYHOCTOAN.VN
1



1 1 ... 1


<i>n</i>


<i>n</i>


<i>n</i> <i>n</i>


  


 <sub></sub><sub></sub>  <sub></sub>   <sub></sub>


 


 


1 <i>n</i>


<i>n</i>
<i>n</i>


<i>n</i> <i>n</i>


 


 <sub></sub>  <sub></sub>


  1 2



<i>n</i>


<i>n</i>
<i>n</i>


 

 

1 .
Áp dụng BĐT cauchy cho <i>n</i> số dương: 1 ;1;1;...;1


<i>n</i>


<i>n</i>
<i>n</i>


 ta có:


1 1 .1.1...1


<i>n</i> <i>n</i>


<i>n</i> <i>n</i> <i><sub>n</sub></i> <i>n</i>


<i>n</i> <i>n</i>


 


  <sub></sub>  <sub></sub>


 


1



1 1 ... 1


<i>n</i>


<i>n</i>


<i>n</i> <i>n</i>


  


 <sub></sub><sub></sub>  <sub></sub>   <sub></sub>


 


 


1 <i>n</i>


<i>n</i>
<i>n</i>


<i>n</i> <i>n</i>


 


 <sub></sub>  <sub></sub>


  1 2



<i>n</i>


<i>n</i>
<i>n</i>


 

 

2 .


Từ

 

1 và

 

2 suy ra: 2


n
n
1
n


n
1
n


n
1
n


n
1


n
2


n
n



n
n


n










 .


Đẳng thức khơng thể xảy ra vì <i>n</i> , <i>n</i>1 ta có:


n
n
1
1
n


n
1


n
n







 .


Vậy: 2


n
n
1
n


n


1 n


n
n


n






 .


<b>Bài 42. </b> <b>[0D4-4]</b> Cho <i>a<sub>i</sub></i>,<i>b<sub>i</sub></i> ,

<i>i</i>1, 2,3

.



a)Chứng minh rằng:



2


3
3
2
2
1
1
2
3
2
2
2
1
2
3
2
2
2


1 a a b b b a b a b a b


a       

 

1 .


b) Giả sử <i>a a</i><sub>1 2</sub><i>a a</i><sub>2</sub> <sub>3</sub><i>a a</i><sub>3 1</sub>4. Tìm giá trị nhỏ nhất của biểu thức: 4 4 4


1 2 3


<i>P</i><i>a</i> <i>a</i> <i>a</i> .
<b>Lời giải </b>



a) TH1: Nếu <i>a</i>1<i>a</i>2 <i>a</i>30 hoặc <i>b</i>1 <i>b</i>2 <i>b</i>3 0 thì BĐT đúng.
TH2: Nếu 2 2 2


1 2 3 0


<i>a</i> <i>a</i> <i>a</i>  .


Xét hàm số:

 

2 2 2

2

2 2 2



1 2 3 2 1 1 2 2 3 3 1 2 3


<i>f x</i>  <i>a</i> <i>a</i> <i>a</i> <i>x</i>  <i>a b</i> <i>a b</i> <i>a b x</i> <i>b</i> <i>b</i> <i>b</i> .

<i>a x b</i><sub>1</sub>  <sub>1</sub>

 

2 <i>a x b</i><sub>2</sub>  <sub>2</sub>

 

2 <i>a x b</i><sub>3</sub>  <sub>3</sub>

20 với mọi <i>x</i> .
Theo định lý về dấu của tam thức bậc hai thì:


2 2 2


1 2 3 0


0


<i>a</i> <i>a</i> <i>a</i>


   


 <sub></sub>

mà 2 2 2



1 2 3 0


<i>a</i> <i>a</i> <i>a</i>  luôn đúng    0

<i>a b</i><sub>1 1</sub><i>a b</i><sub>2 2</sub><i>a b</i><sub>3 3</sub>

2 

<i>a</i><sub>1</sub>2<i>a</i><sub>2</sub>2<i>a</i><sub>3</sub>2



<i>b</i><sub>1</sub>2<i>b</i><sub>2</sub>2<i>b</i><sub>3</sub>2

.
Đẳng thức xảy ra khi 1 2 3


1 2 3


<i>a</i>


<i>a</i> <i>a</i>


<i>b</i>  <i>b</i> <i>b</i> .


b) Áp dụng BĐT

 

1 ta có:

 

 

 


2


2 2 2


4 4 4 4 4 4


1 2 3 . 2 3 1 1 2 2 3 3 1


<i>a</i> <i>a</i> <i>a</i> <i>a</i> <i>a</i> <i>a</i> <sub></sub> <i>a a</i>  <i>a a</i>  <i>a a</i> <sub></sub>

 

2 .

 

 



2 2 2

<sub>2</sub> <sub>2</sub> <sub>2</sub>

2 <sub>2</sub>


1 2 2 3 3 1 . 1 1 1 1 2 2 3 3 1 4


<i>a a</i>  <i>a a</i>  <i>a a</i>    <i>a a</i> <i>a a</i> <i>a a</i>  .


 

 



2 2 2



1 2 2 3 3 1


16
.


3


<i>a a</i> <i>a a</i> <i>a a</i>


   

 

3 .


Thay

 

3 vào

 

2 ta được:



2
2


4 4 4


1 2 3


16
3


<i>a</i> <i>a</i> <i>a</i>    
  .



 16


3


</div>
<span class='text_page_counter'>(21)</span><div class='page_container' data-page=21>

21 DAYHOCTOAN.VN
Vậy giá trị nhỏ nhất của <i>P</i> là: 16


3


<i>P</i> khi <sub>1</sub> <sub>2</sub> <sub>3</sub> 2


3


<i>a</i> <i>a</i> <i>a</i>  .


<b>Bài 43. </b> <b>[0D4-4]</b> Cho <i>x y z</i>, , không âm thỏa mãn: <i>xy</i><i>yz</i><i>zx</i><i>xyz</i>4. Chứng minh rằng:
<i>x</i>  <i>y</i> <i>z</i> <i>xy</i><i>yz</i><i>zx</i>.


<b>Lời giải </b>


Ta có: xyyzzxxyz4 xyyzzx4xyz

 

1 .
Do đó: xyzxyyzzx xyz4xyz (2).


Từ điều kiện đề bày thì , ,<i>x y z</i> khơng đồng thời bằng khơng, hơn nữa chỉ có tối đa một số bằng
khơng. Khơng mất tính tổng qt giả sử rằng: ,<i>x y</i>0.


Từ

 

1 ta có:


xy
y


x


xy
4
z







 .


Khi đó:

 

2 <i>x</i> <i>y</i> 4 <i>xy</i> 4 <i>xy</i> 4 <i>xy</i>


<i>x</i> <i>y</i> <i>xy</i> <i>x</i> <i>y</i> <i>xy</i>


 


    


    .


<i>x</i><i>y</i>

2<i>xy x</i>

<i>y</i>

 4 <i>xy</i>4

<i>x</i><i>y</i>

4<i>xy</i>4<i>xy</i><i>x y</i>2 2.

<i>x</i> <i>y</i> 2

2<i>xy</i>

1<i>x</i>



1<i>y</i>

(3).


i) Nếu

1x



1y

0 thì

 

3 hiển nhiên đúng.


Đẳng thức xảy ra 

xy2

2 xy(1x)(1y)0xy1.
Kết hợp với

 

2 ta có: <i>x</i>  <i>y</i> <i>z</i> 1.


ii) Nếu (1x)(1y)0. Ta có:


 

2

2


2 1 1


<i>x</i> <i>y</i>    <i>x</i> <i>y</i>  (1 <i>x</i>)2 (1 <i>y</i>)22(1<i>x</i>)(1<i>y</i>)4(1<i>x</i>)(1<i>y</i>)<sub>. </sub>


xy2

2 4(1x)(1y)


 

4 .


Nhưng do: 0


xy
y
x


xy
4


z 







 và ,<i>x y</i>  0 4 <i>xy</i>0  4 <i>xy</i>

 

5 .

Từ

 

4 và

 

5 suy ra

 

3 được chứng minh.


<b>Bài 44. </b> <b>Bài 27.</b> Cho các số thực <i>a b c d</i>, , , thoả mãn 4<i>a</i>2<i>b</i>2 2và<i>c d</i> 4 . Tìm giá trị lớn nhất của
biểu thức <i>P</i>2<i>ac bd cd</i>  .


<b>Bài giải: </b>


Ta có

 



2
2


2 4 1


4


<i>c</i>


<i>ac</i> <i>a</i> 

 



2
2


2
4


<i>d</i>


<i>bd</i> <i>b</i> 

 




2


3


8 2


<i>c d</i> <i>cd</i>


<i>cd</i>   


Cộng vế

     

1 , 2 , 3 ta có


2

<sub></sub>

<sub></sub>

2


2 2


2 2 2 2 3


2 4 4 8


4 4 2 8 8


<i>c d</i> <i>c d</i>


<i>c</i> <i>d</i> <i>cd</i>


<i>P</i> <i>ac bd</i> <i>cd</i> <i>a</i>  <i>b</i>      <i>a</i> <i>b</i>   


1



8 ; ; ; ;1; 2; 2
2


<i>P</i>  <i>a b c d</i>   <sub></sub>


 . Vậy giá trị lớp nhất của <i>P</i>bằng 8.


<b>Bài 45. </b> <b>Bài 31. </b> Cho <i>a b c</i>, , là ba cạnh của một tam giác, chứng minh rằng


2 2 2


2(

).



<i>a</i>

<i>b</i>

<i>c</i>

<i>ab</i>

<i>bc</i>

<i>ca</i>



</div>
<span class='text_page_counter'>(22)</span><div class='page_container' data-page=22>

22 DAYHOCTOAN.VN
Trước hết ta chứng minh 2 2


(<i>b c</i> ) <i>a</i>


Tương tự: 2 2 2 2


(<i>a c</i> ) <i>b</i> , (<i>b a</i> ) <i>c</i>
Từ đó suy ra:


2 2 2 2 2 2


2 2 2


(

)

(

)

(

)




2(

) (

).



<i>b</i>

<i>c</i>

<i>a</i>

<i>c</i>

<i>b</i>

<i>a</i>

<i>a</i>

<i>b</i>

<i>c</i>



<i>a</i>

<i>b</i>

<i>c</i>

<i>ab</i>

<i>bc</i>

<i>ca</i>

<i>dpcm</i>







<b>Bài 46. </b> <b>Bài 33</b>. Cho , ,<i>x y z</i>là 3 số dương thỏa mãn điều kiện

<i>x</i>

2

<i>y</i>

2

<i>z</i>

2

2012.

Tìm giá trị
nhỏ nhất của

<i>P</i>

<i>xy</i>

<i>yz</i>

<i>zx</i>



<i>z</i>

<i>x</i>

<i>y</i>



.


<b>Bài giải: </b>




2 2 2 2 2 2


2 2 2 2


2 2 2

2



<i>x y</i>

<i>x z</i>

<i>z y</i>




<i>P</i>

<i>x</i>

<i>y</i>

<i>z</i>



<i>z</i>

<i>y</i>

<i>x</i>







2 2 2 2 2 2


2 2 2

2.2012



<i>x y</i>

<i>x z</i>

<i>z y</i>



<i>z</i>

<i>y</i>

<i>x</i>





Ta có, theo BĐT Cơsi:


2 2 2 2 2 2


2 2 2


2 2 2


<i>x y</i>

<i>x z</i>

<i>z y</i>



<i>x</i>

<i>y</i>

<i>z</i>




<i>z</i>

<i>y</i>

<i>x</i>



2


3.2012

3.2012



<i>P</i>

<i>P</i>



 



3.2012


<i>MinP</i>



khi

2012



3


<i>x</i>

  

<i>y</i>

<i>z</i>



<b>Bài 47. </b> <b>Bài 34. </b> Cho <i>x y z</i>, , là 3 số thực thỏa mãn :<i>x</i><b>2</b> <i>xy</i> <b>4</b><i>y</i><b>2</b> <b>3</b><i>yz</i> <i>z</i><b>2</b> <b>3 2</b>. Chứng minh


rằng <i>x</i> <b>5</b><i>y</i> <i>z</i> <b>13</b> <b>2</b>


<b>2</b> <b>2</b> . Dấu " " xảy ra khi nào?


<b>Bài giải: </b>


Ta có: <i>x</i><b>2</b> <i>xy</i> <b>4</b><i>y</i><b>2</b> <b>3</b><i>yz</i> <i>z</i><b>2</b> (<i>x</i> <b>1</b><i>y</i>)<b>2</b> (<b>3</b><i>y</i> <i>z</i>)<b>2</b> <b>3</b><i>y</i><b>2</b> <b>3 2</b>


<b>2</b> <b>2</b> <b>2</b> .



Áp dụng BĐT Bunhiacốpski ta có:


(<i>x</i> <i>y</i> <i>z</i>) (<i>x</i> <i>y</i>) ( <i>y</i> <i>z</i>) <i>y</i>
<b>2</b>
<b>2</b>


<b>5</b> <b>1</b> <b>3</b> <b>1</b>


<b>2</b> <b>2</b> <b>2</b> <b>2</b> (<i>x</i> <i>y</i>) ( <i>y</i> <i>z</i>) ( <i>y</i>)


<b>2</b>


<b>1</b> <b>3</b> <b>1</b> <b>3</b>


<b>2</b> <b>2</b> <b>6</b> <b>2</b>


( <b>2</b> <b>2</b> ( <b>1</b> ) ) (<b>2</b> <i>x</i> <b>1</b><i>y</i>)<b>2</b> (<b>3</b><i>y</i> <i>z</i>)<b>2</b> <b>3</b><i>y</i><b>2</b>


<b>1</b> <b>1</b>


<b>2</b> <b>2</b> <b>2</b>


<b>6</b>


(<i>x</i> <b>5</b><i>y</i> <i>z</i>)<b>2</b> <b>13</b> <b>2</b>


<b>2</b> <b>2</b> <i>x</i> <i>y</i> <i>z</i>


<b>5</b> <b>13</b>



<b>2</b>


</div>
<span class='text_page_counter'>(23)</span><div class='page_container' data-page=23>

23 DAYHOCTOAN.VN
Dấu " " xảy ra khi


<i>x</i> <i>xy</i> <i>y</i> <i>yz</i> <i>z</i>


<i>y</i>


<i>x</i> <i>y</i> <i>y</i> <i>z</i>


<b>2</b> <b>2</b> <b>2</b>


<b>4</b> <b>3</b> <b>3 2</b>


<b>3</b>
<b>1</b> <b>3</b>
<b>2</b>
<b>2</b> <b>2</b>
<b>1</b>
<b>1</b> <b>1</b>
<b>6</b>

<i>x</i>
<i>y</i>
<i>z</i>


<b>5</b> <b>2 2</b>
<b>2</b> <b>13</b>



<b>2 2</b>
<b>13</b>
<b>3</b> <b>2 2</b>
<b>2</b> <b>13</b>


.


<b>Bài 48. </b> <b>Bài 38. </b> <b> </b>Cho , ,<i>x y z</i>0, <i>x</i>  <i>y</i> <i>z</i> 1 . Tìm <i>MaxP</i> ,


2 2 2 2 2 2


<i>xy</i> <i>yz</i> <i>zx</i>


<i>P</i>


<i>x</i> <i>y</i> <i>y</i> <i>z</i> <i>z</i> <i>x</i>


  


     


<b>Bài giải: </b>


1 1


2


2 2 9 1 1 2 9 2


<i>cyc</i> <i>cyc</i> <i>cyc</i>



<i>xy</i> <i>xy</i> <i>xy</i> <i>xy</i> <i>xy</i>


<i>P</i> <i>xy</i>


<i>x</i> <i>y</i> <i>x</i> <i>y</i> <i>x</i> <i>y</i>


   


  <sub></sub>   <sub></sub> <sub></sub>  <sub></sub>


  <sub></sub>  <sub></sub> <sub></sub>  <sub></sub>




2

1


3 ( ) 1


3


<i>xy</i><i>yz</i><i>zx</i>  <i>x</i> <i>y</i> <i>z</i>   <i>xy</i><i>yz</i><i>zx</i> 

2



2

9 2


2 9


<i>xy</i> <i>x</i> <i>y</i>


<i>x</i> <i>y</i> <i>x</i> <i>y</i> <i>xy</i>


<i>x</i> <i>y</i>





    




Nên 1 2 (2 2 2 ) 1


9 3 9 9


<i>x</i> <i>y</i> <i>y</i> <i>z</i> <i>z</i> <i>x</i>


<i>P</i> <sub></sub>       <sub></sub>


 


<b>Bài 49. </b> <b>Bài 39. </b> <b> </b>Chứng minh:


3 4 2 2 3 4 4 2 3 9


<i>xy</i> <i>yz</i> <i>xz</i> <i>x</i> <i>y</i> <i>z</i>


<i>x</i> <i>y</i> <i>z</i> <i>x</i> <i>y</i> <i>z</i> <i>x</i> <i>y</i> <i>z</i>


 


  


     



<b>Bài giải: </b>


Áp dụng BĐT :


9
1
1
1
1
9
1
1
1
)


( <i>A</i> <i>B</i> <i>C</i>


</div>
<span class='text_page_counter'>(24)</span><div class='page_container' data-page=24>

24 DAYHOCTOAN.VN


Tương tự 9( )(3)


2
81


2
3
2


4 <i>x</i> <i>y</i> <i>z</i>



<i>xz</i>
<i>x</i>


<i>z</i>
<i>z</i>
<i>y</i>
<i>x</i>


<i>xz</i>










Từ (1);(2);(3) ta có


9
27


)
(


2
27
)



(
9


)
(


2
27


<i>z</i>
<i>y</i>
<i>x</i>
<i>z</i>
<i>y</i>
<i>x</i>
<i>z</i>
<i>y</i>
<i>x</i>
<i>z</i>
<i>y</i>
<i>x</i>


<i>zx</i>
<i>yz</i>
<i>xy</i>
<i>z</i>


<i>y</i>
<i>x</i>



<i>Q</i>         













2


3
1


<i>z</i>
<i>y</i>
<i>x</i>
<i>zx</i>
<i>yz</i>


<i>xy</i>    


<b>Cách 2: </b>

xy

yz

zx

x

y

z

.



3x

4y

2z

3y

4z

2x

3z

4x

2y

9




 







Áp dụng


2 2 2 2


(a b c) a b c


A B C A B C


  <sub></sub> <sub></sub> <sub></sub>


  với , ,<i>a b c</i> không âm và <i>A B</i>, dương. Dấu " " xảy ra khi
a b c


A B C ta có:


2


6

2 1



xy

xy

xy

xy

18

2

1



.




3x

4y

2z

2(x

y

z)

2y

x

81 2(x

y

z)

2y

x

81 x

y

z

y

x



 



<sub></sub>

 

<sub></sub>



  

  

<sub></sub>

 

<sub></sub>







<sub></sub>

<sub></sub>



<sub></sub>

 

<sub></sub>



xy

1

18xy



2x

y (1)



3x

4y

2z

81 x

y

z

Dấu " " xảy ra khi   x y z


Tương tự ta có:

yz

1

18yz

2y

z



3y

4z

2x

81 x

y

z





<sub></sub>

<sub></sub>




<sub></sub>

 

<sub></sub>

(2)


Dấu " " xảy ra khi   x y z


zx

1

18zx



2z

x



3z

4x

2y

81 x

y

z





<sub></sub>

<sub></sub>



<sub></sub>

 

<sub></sub>

(3) Dấu " " xảy ra khi   x y z
Cộng vế với vế của (1);(2) và (3) ta được:


xy

yz

zx

1

18(xy

yz

zx)



3x

3y

3z



3x

4y

2z

3y

4z

2x

3z

4x

2y

81

x

y

z





<sub></sub>

<sub></sub>



<sub></sub>

 

<sub></sub>



Lại có 3(<i>xy</i><i>yz</i><i>zx</i>)(<i>x</i> <i>y</i> <i>z</i>)2



 



<sub></sub>

<sub></sub>



<sub></sub>

 

<sub></sub>



2


xy

yz

zx

1

6(x

y

z)



3x

3y

3z



3x

4y

2z

3y

4z

2x

3z

4x

2y

81

x

y

z





x

y

z


9



 




(ĐPCM) Dấu " " <sub>xảy ra khi </sub>  x y z


<b>Cách 3: </b>

xy

yz

zx

x

y z



3x

4y 2z

3y 4z 2x

3z 4x

2y

9



 








9xy

9yz

9zx



x

y z



3x

4y 2z

3y 4z 2x

3z 4x

2y



   



</div>
<span class='text_page_counter'>(25)</span><div class='page_container' data-page=25>

25 DAYHOCTOAN.VN
Ta có:


1 1 1 9


x y z x y z x 2y 3x 4y 2z


2xy xy 9xy


x y z x 2y 3x 4y 2z


  


      


  



    


Tương tự:

2yz

yz

9yz


x

 

y z

y 2z

3y 4z 2x

<sub>; </sub>


2zx

zx

9zx



x

 

y z

z 2x

3z 4x

2y


Cộng vế các bất đẳng thức trên, ta được:






2


9xy 9yz 9zx xy yz zx xy yz zx


2


3x 4y 2z 3y 4z 2x 3z 4x 2y x y z x 2y y 2z z 2x


x y z xy yz zx 2 xy yz zx


2 x y z


3 x y z x 2y y 2z z 2x 3 x 2y y 2z z 2x


 
 


   <sub></sub>   <sub></sub>
        <sub></sub>    <sub></sub>
     
 <sub></sub>   <sub></sub>   <sub></sub>   <sub></sub>
  <sub></sub>    <sub></sub> <sub></sub>    <sub></sub>


Ta chỉ cần chứng minh:

xy

yz

zx

x

y z


x

2y

y 2z

z 2x

3



 







Thật vậy:

x 2y 2x



y

 

x y y x



x y

9xy 2x y xy


9 x 2y




         



Tương tự:

2y z

yz

;

2z

x

zx



9

y 2z

9

z 2x



<sub></sub>

<sub></sub>




<sub>3</sub>


Suy ra

xy

yz

zx

x

y z

.


x

2y

y 2z

z 2x

3



 





(Điều phải chứng minh).


Bất đẳng thức được chứng minh.


Dấu bằng xảy ra khi và chỉ khi x y z.
<b>Bài 50. </b> <b>Bài 42. </b> <b> </b>Chứng minh bất đẳng thức :


2
2
2
2
3
2
3
2
3
1
1
1
2
2


2
<i>z</i>
<i>y</i>
<i>x</i>
<i>x</i>
<i>z</i>
<i>z</i>
<i>z</i>
<i>y</i>
<i>y</i>
<i>y</i>
<i>x</i>


<i>x</i> <sub></sub> <sub></sub> <sub></sub>







 biết , ,<i>x y z</i>


là các số thực dương


<b>Bài giải: </b>


Ta có: 1 1 (1)


2
1


1
.
1
2
2


2 2 2


2
3
2
3


























 <i><sub>x</sub></i> <i><sub>y</sub></i> <i>x</i> <i>y</i> <i>x</i> <i>y</i>


<i>x</i>
<i>y</i>


<i>x</i>
<i>x</i>




1 1 (2)
2


1


2 2 2


2
3
























<i>z</i> <i>y</i> <i>z</i>


<i>y</i>
<i>y</i>


; 1 1 (3)
2


1


2 2 2



2
3























<i>x</i> <i>z</i> <i>x</i>


<i>z</i>
<i>z</i>




Cộng với vế của (1);(2);(3) ta được đpcm.


<b>Bài 51. </b> <b>Bài 44. </b> Cho <i>x</i> và <i>y</i>là hai số dương thỏa mãn

x y

 

2010

. Hãy tìm giá trị nhỏ nhất của biểu


thức:




2010

2010


<i>x</i>

<i>y</i>


<i>P</i>


<i>x</i>

<i>y</i>


<b>Bài giải: </b>

 


2010 <i>y</i> 2010 <i>x</i> 2010( 1  1 ) (  )(1)


<i>P</i> <i>x</i> <i>y</i>


</div>
<span class='text_page_counter'>(26)</span><div class='page_container' data-page=26>

26 DAYHOCTOAN.VN
Theo BĐT Cơ si ta có


<i>y</i>
<i>x</i>
<i>y</i>


<i>x</i>   



4
1


1


. Đẳng thức xảy ra khi <i>x</i> <i>y</i>(2)


TheoBĐT Bunhiacốpski ta có( <i>x</i>  <i>y</i> )2 2(<i>x</i> <i>y</i>)2.20104020 <i>x</i>  <i>y</i>  4020(3)
Đẳng thức xảy ra khi <i>x</i> <i>y</i>


Từ (1);(2) và (3) ta suy ra 2010.4 4020  4020
4020


<i>P</i> . Đẳng thức xảy ra khi <i>x</i> <i>y</i>


Vậy <i>P</i> đạt GTNN là 4020 khi<i>x</i> <i>y</i> 1005.


<b>Bài 52. </b> <b>Bài 45. </b> Cho , ,<i>x y z</i> là các số thực dương thỏa mãn x y z  xyz. Chứng minh rằng:
2


2 <sub>1</sub> <sub>1</sub> 2


1 1 1 1


(1)
<i>y</i>


<i>x</i> <i>z</i>


<i>xyz</i>



<i>x</i> <i>y</i> <i>z</i>


 


  <sub></sub> <sub></sub>   <sub></sub>


<b>Bài giải: </b>


Giả thiết suy ra: 1 1 1 1
xyyzzx  .


Ta Có:
2


2


1 x 1 1 1 1 1 1 1 1


x x xy yz zx x y x z


 


 <sub></sub> <sub></sub> <sub></sub> <sub></sub> <sub></sub> <sub></sub>  <sub></sub> 


  


 


 



1 2 1 1


;" " y z
2 x y z


 


 <sub></sub>   <sub></sub>   


 


Viết hai BĐT tương tự rồi cộng lại ta được:


2


2 2


1 1


1 1   1 1


 <i>y</i>  


<i>x</i> <i>z</i>


<i>x</i> <i>y</i> <i>z</i>


1 1 1



3 ;" " x y z
x y z


 <sub> </sub>  <sub>   </sub>


 


 


Ta sẽ CM:3 1 1 1 xyz
x y z


 <sub> </sub> <sub></sub>


 


 

   



2 2


3 xy yz zx xyz x y z


      


xy

 

2 y z

 

2 z x

2 0Điều này lng đúng


Dấu bằng có khi và chỉ khi , ,<i>x y z</i>Vậy (I) được CM, dấu bằng có khi và chỉ khi<i>x</i>  <i>y</i> <i>z</i> 3


<b>Bài 53. </b> <b>Bài 49. </b> <b> </b>Cho , ,<i>a b c</i>là các số dương thỏa mãn : <i>ab bc ca</i>  3 . Chứng minh rằng :





2 2 2


1

1

1

1



1 a

b

c

1 b c

a

1 c a

b

abc



<b>Bài giải: </b>


Từ giả thiết 3 2 2 2


3<b>ab bc ca</b>  3 <b>a b c</b> <b>abc</b>1
Nên ta có :




2 2


1 1 1 1


1 a b c abc a b c  a ab bc ca  3a
Tương tự




2 2


1 1 1 1



;


</div>
<span class='text_page_counter'>(27)</span><div class='page_container' data-page=27>

27 DAYHOCTOAN.VN


<b>Bài 54. </b> <b>Bài 51. </b> <b> </b>Cho a,b,c là ba số thực dương thỏa mãn điều kiện :

3


3c



1


2b



1


a


1






. Tìm giá trị lớn
nhất của biểu thức : <sub>3</sub> <sub>3</sub> <sub>3</sub> <sub>3</sub> <sub>3</sub> <sub>3</sub>


27c


a



3ca


c



27


8b



6bc



b



8


a



2ab


F











.


<b>Bài giải: </b>


Với ,<i>x y</i>0, ta có :<i>x</i>3<i>y</i>3<i>xy x</i>( <i>y</i>) .Từ đó :


y


x



1


y



x


xy




3


3



)



y


1


x


1


(


4


1


y


x



1








Vậy : <sub>3</sub> <sub>3</sub> <sub>3</sub> <sub>3</sub> <sub>3</sub> <sub>3</sub>


c


27


a




3ca


c



27


8b



6bc


b



8


a



2ab


F












2


3


)


a


1


3c




1


3c



1


2b



1


2b



1


a


1


(


4



1

<sub></sub>

<sub></sub>

<sub></sub>

<sub></sub>

<sub></sub>

<sub></sub>





Dấu bằng xảy ra khi và chỉ khi <i>a</i>2<i>b</i>3<i>c</i>1,khi đó


2


3


max


<i>F</i>



<b>Bài 55. </b> Cho <i>a</i> và <i>b</i> là hai số thực dương, <i>m</i> là số tự nhiên. Chứng minh: 1


1 1 2



<i>m</i> <i>m</i>


<i>m</i>


<i>b</i> <i>a</i>


<i>a</i> <i>b</i>




 <sub></sub>  <sub> </sub>  <sub></sub>


   


    <b> </b>


<b>Bài giải: </b>


0, 0


<i>a</i> <i>b</i> nên 1 <i>b</i> 0,1 <i>a</i> 0


<i>a</i> <i>b</i>


   


Áp dụng bđt Cosi: 1


1 1 2 4 2



<i>m</i> <i>m</i>


<i>m</i> <i>m</i>


<i>b</i> <i>a</i>


<i>a</i> <i>b</i>




 <sub></sub>  <sub> </sub>  <sub></sub> <sub></sub>


   


   


Nên 1 1 2 1 1


<i>m</i> <i>m</i> <i>m</i> <i>m</i>


<i>b</i> <i>a</i> <i>b</i> <i>a</i>


<i>a</i> <i>b</i> <i>a</i> <i>b</i>


 <sub></sub>  <sub> </sub>  <sub></sub>  <sub></sub>   <sub></sub> 


       


       



Hay: 1 1 2 2


<i>m</i> <i>m</i> <i>m</i>


<i>b</i> <i>a</i> <i>b</i> <i>a</i>


<i>a</i> <i>b</i> <i>a</i> <i>b</i>


 <sub></sub>  <sub> </sub>  <sub></sub>  <sub> </sub> 


     


     


Áp dụng bđt Cosi cho 2 số <i>b</i> 0,<i>a</i> 0


<i>a</i>  <i>b</i>  : Ta có: 2


<i>b</i> <i>a</i>


<i>a</i> <i>b</i>


<b>Bài 56.</b> <sub>Cho hai số </sub><i><sub>x y</sub></i><sub>,</sub> <sub> thoả mãn </sub> 2 2


4<i>x</i> <i>y</i> 4. Tìm giá trị lớn nhất, giá trị nhỏ nhất của <i>M</i> <i>x</i>23<i>xy</i>2<i>y</i>2


<b>Bài giải: </b>


Ta có



2 2 2 2


2 2 2 2


4( - 3 2 ) 4 -12 8


4 4


<i>x</i> <i>xy</i> <i>y</i> <i>x</i> <i>xy</i> <i>y</i>


<i>M</i>


<i>x</i> <i>y</i> <i>x</i> <i>y</i>


 


 


 


+ <i>y</i>0 thì <i>M</i> 1


+ <i>y</i>0 thì


2
2
4 -12 8


, ( ) (*)


4 1


<i>t</i> <i>t</i> <i>x</i>


<i>M</i> <i>t</i>


<i>t</i> <i>y</i>




 




Gọi <i>M</i> là một giá trị bất kỳ của nó thì

 

* 4

<i>M</i> 1

<i>t</i>212<i>t</i><i>M</i>  8 0có nghiệm <i>t</i>
*, 1, 7


12


</div>
<span class='text_page_counter'>(28)</span><div class='page_container' data-page=28>

28 DAYHOCTOAN.VN
*, <i>M</i> 1 để

 

* có nghiệm thì <sub>’</sub> <sub>4</sub>

2<sub>– 9</sub> <sub>–1</sub>

<sub>0</sub>


<i>D</i>   <i>M</i> <i>M</i>  <i>M</i> 9 85 9; 85


2 2


 <sub></sub> <sub></sub> 


 



 


Vậy giá trị nhỏ nhất của 9 85
2
<i>M</i>  


<b>Bài 57. </b> Cho <i>x y</i>, thỏa mãn: <i>x</i>2<i>y</i>2 1. Tìm giá trị lớn nhất của biểu thức: <i>P</i> 2<i>x</i>2<i>xy</i> 2<i>y</i>2


<b>Bài giải: </b>
<i><b>Cách 1:</b></i> Ta xét 2 trường hợp:


TH1:<i>y</i> 0 <i>x</i>2 1. Khi đó <i>P</i> 2


TH2: <i>y</i>0 khi đó ta có thể đặt <i>x</i><i>k y</i>.
ĐK bài toán trở thành 2 2


(<i>k</i> 1)<i>y</i> 1 và <i>P</i>( 2<i>k</i> <i>k</i> 2)<i>y</i>2
Do đó


2 2 2


2


2 2 2


( 2 2) 2 2


( 2) ( 2) 0


1 ( 1) 1



<i>P</i> <i>k</i> <i>k</i> <i>y</i> <i>k</i> <i>k</i>


<i>P</i> <i>P</i> <i>k</i> <i>k</i> <i>P</i>


<i>k</i> <i>y</i> <i>k</i>


   


        


  (*)


Để (*) có nghiệm thì  0 hay 1 4.( 2)( 2) 0 9 2 0
4


<i>P</i> <i>P</i> <i>P</i>


       suy ra 3


2


<i>P</i>
Khi 3


2


<i>P</i> thay vào (*) được <i>k</i>  1
3 2 2



Suy ra


2
2


2
2


1 1 (3 2 2)


1 1 18 12 2


1
3 2 2
<i>y</i>


<i>k</i>




  


 <sub></sub> <sub> </sub> 


 <sub></sub> 


 





Nếu <i>y</i> 3 2 2


18 12 2




 thì


1
18 12 2
<i>x</i>




Nếu <i>y</i> 2 2 3


18 12 2




 thì


1
18 12 2


<i>x</i> 




Vậy ax



3
2
<i>m</i>


<i>P</i>  với <i>x y</i>; nhận các giá trị như trên.


<i><b>Cách 2:</b></i> Áp dụng BĐT <i>ab</i><i>a</i>2<i>b</i>2 ta được


 



2 2


2 2 2 1 . . 2 2 1 .


<i>P</i> <i>x</i>  <i>y</i>   <i>x</i>  <i>y</i>



2 2


2 2


2 2 2 1 . 2 2 1 . 3


2 2


2 2


<i>x</i> <i>y</i>


<i>x</i> <i>y</i>



  


   


</div>
<span class='text_page_counter'>(29)</span><div class='page_container' data-page=29>

29 DAYHOCTOAN.VN
Trong mặt phẳng tọa độ chọn <i>u</i> và <i>v</i> sao cho


2 2
3


.( )


2


<i>P</i> <i>u</i> <i>v</i>


<i>u</i> <i>v</i> <i>x</i> <i>y</i>


  




  




Khi chọn xong rồi thì ta có ngay BĐT: <i>u</i>   <i>v</i> <i>u v</i>



Từ đó tìm được GTLN của <i>P</i> rồi xét điều kiện xảy ra dấu “=”  <i>u</i> <i>v</i>
<b>Bài 58. </b> Cho các số <i>x y z t</i>; ; ; và <i>x</i>4;<i>y</i>6;<i>z</i>7;<i>t</i>8. Tìm giá trị lớn nhất của


<i>A</i> 1

<i>xyz t</i> 8 <i>xyt z</i> 7 <i>xzt y</i> 6 <i>yzt x</i> 4


<i>xyzt</i>


       


<b>Bài giải: </b>


Từ gt ta có <i>A</i> <i>t</i> 8 <i>z</i> 7 <i>y</i> 6 <i>x</i> 4


<i>t</i> <i>z</i> <i>y</i> <i>x</i>




 <sub></sub>  <sub></sub> <sub></sub> 


= 8 8 7 7 6 6 4 4


8 7 6 4


<i>y</i>


<i>t</i> <i>z</i> <i>x</i>


<i>t</i> <i>z</i> <i>y</i> <i>x</i>





 <sub></sub>  <sub></sub> <sub></sub> 


1 1 1 1


2 8 2 7 2 6 2 4 2 8 2 7 2 6 2 4


<i>t</i> <i>z</i> <i>y</i> <i>x</i>


<i>t</i> <i>z</i> <i>y</i> <i>x</i>


       


Vậy GTLN của <i>A</i> là MaxA= 1 1 1 1


2 82 7 2 6 2 4 khi


4 4


6 6


7 7


8 8


<i>x</i>
<i>y</i>
<i>z</i>
<i>t</i>
  



 




 


 <sub> </sub>


hay


8; 12 14; 16


<i>x</i> <i>y</i> <i>z</i> <i>t</i>


<b>Bài 59. </b> Tìm giá trị nhỏ nhất của biểu thức: 5
8
<i>x</i>
<i>A</i>


<i>x</i>



 Với <i>x</i>

14;

.


<b>Bài giải: </b>



5 3 8 3 1


8 8


2 2


8 8 8


<i>x</i> <i>x</i>


<i>A</i> <i>x</i> <i>x</i>


<i>x</i> <i>x</i> <i>x</i>


 


       


  


- Áp dụng Cosi 8 3 6


2 8


<i>x</i>


<i>x</i>


 <sub></sub> <sub></sub>



 . Đẳng thức xảy ra khi:


8 3


2 8


<i>x</i>


<i>x</i>


 <sub></sub> <sub></sub>


 <i>x</i>14.
- Với 14 1 8 1 6


2 2


<i>x</i>  <i>x</i>  Đẳng thức xảy ra khi <i>x</i>14.
Vậy: 3 6


2


<i>A</i> Đẳng thức xảy ra khi <i>x</i>14.


<b>Bài 60. </b> Cho các số dương <i>a b c</i>; ; thỏa mãn điều kiện <i>a b c</i>  3. Chứng minh: <sub>2</sub> <sub>2</sub> <sub>2</sub> 3


1 1 1 2


<i>a</i> <i>b</i> <i>c</i>



<i>b</i>  <i>c</i>  <i>a</i> 


  


<b>Bài giải: </b>


Ta có:


2 2


2 2


1 1 2 2


<i>a</i> <i>ab</i> <i>ab</i> <i>ab</i>


<i>a</i> <i>a</i> <i>a</i>


<i>b</i>   <i>b</i>   <i>b</i>  


  .


</div>
<span class='text_page_counter'>(30)</span><div class='page_container' data-page=30>

30 DAYHOCTOAN.VN


<sub>2</sub> <sub>2</sub> <sub>2</sub> 3


1 1 1 2 2


<i>a</i> <i>b</i> <i>c</i> <i>ab bc</i> <i>ca</i>



<i>a</i> <i>b</i> <i>c</i>


<i>b</i> <i>c</i> <i>a</i>


 


      


  


(Do <i>a b c</i>  3 nên dễ có: <i>ab bc ca</i>  3).
Đẳng thức xảy ra khi <i>a</i>  <i>b</i> <i>c</i> 1.


<b>Bài 61. </b> CMR với mọi tam giác <i>ABC</i> bất kì ta có:
1) sin2 <i>A</i>sin2<i>B</i>sin2<i>C</i> 2 2 cos .cos .cos<i>A</i> <i>B</i> <i>C</i>


2) 1 sin 1 sin 1 sin 2 cos cos cos


4 4 4


<i>A</i> <i>B</i> <i>C</i>


<i>A</i> <i>B</i> <i>C</i>  


      <sub></sub>   <sub></sub>


 


<b>Bài giải: </b>



1) 1 cos 2 1 cos 2 2 2


sin 2 cos( ).cos( ) cos


2 2


<i>A</i> <i>B</i>


<i>VT</i>      <i>C</i>   <i>A</i><i>B</i> <i>A</i><i>B</i>  <i>C</i>




2 cos<i>C</i> cos(<i>A B</i>) cos<i>C</i> 2 cos<i>C</i> cos(<i>A B</i>) cos(<i>A B</i>)


        


2 2cos .cos .cos<i>A</i> <i>B</i> <i>C</i>
 


2)Áp dụng BĐT Bunhia ta có:


1 sin 1 sin 2(2 sin sin ) 2(2 2sin cos )


2 2


<i>A B</i> <i>A B</i>


<i>A</i> <i>B</i> <i>A</i> <i>B</i>  



       


2 1 cos 2 2 cos , (1)


2 4


<i>C</i> <i>C</i>


  


1 sin 1 sin 2 2 cos , (2) 1 sin 1 sin 2 2 cos , (3)


4 4


<i>A</i> <i>B</i>


<i>B</i> <i>C</i> <i>C</i> <i>A</i>


       


Cộng theo vế ba BĐT trên suy ra điều phải chứng minh.
Dấu đẳng thức xảy ra khi chỉ khi tam giác <i>ABC</i> đều.


<b>Bài 62. </b> Cho đường tròn có bán kính cố định bằng <i>R</i><sub>0</sub>, tam giác <i>ABC</i> nội tiếp đờng trịn đó. Gọi <i>m m m<sub>a</sub></i>, <i><sub>b</sub></i>, <i><sub>c</sub></i>
lần lợt là độ dài đờng trung tuyến kẻ từ đỉnh <i>A B C</i>, , của tam giác <i>ABC</i>. Tìm giá trị nhỏ nhất của
biểu thức

sin

sin

sin



<i>a</i> <i>b</i> <i>c</i>


<i>A</i>

<i>B</i>

<i>C</i>




<i>P</i>



<i>m</i>

<i>m</i>

<i>m</i>





<b>Bài giải: </b>


Áp dụng định lí Sin ta có :


0

1



(1)


2

<i><sub>a</sub></i>

2

<i><sub>b</sub></i>

2

<i><sub>c</sub></i>


<i>a</i>

<i>b</i>

<i>c</i>



<i>P</i>



<i>R</i>

<i>m</i>

<i>m</i>

<i>m</i>



 


 


 


 





Mặt khác:



2 2 2


2

<sub>4</sub>

2

<sub>3</sub>

2

<sub>2</sub>

2 2 2


2

4



<i>a</i> <i>a</i>


<i>b</i>

<i>c</i>

<i>a</i>



<i>m</i>

<i>m</i>

<i>a</i>

<i>a</i>

 

<i>b</i>

<i>c</i>



Theo BĐT Côsi:

<sub>4</sub>

2

<sub>3</sub>

2

<sub>4 3 </sub>



<i>a</i> <i>a</i>


<i>m</i>

<i>a</i>

<i>a m</i>


Suy ra


2


2 2 2


3


2

<i>a</i>


<i>a</i>

<i>a</i>



<i>m</i>

<i>a</i>

 

<i>b</i>

<i>c</i>

,


Tơng tự:


2


2 2 2


3


2

<i>b</i>


<i>b</i>

<i>b</i>



<i>m</i>

<i>a</i>

 

<i>b</i>

<i>c</i>

,


2


2 2 2


3


2

<i>c</i>


<i>c</i>

<i>c</i>



<i>m</i>

<i>a</i>

 

<i>b</i>

<i>c</i>



Thay vào

 

1 :


0

3


<i>P</i>



<i>R</i>




KL: Giá trị nhỏ nhất của


0

3


<i>P</i>



<i>R</i>



</div>
<span class='text_page_counter'>(31)</span><div class='page_container' data-page=31>

31 DAYHOCTOAN.VN


<b>Bài 63. </b> Cho <i>x y z</i>, , là các số thực dương thỏa mãn <i>xy</i><i>yz</i><i>zx</i>3. Chứng minh bất đẳng thức:


2 2 2


3 3 3 1


8 8 8


<i>x</i> <i>y</i> <i>z</i>


<i>x</i> <i>y</i> <i>z</i>



  


   .


<b>Bài giải: </b>


Theo bất đẳng thức Cauchy cho các số thực dương ta có:


2 2


3 2 ( 2) ( 2 4) 6


8 ( 2)( 2 4)


2 2


<i>x</i> <i>x</i> <i>x</i> <i>x</i> <i>x</i>


<i>x</i>   <i>x</i> <i>x</i>  <i>x</i>        


2 2


2
3


2
6
8


<i>x</i> <i>x</i>



<i>x</i> <i>x</i>


<i>x</i>


 


 


Tương tự, ta cũng có: 2 <sub>2</sub> 2 2 <sub>2</sub> 2


3 3


2 2


;


6 6


8 8


<i>y</i> <i>y</i> <i>z</i> <i>z</i>


<i>y</i> <i>y</i> <i>z</i> <i>z</i>


<i>y</i> <i>z</i>


 



   


  .


Từ đó suy ra:


2 2 2 2 2 2


2 2 2


3 3 3


2 2 2


6 6 6


8 8 8


<i>x</i> <i>y</i> <i>z</i> <i>x</i> <i>y</i> <i>z</i>


<i>x</i> <i>x</i> <i>y</i> <i>y</i> <i>z</i> <i>z</i>


<i>x</i> <i>y</i> <i>z</i>


    


     


   . (1)



Mặt khác, theo bất đẳng thức Cauchy - Schwarz:


2 2 2 2


2 2 2 2 2 2


2 2 2 2( )


6 6 6 ( ) 18


<i>x</i> <i>y</i> <i>z</i> <i>x</i> <i>y</i> <i>z</i>


<i>x</i> <i>x</i> <i>y</i> <i>y</i> <i>z</i> <i>z</i> <i>x</i> <i>y</i> <i>z</i> <i>x</i> <i>y</i> <i>z</i>


 


  


            (2)


Ta chứng minh:

 



2


2 2 2


2( )


1 3



( ) 18


<i>x</i> <i>y</i> <i>z</i>


<i>x</i> <i>y</i> <i>z</i> <i>x</i> <i>y</i> <i>z</i>


 




     


Thật vậy, ta có:


<i>x</i>2<i>y</i>2    <i>z</i>2 (<i>x</i> <i>y</i> <i>z</i>) 18


 

2

 



2 18


<i>x</i> <i>y</i> <i>z</i> <i>x</i> <i>y</i> <i>z</i> <i>xy</i> <i>yz</i> <i>zx</i>


         


 

2



12 0


<i>x</i> <i>y</i> <i>z</i> <i>x</i> <i>y</i> <i>z</i>



       


Nên

 

3 2(<i>x</i> <i>y</i> <i>z</i>)2<i>x</i>2<i>y</i>2    <i>z</i>2 (<i>x</i> <i>y</i> <i>z</i>) 18
<i>x</i>2<i>y</i>2    <i>z</i>2 <i>x</i> <i>y</i> <i>z</i> 6


Mặt khác, do <i>x, y, z</i> là các số dương nên ta có: <i>x</i>2<i>y</i>2<i>z</i>2 <i>xy</i><i>yz</i><i>zx</i>,


3( )


<i>x</i>  <i>y</i> <i>z</i> <i>xy</i><i>yz</i><i>zx</i>


Mà <i>xy</i><i>yz</i><i>zx</i>3 nên bất đẳng thức (3) đúng.
Từ (1), (2) và (3), ta có đpcm.


Đẳng thức xảy ra khi và chỉ khi <i>x</i>  <i>y</i> <i>z</i> 1.
<b>Bài 64. </b> Tìm GTLN của

 

2

3



1 . 3 , 1;3


<i>y</i> <i>x</i> <i>x</i> <i>x</i> 


<b>Bài giải: </b>


Ta có :72<i>y</i>(3<i>x</i>3) (6 2 )2  <i>x</i> 3


Áp dụng Côsi :<sub>(3</sub><i><sub>x</sub></i> <sub>3) (3</sub><i><sub>x</sub></i>  <sub>3) (6 2 ) (6 2 ) (6 2 )</sub><i><sub>x</sub></i>   <i><sub>x</sub></i>   <i><sub>x</sub></i> <sub>5 72</sub>5 <i><sub>y</sub></i>


13 13


5 5



3 3


min


8.5 8.5


<i>y</i> <i>y</i>


    khi 3


5


<i>x</i>


<b>Bài 65. </b> Cho <i>a b c</i>, , là các số dương thỏa mãn <i>a b c</i>  1. Chứng minh rằng:


<i>a b</i> <i>b c</i> <i>c a</i> 3


<i>ab c</i> <i>bc a</i> <i>ca b</i>


 <sub></sub>  <sub></sub>  <sub></sub>


  


</div>
<span class='text_page_counter'>(32)</span><div class='page_container' data-page=32>

32 DAYHOCTOAN.VN


Biến đổi 1 1


1 (1 )(1 )



<i>a b</i> <i>c</i> <i>c</i>


<i>ab c</i> <i>ab</i> <i>b a</i> <i>a</i> <i>b</i>


 <sub></sub>  <sub></sub> 


     


Từ đó 1 1 1


(1 )(1 ) (1 )(1 ) (1 )(1 )


<i>c</i> <i>b</i> <i>a</i>


<i>VT</i>


<i>a</i> <i>b</i> <i>c</i> <i>a</i> <i>c</i> <i>b</i>


  


  


     


Do <i>a b c</i>, , dương và <i>a b c</i>  1 nên <i>a b c</i>, , thuộc khoảng

 

0;1  1 <i>a</i>,1<i>b</i>,1<i>c</i> dương
áp dụng bất đẳng thức Côsi cho ba số dương ta được


3



1 1 1


3. . .


(1 )(1 ) (1 )(1 ) (1 )(1 )


<i>c</i> <i>b</i> <i>a</i>


<i>VT</i>


<i>a</i> <i>b</i> <i>c</i> <i>a</i> <i>c</i> <i>b</i>


  




      3 (đpcm)


Đẳng thức xảy ra khi và chỉ khi 1


3


<i>a</i>  <i>b</i> <i>c</i>


<b>Bài 66. </b> Cho các số thực

<i>x, y</i>

thỏa mãn

<i>x</i>

2

  

<i>xy y</i>

2

3

Chứng minh rằng:


2 2


4 3 3

<i>x</i>

<i>xy</i>

3

<i>y</i>

4 3 3




  



<b>Bài giải: </b>


Đặt 2 2


3


<i>P</i><i>x</i> <i>xy</i> <i>y</i> và <i>Q</i><i>x</i>2<i>xy</i><i>y</i>2.
Khi đó 0 <i>Q</i> 3


Nếu <i>y</i>0 thì 0 <i>Q</i> <i>x</i>2 3 và khi đó 4 3 3 0   <i>Q</i> <i>x</i>2 3 4 3 3 bất đẳng thức cần chứng
minh đúng


Nếu <i>y</i>0 thì


2 2


2 2


3
.<i>P</i> .<i>x</i> <i>xy</i> <i>y</i>


<i>P</i> <i>Q</i> <i>Q</i>


<i>Q</i> <i>x</i> <i>xy</i> <i>y</i>


 


 



 
Chia cả tử và mẫu cho 2


0


<i>y</i>  và đặt <i>x</i> <i>t</i>
<i>y</i>  thì


2
2


3
.


1
<i>t</i> <i>t</i>


<i>P</i> <i>Q</i>


<i>t</i> <i>t</i>
 


 
+ Để ý rằng 0 <i>Q</i> 3, nên cần chứng minh:


2
2



3


3 1 3


4 3 3

<i>t</i> <i>t</i>

4 3 3



<i>R</i>


<i>t</i> <i>t</i>
 


 


<sub></sub>

<sub></sub>



Ta có



2


2
2


3


1 1 3 0


1
<i>t</i> <i>t</i>



<i>R</i> <i>R</i> <i>t</i> <i>R</i> <i>t</i> <i>R</i>


<i>t</i> <i>t</i>
 


       


 

 

1
Nếu <i>R</i>1 thì

 

1 là phương trình bậc nhất, có nghiệm <i>t</i> 2

 

2


+ Với <i>R</i>1, do

 

1 ln có nghiệm <i>t</i>, nên  

<i>R</i>1

24

<i>R</i>1



<i>R</i> 3

0
Giải bất phương trình này thu được


3 3


4 3 3

4 3 3



<i>R</i>


<sub> </sub>



và <i>R</i>1

 

3
Từ

   

1 , 2 và

 

3 suy ra điều phải chứng minh.


<b>Bài 67. </b> Chứng minh rằng với mọi <i>ABC</i> nhọn ta luôn có tan .tan .tan<i>A</i> <i>B</i> <i>C</i>1.
<b>Hướng dẫn giải</b>


Vì <i>ABC</i> nhọn nên tan , tan , tan<i>A</i> <i>B</i> <i>C</i>0

 

1 .
Lại có tan

<i>A B</i>

tan

 <i>C</i>

 tan<i>C</i>0.





tan tan
0
tan


<i>A</i> <i>B</i>


<i>A</i> <i>B</i>


 <sub></sub>


 nên



tan tan


tan . tan 1 1


tan


<i>A</i> <i>B</i>


<i>A</i> <i>B</i>


<i>A B</i>


  



 .


</div>
<span class='text_page_counter'>(33)</span><div class='page_container' data-page=33>

33 DAYHOCTOAN.VN
Từ ba bất đẳng thức vừa kể trên suy ra 2 2 2


tan <i>A</i>. tan <i>B</i>. tan <i>C</i>1

 

2 .
Vậy tan .tan .tan<i>A</i> <i>B</i> <i>C</i>1 (do

 

1 và

 

2 ).


<b>Bài 68. </b> Cho các số thực dương <i>x y z</i>, , thỏa mãn điều kiện <i>x</i>5<i>y</i>5<i>z</i>5 3.
Chứng minh rằng


5 5 5


2 4 2 4 2 2


5 7 3


2 3 4 2 14 6 2 19 2


<i>x</i> <i>y</i> <i>z</i>


<i>y</i>  <i>y</i>  <i>z</i> <i>x</i>  <i>z</i> <i>x</i>  <i>x</i> <i>y</i>  <i>x</i><i>x</i>  <i>y</i>  .
<b>Hướng dẫn giải</b>


Ta có


5 5


2



2
2 3 2


2 3 2


<i>x</i> <i>x</i>


<i>y</i> <i>y</i>


<i>y</i> <i>y</i>


    


  (1). Đẳng thức xảy ra khi và chỉ khi <i>y</i>1.
Xét hàm số 4 2


( ) 4 2 14


<i>f z</i> <i>z</i>  <i>z</i><i>x</i>  <i>x</i> ( với <i>x</i> là tham số) trên

0;

ta có
3


( ) 4 4; ( ) 0 1


<i>f z</i>  <i>z</i>  <i>f z</i>   <i>z</i>


Bảng biến thiên của hàm số <i>f z</i>

 

như sau.


Từ bảng biến thiên ta có:

 

2



( ) 1 2 11 10


<i>f z</i>  <i>f</i> <i>x</i>  <i>x</i>  . Đẳng thức xảy ra khi và chỉ khi <i>z</i>1,<i>x</i>1<i>. </i>
Khi đó


5 5 5


4 2


5 5


4 2 14 10 2


<i>y</i> <i>y</i> <i>y</i>


<i>z</i> <i>x</i>  <i>z</i> <i>x</i>   (2).


Xét hàm số 4 2 2


( ) 6 2 19


<i>g x</i> <i>x</i> <i>y</i>  <i>x</i><i>x</i>  <i>y</i> ( với <i>y</i> là tham số) trên

0;

ta có
3


( ) 4 2 6; ( ) 0 1


<i>g x</i>  <i>x</i>  <i>x</i> <i>g x</i>   <i>x</i>


Bảng biến thiên của hàm số <i>g x</i>

 

như sau.



Từ bảng biến thiên ta có:

 

2


( ) 1 2 15 14


<i>g x</i> <i>g</i>  <i>y</i>  <i>y</i>  . Đẳng thức xảy ra khi và chỉ khi <i>x</i>1<i>, y</i>1<i>. </i>
Khi đó


5 5 5


4 2 2


7 7


6 2 19 14 2


<i>z</i> <i>z</i> <i>z</i>


<i>x</i> <i>y</i>  <i>x</i><i>x</i>  <i>y</i>   (3).


</div>
<span class='text_page_counter'>(34)</span><div class='page_container' data-page=34>

34 DAYHOCTOAN.VN


<b>Bài 69. </b> Cho <i>a b c</i>, , 0. Chứng minh rằng 2<i>abc</i>

<i>a</i>1

 

2 <i>b</i> 1

 

2 <i>c</i> 1

2  2

<i>a b c</i>  

2 2.
<b>Hướng dẫn giải</b>


Trong ba số <i>a</i>1,<i>b</i>1,<i>c</i>1 ln tồn tại hai số có tích khơng âm (ngun lý Dirchlet). Khơng mất
tính tổng qt, giả sử

<i>b</i>1



<i>c</i> 1

0.


Sử dụng bất đẳng thức Cauchy-Schwarz ta có



 

2

 

2

 

2

2 1

2





1 1 1 1 2 2 1 2


2


<i>a</i>  <i>b</i>  <i>c</i>  <i>a</i>  <i>b</i> <i>c</i>  <i>a</i>  <i>b c</i> .


Do đó 1

 

2

 

2

2





2 1 1 1 2 1 2


2


<i>abc</i>  <sub></sub> <i>a</i>  <i>b</i>  <i>c</i> <sub></sub><i>abc</i>  <i>a</i>  <i>b c</i>
Mà <i>abc</i>  2

<i>a</i> 1 2



     <i>b c</i>

<i>a b c a b</i>

1



<i>c</i>   1

<i>a b c</i>.


Suy ra 2 1

1

 

2 1

 

2 1

2
2


<i>abc</i>  <sub></sub> <i>a</i>  <i>b</i>  <i>c</i> <sub></sub>  <i>a b c</i> chính là (1).
Dấu “=” khi và chỉ khi <i>a</i>  <i>b</i> <i>c</i> 1 hoặc 0, 1 1


2


<i>a</i> <i>b</i>  <i>c</i> và các hoán vị.


<b>Bài 70. </b> Cho các số thực không âm <i>a b c</i>, , thỏa mãn <i>a b c</i>  1. Chứng minh rằng 3 5 5
4



<i>ab</i> <i>ac</i> <i>bc</i> .
<b>Hướng dẫn giải</b>


Theo giả thiết ta có



1 0 0 1


1 0 0 1


<i>c</i> <i>a b</i> <i>a b</i>


<i>a c</i> <i>b</i> <i>b</i>


       
      
Khi đó




 

2

2



3 5 3 2 3 1 2 1 (


3 2


<i>ab</i> <i>ac</i> <i>bc</i> <i>ab</i> <i>c a b</i> <i>bc</i> <i>ab</i> <i>a b</i> <i>a b</i> <i>b</i> <i>a b</i>


<i>a b</i> <i>a b</i> <i>b</i> <i>b</i> <i>ab</i>



             


 


 <sub></sub>    <sub></sub>   


Xét hàm <i>f x</i>

 

  <i>x</i>2 <i>x x</i>, 

 

0;1 .
Chứng minh được

 

1

 

0;1


4


<i>f x</i>   <i>x</i>


Theo chứng minh trên <i>a b</i> 

 

0;1 ;<i>b</i>

 

0;1 nên

1;

 

1; 0


4 4


<i>f a</i><i>b</i>  <i>f b</i>  <i>ab</i> .
Suy ra 3 5 3.1 2.1 5


4 4 4


<i>ab</i> <i>ac</i> <i>bc</i>  


Dấu đẳng thức xảy ra khi 0; 1
2


<i>a</i> <i>b</i> <i>c</i> .


<b>Bài 71. </b> Cho <i>x y z</i>, , là các số thực dương thỏa mãn . .<i>x y z</i> 3. Tìm GTNN của biểu thức:



9 9 9 9 9 9


3 3 6 6 3 3 6 6 3 3 6 6


2<i>x</i> <i>y</i> 2<i>y</i> <i>z</i> 2<i>z</i> <i>x</i>


<i>P</i>


<i>x y</i> <i>y</i> <i>x</i> <i>z y</i> <i>y</i> <i>z</i> <i>x z</i> <i>z</i> <i>x</i>


  


  


      .


<b>Hướng dẫn giải</b>


</div>
<span class='text_page_counter'>(35)</span><div class='page_container' data-page=35>

35 DAYHOCTOAN.VN
9 9


3
3 3 6 6


2<i>x</i> <i>y</i>


<i>x</i>


<i>x y</i> <i>y</i> <i>x</i>





 


  <b>. </b>Từ đó ta có:


9 9 9 9 9 9


3 3 3


3 3 6 6 3 3 6 6 3 3 6 6


2 2 2


3 3 3


<i>x</i> <i>y</i> <i>y</i> <i>z</i> <i>z</i> <i>x</i>


<i>P</i> <i>x</i> <i>y</i> <i>z</i> <i>xyz</i>


<i>x y</i> <i>y</i> <i>x</i> <i>z y</i> <i>y</i> <i>z</i> <i>x z</i> <i>z</i> <i>x</i>


  


       


      <i><b>. </b></i>


Đẳng thức xảy ra khi và chỉ khi 6



3


<i>x</i>  <i>y</i> <i>z</i> .
<b>Bài 72. </b> Cho a, b, c là các số thực dương thỏa mãn

2 2



<i>a b</i> <i>c</i>  <i>b c</i>. Tìm giá trị nhỏ nhất của biểu thức:

 

2

 

2

 

2







1 1 1 4


1 1 1


1 1 1


<i>P</i>


<i>a</i> <i>b</i> <i>c</i>


<i>a</i> <i>b</i> <i>c</i>


   


  


   .


<b>Hướng dẫn giải</b>


Từ điều kiện rút ra

2

<sub>2</sub> <sub>2</sub>

2


2 2


<i>a b</i> <i>c</i> <i>a b</i> <i>c</i> <i>b</i> <i>c</i> <i>b</i> <i>c</i>


<i>a</i>


        .


Áp dụng BĐT Cauchy ta có:


 

2



 







1 2 4


1 1 1 1 1


1


<i>P</i>


<i>b</i> <i>c</i> <i>a</i> <i>b</i> <i>c</i>


<i>a</i>


  


    









2
2


2


2
1


1 1 2


1 1 2 2


4 4


<i>a</i>


<i>b</i> <i>c</i> <i>b c</i>


<i>a</i> <i>a</i>




 


      <sub></sub>  <sub></sub> 


  .



Suy ra:


 



3 2


3


2 6 1


, 0
1


<i>a</i> <i>a</i> <i>a</i>


<i>P</i> <i>f a a</i>


<i>a</i>


  


  


 .


 


4


2 5 1 1



' 0


5
1


<i>a</i>


<i>f</i> <i>a</i> <i>a</i>


<i>a</i>


   


 .


Lập bảng biến thiên rút ra được: 91


108


<i>P</i> , xảy ra khi 1, 5
5


<i>a</i> <i>b</i> <i>c</i> .


<b>Bài 73. </b> Cho <i>x y z</i>, , là các số thực dương thay đổi và thỏa mãn:

<i>x</i>

2

<i>y</i>

2

<i>z</i>

2

<i>xyz</i>

. Chứng minh:


2 2 2



1
2


<i>x</i> <i>y</i> <i>z</i>


<i>x</i> <i>yz</i> <i>y</i> <i>zx</i><i>z</i> <i>xy</i>


<b>Lời giải </b>


Đặt <i>P</i> <sub>2</sub> <i>x</i> <sub>2</sub> <i>y</i> <sub>2</sub> <i>z</i>


<i>x</i> <i>yz</i> <i>y</i> <i>zx</i> <i>z</i> <i>xy</i>


  


  


Vì , ,<i>x y z</i>0, áp dụng BĐT Cơsi ta có


2 2 2


2 2 2


1 1 1


2 2 2


2 2 2


1 2 2 2 1 1 1 1 1 1 1





4 4


1 1 1 1


2 2 2 2


<i>x</i> <i>y</i> <i>z</i>


<i>P</i>


<i>yz</i> <i>zx</i> <i>xy</i>


<i>x yz</i> <i>y zx</i> <i>z xy</i>


<i>y</i> <i>z</i> <i>z</i> <i>x</i> <i>x</i> <i>y</i>


<i>yz</i> <i>zx</i> <i>xy</i>


<i>xy</i> <i>yz</i> <i>zx</i> <i>x</i> <i>y</i> <i>z</i> <i>xyz</i>


<i>xyz</i> <i>xyz</i> <i>xyz</i>


     


  <sub></sub> <sub></sub>


 <sub></sub>   <sub></sub>       



 


 


   


   


</div>
<span class='text_page_counter'>(36)</span><div class='page_container' data-page=36>

36 DAYHOCTOAN.VN


<b>Bài 74. </b> Cho các số không âm <i>a b c</i>, , thỏa mãn <i>a b c</i>  2. Chứng minh rằng<b>:</b>


2


3 3 3


4 4 4


<i>a</i> <i>b</i> <i>c</i>


<i>a</i> <i>bc</i> <i>b</i> <i>ca</i> <i>c</i> <i>ab</i>


  


  


<b>Lời giải </b>


Áp dụng BĐT Cauchy Shwars ta có





2


3 3 3


4 4 4


=2


3 3 3


4 4 4


<i>a</i> <i>b</i> <i>c</i>


<i>VT</i> <i>a b c</i>


<i>a</i> <i>bc</i> <i>b</i> <i>ca</i> <i>c</i> <i>ab</i>


<i>a</i> <i>b</i> <i>c</i>


<i>a</i> <i>bc</i> <i>b</i> <i>ca</i> <i>c</i> <i>ab</i>


 


 


   <sub></sub>   <sub></sub>



    


 


 


 


 


 


    


 




Mặt khác 3 3


3 4 3 4 3 4 3


4


<i>cylic</i>


<i>a</i> <i>bc</i> <i>ca</i> <i>ab</i>


<i>a</i> <i>bc</i> <i>b</i> <i>ca</i> <i>c</i> <i>ab</i>



<i>a</i> <i>bc</i>


 


  <sub></sub>   <sub></sub>


  


 
















2


2
2 2 2 2 2 2



2
2 2 2 2 2 2


3


4 3 4 3 4 3


=


4


= 1


2
<i>bc ca</i> <i>ab</i>


<i>bc</i> <i>a</i> <i>bc</i> <i>ca</i> <i>b</i> <i>ca</i> <i>ab</i> <i>c</i> <i>ab</i>


<i>bc ca</i> <i>ab</i>


<i>b c</i> <i>c a</i> <i>a b</i> <i>abc</i>
<i>bc ca</i> <i>ab</i>


<i>b c</i> <i>c a</i> <i>a b</i> <i>abc a b c</i>
 




    



 


  


 




    




Tư đó suy ra


3 1 2


3 3 3


4 4 4


<i>a</i> <i>b</i> <i>c</i>


<i>a</i> <i>bc</i> <i>b</i> <i>ca</i> <i>c</i> <i>ab</i>


    


  


Dấu bằng xẩy ra khi

, ,

2 2 2, ,
3 3 3

<i>a b c</i>   <sub></sub>


  hoặc

<i>a b c</i>, ,

 

 1,1, 0

cùng các hốn vị của nó.
<b>Bài 75. </b> Cho <i>x y z</i>, , là các số thực thỏa mãn <i>x</i>2<i>y</i>2<i>z</i>2 9, tìm giá trị lớn nhất của biểu thức:




2 + –
<i>F</i>  <i>x</i><i>y z</i> <i>xyz</i>.


<b>Bài giải: </b>


Do vai trò <i>x y z</i>, , bình đẳng giả sử <i>x</i>  <i>y</i>  <i>z</i> 2


3


<i>z</i>


  .


Ta có <i>F</i> 2

<i>x</i><i>y</i>

 

 2 –<i>xy z</i>



Áp dụng bất đẳng thức Bunhiacốpxki ta có:


<sub></sub>2

<i>x</i><i>y</i>

 

 2 <i>xy z</i>

<sub></sub>2 <sub></sub>22 

2 <i>xy</i>

 

2 <sub> </sub> <i>x</i><i>y</i>

2<i>z</i>2<sub></sub>


 

2

<sub>2</sub> <sub>2</sub>



<sub>2</sub> <sub>2</sub> <sub>2</sub>



2 <i>x</i> <i>y</i> 2 <i>xy z</i> 8 4<i>xy</i> <i>x y</i> <i>x</i> 2<i>xy</i> <i>y</i> <i>z</i>



          


 


</div>
<span class='text_page_counter'>(37)</span><div class='page_container' data-page=37>

37 DAYHOCTOAN.VN


2 2 2


2 <i>xy</i> <i>x</i> <i>y</i>  9 <i>z</i> 6 nên   3 <i>t</i> 3, từ đó bất đẳng thức (1) trở thành:


2



8 4 9 2


<i>F</i>  <i>t</i> <i>t</i>  <i>t</i>  <i>F</i> 2<i>t</i>3 <i>t</i>2 20<i>t</i>72
Xét hàm số

 

3 2


2 20 72


<i>f t</i>  <i>t</i>  <i>t</i> <i>t</i> với <i>t</i> 

3;3

, ta chứng minh:
<i>f t</i>

 

2<i>t</i>3 <i>t</i>2 20<i>t</i>72 100 (2)


3 2


2<i>t</i> <i>t</i> 20<i>t</i> 28 0


    


2




2 2 3 14 0


<i>t</i> <i>t</i> <i>t</i>


    


 

2



2 2 7 0


<i>t</i> <i>t</i>


    .


Do <i>t</i> 

3;3

nên bất đẳng thức trên đúng, dấu ‘=’ xảy ra khi và chỉ khi <i>t</i> 2.
Suy ra: <i>F</i>10 (3)


Dấu ‘=” của bđt (1) xảy ra 


2 2 2


2 2


0
9
<i>xy</i>


<i>x</i> <i>y</i> <i>z</i>


<i>x</i> <i>y</i> <i>z</i>





 <sub></sub> <sub></sub>


 


   


. Xét hệ: 2 2 2


2 2


0
9
2


<i>xy</i>


<i>x</i> <i>y</i> <i>z</i>


<i>x</i> <i>y</i> <i>z</i>


<i>xy</i>




 <sub></sub> <sub></sub>



 


 <sub></sub> <sub></sub> <sub></sub>


  



<i>x y z</i>, ,

 

2; 1; 2 ,

 

1; 2; 2



    .


Vậy gtln của <i>F</i>10 đạt được 

<i>x y z</i>, ,

 

 2; 1; 2

và các hoán vị của chúng.
<b>Bài 76. </b> Giả sử <i>x y</i>, là các số thực dương thỏa mãn <i>x</i> <i>y</i> 1. Chứng minh rằng:


3 3


2 5


3 3


<i>xy</i>  <i>x</i>  <i>y</i> >


36 2 104 5
81


<b>Bài giải: </b>



<sub></sub>

<sub></sub>

3


3 3 3 3 2 3


1 1 1 1 4 4


4
3<i>xy</i><i>x</i> <i>y</i> 3<i>xy x</i><i>y</i>  <i>x</i> <i>y</i>  <i>x</i> 3<i>xy x</i><i>y</i> <i>y</i>  <i><sub>x</sub></i><i><sub>y</sub></i>  (1)
Mặt khác,




2


2


1 1 1 4


4 1


4 9 9


<i>x</i> <i>y</i> <i>xy</i>


<i>xy</i> <i>x</i> <i>y</i> <i>xy</i>


      


 (2)


Từ (1) và (2) ta có:



3 3


2 5


3 3


<i>xy</i>  <i>x</i>  <i>y</i>




3 3


4 9 2 5


9 2 5 5 1 1 4 5 36 2 104 5


9<i>xy</i> 3 <i>x</i> <i>y</i> 3<i>xy</i> 81 3 81




 


 


  <sub></sub>  <sub></sub>  




 



<b>Bài 77. </b> Cho <i>a b c</i>, , là các số thực dương. Chứng minh rằng 2<i>a</i> 2<i>b</i> 2<i>c</i> 3


<i>a</i><i>b</i><i>b</i><i>c</i><i>c</i><i>a</i> 


<b>Lời giải </b>


Đặt <i>x</i> <i>b</i>, <i>y</i> <i>c</i>, <i>z</i> <i>a</i>


<i>a</i> <i>b</i> <i>c</i>


   , ta có <i>x y z</i>, , 0 và <i>xyz</i>1.
Bất đẳng thức đã cho trở thành: 2 <sub>2</sub> 2 <sub>2</sub> 2 <sub>2</sub> 3


</div>
<span class='text_page_counter'>(38)</span><div class='page_container' data-page=38>

38 DAYHOCTOAN.VN
Giả sử <i>xy</i>  1 <i>z</i> 1


* Ta chứng minh đẳng thức sau: 1 <sub>2</sub> 1 <sub>2</sub> 2

 

1
1<i>x</i> 1<i>y</i> 1<i>xy</i>
Thật vậy,

 

2 2

2



2



1  2<i>x</i> <i>y</i> 1<i>xy</i> 2 1<i>x</i> 1<i>y</i>
 

1 <i>xy</i>



<i>x</i><i>y</i>

2 0(đúng)


Ta có: Theo bất đẳng thức Bunhiacopski
2


2 2 2 2 2 2


2 2 2 2 1 1



2 4


1 <i>x</i> 1 <i>y</i> 1 <i>x</i> 1 <i>y</i> 1 <i>x</i> 1 <i>y</i>


     


    


     


 <sub></sub> <sub></sub>  <sub></sub> <sub></sub> <sub></sub> <sub></sub> <sub></sub> <sub></sub> <sub></sub> <sub></sub>


 


Theo bất đẳng thức (1) suy ra: 4 1 <sub>2</sub> 1 <sub>2</sub> 8 8


1 1 1 1


<i>z</i>


<i>x</i> <i>y</i> <i>xy</i> <i>z</i>


 <sub></sub> <sub></sub> <sub></sub>


 <sub></sub> <sub></sub>  <sub></sub> <sub></sub>


 


Suy ra 2 <sub>2</sub> 2 <sub>2</sub> 2 2



1 1 1


<i>z</i>


<i>x</i>  <i>y</i>  <i>z</i>


  


Mặt khác, ta lại có 2 <sub>2</sub> 2
1<i>z</i> 1<i>z</i>


Suy ra 2 <sub>2</sub> 2 <sub>2</sub> 2 <sub>2</sub> 2 2 2


1 1 1 1 1


<i>z</i>


<i>x</i>  <i>y</i>  <i>z</i>  <i>z</i>  <i>z</i>


    


Do vậy, ta sẽ chứng minh : 2 2 2 3


1 1


<i>z</i>


<i>z</i>  <i>z</i> 



 


Thật vậy, ta có: 2 2 2 3 2 2 1

2 3 1



1 1


<i>z</i>


<i>z</i> <i>z</i> <i>z</i>


<i>z</i>  <i>z</i>     


 


 


2<i>z</i> 2 2 1<i>z</i> <i>z</i> 1 <i>z</i> 0


      

2<i>z</i> 1<i>z</i>

20(luôn đúng).
Vậy bất đẳng thức đã được chứng minh.


Dấu “=” xảy ra khi <i>x</i>  <i>y</i> <i>z</i> 1.


<b>Bài 78. </b> Cho các <b>số</b> thực không âm <i>a b c</i>, , thỏa mãn 2 2 2


2


<i>a</i>   <i>b</i> <i>c</i> . Chứng minh rằng


1 1 1 9



2<i>ab</i>2<i>bc</i>2<i>ca</i> 4


<b>Lời giải </b>


Bất đẳng thức tương đương với


2 2 2 9 3


1 1 1 3


2 2 2 2 2 2 2 2


<i>ab</i> <i>bc</i> <i>ca</i>


<i>ab</i> <i>bc</i> <i>ca</i> <i>ab</i> <i>bc</i> <i>ca</i>


 <sub> </sub>  <sub> </sub>  <sub>   </sub> <sub></sub> <sub></sub> <sub></sub>


 <sub></sub>   <sub></sub>   <sub></sub>  <sub></sub> <sub></sub> <sub></sub>


     


2 2 2

2 2 2 2 2 2 2 2 2


2a 2a 2a


2 2 2a ( )


<i>ab</i> <i>b</i> <i>b</i> <i>b</i>



<i>ab</i>  <i>a</i> <i>b</i> <i>c</i> <i>b</i>  <i>a</i> <i>b</i> <i>c</i> <i>c</i> <i>a b</i> <i>a</i> <i>b</i> <i>c</i> <i>c</i>


           


2 2 2


2 2 2 2 2 2 2 2


1 ( ) 1


2 ( ) ( ) 2


<i>a b</i> <i>a</i> <i>b</i>


<i>c</i> <i>a</i> <i>c</i> <i>b</i> <i>c</i> <i>a</i> <i>c</i> <i>b</i>


 




  <sub></sub>  <sub></sub>


   <sub></sub>   <sub></sub> (Sử dụng BẤT ĐẲNG THỨC Cauchy-Schwarz)


Tương tự ta có


2 2 2 2


2 2 2 2 2 2 2 2



1 1


;


2 2 2 2


<i>bc</i> <i>b</i> <i>c</i> <i>ca</i> <i>c</i> <i>a</i>


<i>bc</i> <i>a</i> <i>b</i> <i>a</i> <i>c</i> <i>ca</i> <i>c</i> <i>b</i> <i>a</i> <i>b</i>


   


 <sub></sub>  <sub></sub>  <sub></sub>  <sub></sub>


</div>
<span class='text_page_counter'>(39)</span><div class='page_container' data-page=39>

39 DAYHOCTOAN.VN


Cộng vế với vế các bất đẳng thức trên ta có đpcm, chỉ ra dấu "=" khi 2.
3
<i>a</i>  <i>b</i> <i>c</i>
<b>Bài 79. </b> Cho ba số dương <i>a, b, c</i>. Chứng minh rằng:


4 4 4


1 1 1 25


<i>a</i> <i>b</i> <i>c</i>


<i>b c</i> <i>c a</i> <i>a b</i>


 <sub></sub>  <sub></sub>  <sub> </sub>



 <sub></sub>  <sub></sub>  <sub></sub> 


    . (1)


<b>Lời giải </b>


Không giảm tổng quát, giả sử c là số lớn nhất trong ba số <i>a, b, c</i>.
Đặt <i>S</i>   <i>a</i> <i>b</i> <i>c R</i>, <i>ab</i><i>bc</i><i>ca P</i>, <i>abc</i>.


3 2


3 2
3


3


(1) ( 3 )( 3 )( 3 ) 25( )( )( )


3 ( ) 9 ( ) 27


25[ ( ) ( ) ]


4 9 27 25( )


4 13 0.(2)


<i>S</i> <i>a S</i> <i>b S</i> <i>c</i> <i>S</i> <i>a S</i> <i>b S</i> <i>c</i>


<i>S</i> <i>S a b c</i> <i>S ab bc ca</i> <i>abc</i>



<i>S</i> <i>S a b c</i> <i>S ab bc ca</i> <i>abc</i>


<i>S</i> <i>SR</i> <i>P</i> <i>SR</i> <i>P</i>


<i>S</i> <i>SR</i> <i>P</i>


       


       


       


    


   




Ta chứng minh (2) đúng. Thật vậy,
3


2
2
2


2


(2) ( ) 4( )( ) 13



=( )[( ) -4( )] 13
= ( )[( ) -4 ] 13


=( )( ) -4 ( ) 13
( )( ) (9 4 4


<i>VT</i> <i>a b c</i> <i>a b c ab bc ca</i>


<i>a b c</i> <i>a b c</i> <i>ab bc ca</i> <i>abc</i>


<i>a b c</i> <i>a b c</i> <i>ab</i> <i>abc</i>


<i>a b c a b c</i> <i>ab a b c</i> <i>abc</i>


<i>a b c a b c</i> <i>ab c</i> <i>a</i>


        


      


    


      


        ) 0


(do 9 4 4 0).


<i>b</i>



<i>c</i> <i>a</i> <i>b</i> <i>c</i>




   


Vậy ta có điều phải chứng minh.


<b>Bài 80. </b> Cho ba số thực dương <i>a, b, c</i> chứng minh rằng


2 2 2


2 2 2 2 2 2 2 2 2 0


2 2 2


<i>a</i> <i>bc</i> <i>b</i> <i>ca</i> <i>c</i> <i>ab</i>


<i>a</i> <i>b</i> <i>c</i> <i>b</i> <i>c</i> <i>a</i> <i>c</i> <i>a</i> <i>b</i>


  


  


     


<b>Lời giải </b>


Bất đẳng thức tương đương với



2 2 2


2 2 2 2 2 2 2 2 2


( ) ( ) ( )


3


2 2 2


<i>b c</i> <i>c</i> <i>a</i> <i>a b</i>


<i>a</i> <i>b</i> <i>c</i> <i>b</i> <i>c</i> <i>a</i> <i>c</i> <i>a</i> <i>b</i>


  


  


     


Ta có


2 2 2 2


2 2 2 2 2 2 2 2 2 2 2


( ) ( )


2 ( ) ( )



<i>b c</i> <i>b c</i> <i>b</i> <i>c</i>


<i>a</i> <i>b</i> <i>c</i> <i>a</i> <i>b</i> <i>a</i> <i>c</i> <i>a</i> <i>b</i> <i>a</i> <i>c</i>


 <sub></sub>  <sub></sub> <sub></sub>


       <b>. </b>


Tương tự với hai BẤT ĐẲNG THỨC còn lại suy ra đpcm.
<b>Bài 81. </b> Cho <i>a b c</i>; ; 0 thỏa mãn <i>a b c abc</i>   4. Chứng minh rằng




2
2


<i>a</i> <i>b</i> <i>c</i>


<i>a b c</i>
<i>b c</i>  <i>a c</i>  <i>a b</i>   


<b>Lời giải </b>


Áp dụng bất đẳng thức Bunhiakopxki ta được






2


<i>a b c</i>


<i>a</i> <i>b</i> <i>c</i>


<i>T</i>


<i>c b</i> <i>a</i> <i>c</i> <i>a b</i> <i>a b c</i> <i>b a</i> <i>c</i> <i>c a b</i>


 


   


</div>
<span class='text_page_counter'>(40)</span><div class='page_container' data-page=40>

40 DAYHOCTOAN.VN


Lại có

<i>a b c</i> <i>b a c</i> <i>c a b</i>

2 

<i>a b c</i> 



2<i>ab</i>2<i>bc</i>2<i>ac</i>


Suy ra:





2


<i>a b c</i> <i>a b c</i>


<i>T</i>


<i>ab bc</i> <i>ac</i>


   





  (*)


Ta sẽ chứng minh <i>a b c</i>  <i>ab bc ca</i>  (1)


Đặt ; ; ( 2)


4


<i>S</i>
<i>a</i> <i>b</i> <i>S</i> <i>ab</i><i>P</i> <i>P</i>


Từ giả thiết suy ra 4
1
<i>S</i>
<i>c</i>


<i>P</i>



  <i>S</i> 4.


Vậy

(

1

)

4

4

1

 

2

2


1 1


<i>S</i> <i>S</i>


<i>S</i>



<i>S</i> <i>P</i> <i>P P</i> <i>S</i> <i>S</i>


<i>P</i> <i>P</i>





       


  (2)


Nếu <i>P</i>   1 <i>S</i> 0 <i>VT</i> 0 <i>VP</i>.
Nếu <i>P</i>  1 <i>S</i> 0. Ta có



2 2


1 1


4 4


<i>S</i> <i>S</i>


<i>P P</i> <i>S</i>  <sub></sub>  <i>S</i><sub></sub>


  (vì

4



2


<i>S</i>



<i>P</i>

).


Suy ra

 



2


2 2


1 2 2


16


<i>S</i>


<i>P P</i> <i>S</i>  <i>S</i>  <i>S</i> (vì <i>S</i> 4).
Vậy: <i>a b c</i>  <i>ab bc ac</i>  . Từ (*) suy ra


2


<i>a</i> <i>b</i> <i>c</i>


<i>T</i>    .


<b>Bài 82. </b> Cho <i>a b c</i>, , là các số thực dương. Chứng minh bất đẳng thức








2 2 2


10



2


<i>a</i> <i>b</i> <i>c</i> <i>abc</i>


<i>b c</i> <i>a</i> <i>c</i> <i>a b</i> <i>a b b c c</i> <i>a</i>


  <sub></sub>  <sub></sub>  <sub></sub> <sub></sub>


 <sub></sub>   <sub></sub>   <sub></sub>  <sub></sub> <sub></sub> <sub></sub>


      .


<b>Lời giải </b>


Biến đổi bất đẳng thức như sau




<sub></sub>

<sub></sub>



2


10 2


<i>a</i> <i>a b a c</i>


<i>abc</i> <i>a b</i>


<i>b c</i>


 



  






<sub></sub>

<sub></sub>

<sub></sub>

<sub></sub>



2 2


3 3


2 10 2


<i>a</i> <i>a</i> <i>bc</i>


<i>a</i> <i>a</i> <i>abc</i> <i>a b</i>


<i>b c</i>


 <sub></sub> 


 


     




 



 






<sub></sub>

<sub></sub>

<sub></sub>

<sub></sub>



2


3


2 5 0


<i>a</i> <i>a b a c</i>


<i>a</i> <i>abc</i> <i>a b</i>


<i>b c</i>


 


     








<sub></sub>

<sub></sub>

<sub></sub>

<sub></sub>



2



3 2


2 0


<i>a</i> <i>a b a c</i>


<i>a</i> <i>abc a</i> <i>b c</i>
<i>b c</i>


 


     








<sub></sub>

<sub></sub>

<sub></sub>



<sub>   </sub>



2


2 0


2 2


0
<i>a</i> <i>a b</i> <i>a c</i>



<i>a a b</i> <i>a c</i>
<i>b c</i>


<i>a</i> <i>b</i> <i>c</i>


<i>a a b</i> <i>a c</i>
<i>b c</i>


 


    



 


   










Theo bất đẳng thức Schur thì bất đẳng thức cuối đúng nên bất đẳng thức ban đầu được chứng
minh.


<b>Bài 83. </b> Cho , ,<i>x y z</i>0 và <i>x</i>  <i>y</i> <i>z</i> 1. Chứng minh rằng:



2 2


2 2 1


</div>
<span class='text_page_counter'>(41)</span><div class='page_container' data-page=41>

41 DAYHOCTOAN.VN


<b>Lời giải </b>


Cần chứng minh








2 2


2 2


2 2


1


2 2


<i>xy</i> <i>z x</i> <i>y</i> <i>z</i> <i>x</i> <i>y</i>


<i>x</i> <i>y</i> <i>z</i> <i>xy</i>



<i>x</i> <i>z</i> <i>y</i> <i>z</i> <i>x</i> <i>y</i> <i>x</i> <i>y</i> <i>z</i> <i>xy</i>


    



  


        


Sử dụng bất đẳng thức Cauchy, ta có:


 

 



2 2 2 2


2 2


2 2


2 2 2 4 2


2 2


<i>x</i> <i>y</i> <i>x</i> <i>y</i> <i>xy</i> <i>x</i> <i>y</i> <i>x</i> <i>y</i> <i>x</i> <i>y</i>


<i>x</i> <i>y</i> <i>x</i> <i>y</i>


         


   



Cần chứng minh:

<i>z</i><i>x</i>



<i>z</i><i>y</i>

 <i>z</i> <i>xy</i>




2


2 2


2 0


<i>z</i> <i>xy</i> <i>z x</i> <i>y</i> <i>z</i> <i>xy</i> <i>z xy</i> <i>z</i> <i>x</i> <i>y</i>


          (Đúng)


Đẳng thức xảy ra


1
2
0
<i>x</i> <i>y</i>
<i>z</i>
  


 
 


.



<b>Bài 84. </b> Cho 3 số thực không âm <i>x y</i>, , z thỏa mãn 2 2 2
2


<i>x</i> <i>y</i> <i>z</i>  . Chứng minh rằng:


2 2 2


(<i>x</i><i>y y</i>)( <i>z z</i>)(  <i>x</i>) 4<i>xyz xy</i>( <i>yz</i><i>zx</i> 2) 4<i>xyz x</i>( <i>y</i> <i>z</i> )


<b>Lời giải </b>


Ta chứng minh bổ đề sau:


Bổ đề: Xét biểu thức 2 2 2


( ) <i><sub>z</sub></i> ( ) <i><sub>x</sub></i> ( ) <i><sub>y</sub></i>


<i>S</i> <i>x</i><i>y S</i>  <i>y</i><i>z S</i>  <i>x</i> <i>z S</i>
Nếu <i>x</i> <i>y</i> <i>z</i> và <i>S Sy</i>; <i>y</i><i>S Sz</i>; <i>y</i><i>Sx</i> 0 thì <i>S</i>0


Chứng minh


2 2 2


2 2


( ) ( ) ( )


( )( ) ( )( ) 2( )(y z) 0



<i>z</i> <i>x</i> <i>y</i>


<i>z</i> <i>y</i> <i>y</i> <i>x</i> <i>y</i>


<i>S</i> <i>x</i> <i>y S</i> <i>y</i> <i>z S</i> <i>x</i> <i>z S</i>


<i>S</i> <i>S</i> <i>x</i> <i>y</i> <i>S</i> <i>S</i> <i>y</i> <i>z</i> <i>x</i> <i>y</i> <i>S</i>


     


         


Chứng minh


Nếu . .<i>x y z</i>0 Bất đẳng thức luôn đúng
Nếu . .<i>x y z</i>0. Ta có


2 2 2


( )( )( ) 4 ( 2) 4 ( )


( )( )( )


2 2
4


<i>x</i> <i>y y</i> <i>z z</i> <i>x</i> <i>xyz xy</i> <i>yz</i> <i>zx</i> <i>xyz x</i> <i>y</i> <i>z</i>


<i>x</i> <i>y y</i> <i>z z</i> <i>x</i>



<i>xy</i> <i>yz</i> <i>zx</i>
<i>xyz</i>


         


  


     


( )( )( )


2 2 0


4


<i>x</i> <i>y y</i> <i>z z</i> <i>x</i>


<i>xy</i> <i>yz</i> <i>zx</i>
<i>xyz</i>


  


       (1)


Ta có


2 2 2


( )(y )( ) ( ) ( ) ( )



2


4 4


<i>x</i> <i>y</i> <i>z z</i> <i>x</i> <i>x y</i> <i>z</i> <i>y z</i> <i>x</i> <i>z x</i> <i>y</i>


<i>xyz</i> <i>xyz</i>


       


</div>
<span class='text_page_counter'>(42)</span><div class='page_container' data-page=42>

42 DAYHOCTOAN.VN


2 2 2


2 2 2


1


2(1 ) ( ) ( ) ( )


2


<i>xy</i> <i>yz</i> <i>zx</i>


<i>x</i> <i>y</i> <i>y</i> <i>z</i> <i>z</i> <i>x</i>


<i>x</i> <i>y</i> <i>z</i>



  <sub></sub> <sub></sub>


  <sub></sub>      <sub></sub>


 


Do đó (1) 2 1 1 2 1 1 2 1 1


( ) ( ) ( ) 0


4 2 4 2 4 2


<i>x</i> <i>y</i> <i>y</i> <i>z</i> <i>z</i> <i>x</i>


<i>xy</i> <i>yz</i> <i>zx</i>


     


  <sub></sub>  <sub></sub>  <sub></sub>  <sub></sub>  <sub></sub>  <sub></sub>


 


   


Đặt 1 1 ; 1 1 ; 1 1


4 2 4 2 4 2


<i>z</i> <i>x</i> <i>y</i>



<i>S</i> <i>S</i> <i>S</i>


<i>xy</i> <i>yz</i> <i>xz</i>


     


<sub></sub>  <sub></sub> <sub></sub>  <sub></sub> <sub></sub>  <sub></sub>


 


   


Giả sử 1 1 0


4 2


<i>y</i>


<i>x</i> <i>y</i> <i>z</i> <i>S</i>


<i>xz</i>


 


   <sub></sub>  <sub></sub>


 


Mà <sub>2</sub> <sub>2</sub> <sub>2</sub> <sub>2</sub>



1 1 1 1 4


+ =


4 2 4 2 4


2 (x 2 ) 4 2


(y )( ) 4 2


= 0


4 2 4 2


<i>y</i> <i>x</i>


<i>y</i> <i>z</i> <i>xyz</i>


<i>S</i> <i>S</i>


<i>xz</i> <i>yz</i> <i>xyz</i>


<i>yz</i> <i>yz</i> <i>xyz</i>


<i>z x</i> <i>y</i> <i>z</i> <i>xyz</i>


<i>xyz</i> <i>xyz</i>


 



   


 


   


 


Tương tự S<i>y</i><i>Sz</i> 0


Áp dụng bổ đề suy ra điều cần chứng minh.
Đẳng thức xảy ra khi 2


3
<i>x</i>  <i>y</i> <i>z</i> .


<b>Bài 85. </b> Cho <i>a b c</i>, , là các số thực dương. Tìm giá trị nhỏ nhất của biểu thức:


<i>a</i> <i>b</i> <i>c</i>


<i>P</i>


<i>b c</i> <i>c</i> <i>a</i> <i>a b</i>


     


<sub></sub> <sub></sub> <sub></sub> <sub></sub> <sub></sub> <sub></sub>


  



     


3 3 3


<b>Bài giải: </b>


Ta có: <i>a</i> <i>b</i>  <i>a b</i> <sub></sub>


 


3


3 3


2 2



<i>c</i> <i>c</i>


<i>a b</i> <i>a</i> <i>b</i>


 


<sub></sub> <sub></sub> 


 


 


3 3



3 3


4


Do đó




<i>c</i> <i>c</i>


<i>a b</i> <i>a</i> <i>b</i>


 <sub> </sub>


 <sub></sub>  <sub></sub>


 


3 <sub>3</sub>


3 3


4 (1). Đẳng thức xảy ra  <i>a</i> <i>b</i>.


Tương tự ta có:




<i>a</i> <i>a</i>



<i>b c</i> <i>b</i> <i>c</i>


 <sub> </sub>


 <sub></sub>  <sub></sub>


 


3 <sub>3</sub>


3 3


4 (2),



<i>b</i> <i>b</i>


<i>c a</i> <i>c</i> <i>a</i>


 <sub> </sub>


 <sub></sub>  <sub></sub>


 


3 <sub>3</sub>


3 3


4 (3).



Từ (1), (2), (3) ta có


 

 





<i>a</i> <i>b</i> <i>c</i>


<i>P</i> <i>a</i> <i>b</i> <i>b</i> <i>c</i> <i>c</i> <i>a</i>


<i>c</i> <i>c</i> <i>a</i> <i>a</i> <i>b</i> <i>b</i> <i>c</i> <i>c</i> <i>a</i> <i>a</i> <i>b</i>


 


         <sub></sub>   <sub></sub>


       


<b>b</b>


3 3 3


3 3 3 3 3 3


3 3 3 3 3 3 3 3 3 3 3 3


1 1 1 1


4 3



2


<i>P</i>
    9 3 3 3


2 2 8


Đẳng thức xảy ra   <i>a</i> <i>b</i> <i>c</i>
Vậy <i>MinP</i>3


</div>
<span class='text_page_counter'>(43)</span><div class='page_container' data-page=43>

43 DAYHOCTOAN.VN


<b>Bài 86. </b> Cho các số dương <i>a b c</i>, , thỏa mãn <i>a b c</i>   2 <i>abc</i>. Tìm giá trị nhỏ nhất của<i>S</i>


<i>a</i> <i>b</i> <i>c</i>


  1 1 1.


<b>Bài giải: </b>


Chú ý rằng <i>a b c</i>   2 <i>abc</i> nên suy ra (<i>a</i>1)(<i>b</i>1)(<i>c</i> 1) (<i>a</i>1)(<i>b</i>  1) (<i>b</i> 1)(<i>c</i>  1) (<i>c</i> 1)(<i>a</i>1).
Do vậy ta thu được


<i>a</i> <i>b</i> <i>c</i>


<i>a</i> <i>b</i> <i>c</i>


      


   <sub></sub> <sub></sub> <sub></sub>



1 1 1 1 1 1


1 2


1 1 1


1 1 1 <sub>1</sub> <sub>1</sub> <sub>1</sub> .


Áp dụng bất đẳng thức <i>Cauchy-Schwarz</i> ta được


<i>a</i> <i>b</i> <i>c</i>


<i>a</i> <i>b</i> <i>c</i>


 


 


 <sub>    </sub>  <sub></sub> <sub></sub> <sub></sub>


 


 


 <sub></sub> <sub></sub> <sub></sub> <sub></sub> <sub></sub>


 


1 1 1 1 1 1 9



1 1 1


1 1 1 <sub>2</sub>


1 1 1


.


Từ hai điều trên ta suy ra


<i>a</i>  <i>b</i> <i>c</i>


1 1 1 3


2.
Vậy <i>S</i> nhỏ nhất bằng 3


2, dấu bằng xảy ra khi <i>a</i> <i>b</i> <i>c</i> và là nghiệm của <i>a</i>  <i>a</i>     <i>a</i> <i>b</i> <i>c</i>


3 <sub>3</sub> <sub>2 0</sub> <sub>2</sub>


.
<b>Bài 87. </b> Cho <i>x y z</i>, , là các số thực dương thỏa mãn <i>xy</i><i>yz</i><i>zx</i>3<i>xyz</i>. Chứng minh


<i>y</i>

 

<i>z</i>

 

<i>x</i>

 



<i>x y</i>   <i>y z</i>   <i>z x</i>  


2 2 2



2 2 2


3
1
2


1 1 1


<b>Bài giải: </b>


Ta có thể viết lại giả thiết <i>xy</i><i>yz</i><i>zx</i>3<i>xyz</i>thành


<i>x</i>  <i>y</i> <i>z</i>


1 1 1
3


Đặt <i>a</i> ,<i>b</i> ,<i>c</i>


<i>x</i> <i>y</i> <i>z</i>


1  1 1. Ta có <i>a b c</i>  3


Thay vào (1), ta cần chứng minh <i>a</i> <i>b</i> <i>c</i>


<i>b</i>  <i>c</i>  <i>a</i> 


 2  2  2



3


1 1 1 2


Thật vậy <i>a</i> <i>a</i> <i>ab</i> <i>a</i> <i>ab</i> <i>a</i> <i>ab</i>
<i>b</i>   <i>b</i>   <i>b</i>  


 


2 2


2 2


1 1 2 2


Làm tương tự và cộng lại ta có




<i>a</i> <i>b</i> <i>c</i>


<i>a</i> <i>b</i> <i>c</i> <i>ab bc</i> <i>ca</i>


<i>b</i>  <i>c</i>  <i>a</i>      


 2  2  2


1


1 1 1 2



Ta có bất đẳng thức quen thuộc

<i>a b c</i> 

2 3

<i>ab bc ca</i> 


Do đó <i>a</i> <i>b</i> <i>c</i>

<i>a</i> <i>b</i> <i>c</i>

<i>a</i> <i>b</i> <i>c</i>



<i>b</i>  <i>c</i>  <i>a</i>      


  


2


2 2 2


1


1 1 1 6


Sử dụng giả thiết <i>a b c</i>  3 ta suy ra


<i>a</i> <i>b</i> <i>c</i>


<i>b</i>  <i>c</i>  <i>a</i> 


 2  2  2


3


1 1 1 2


</div>
<span class='text_page_counter'>(44)</span><div class='page_container' data-page=44>

44 DAYHOCTOAN.VN



<b>Bài 88. </b> Cho các số thực dương <i>a</i>, <i>b</i>, <i>c</i> thỏa mãn <i>a b c</i>  1. Chứng minh rằng

2 2 2



2 2 2 2 2 2 8
<i>ab bc</i> <i>ca</i>


<i>a</i> <i>b</i> <i>c</i>


<i>a b</i> <i>b c</i> <i>c a</i>


  <sub></sub> <sub></sub> <sub></sub>


  .


<b>Bài giải: </b>


Đặt <i>t</i><i>ab bc ca</i>  , suy ra


2


1
0


3 3


<i>a b c</i>


<i>t</i>  


  



Áp dụng điều kiện <i>a b c</i>  1, bất đẳng thức cần chứng minh trở thành


2



2 8 2


2
<i>t</i>


<i>a b c</i> <i>ab bc ca</i>


<i>t</i> <i>abc a b c</i>


 


 <sub></sub>      <sub></sub>


  




2 8 1 2


2


<i>t</i>


<i>t</i>


<i>t</i> <i>abc</i>



  




Do <i>abc</i>0 nên ta có đánh giá: <sub>2</sub> <sub>2</sub> 1


2


<i>t</i> <i>t</i>


<i>t</i>  <i>abc</i> <i>t</i> <i>t</i>.
Để kết thúc bài toán ta sẽ chứng minh 1 8 1 2

<i>t</i>

 



<i>t</i>    .


Thật vậy

 

2


16<i>t</i> 8<i>t</i> 1 0


    

2


4<i>t</i> 1 0


   . Điều này luôn đúng.


Dấu bằng xảy ra khi và chỉ khi


1
0



1
4
<i>a b c</i>


<i>abc</i>


<i>t</i> <i>ab bc ca</i>


   


 <sub></sub>





    




Khi đó <i>a</i>, <i>b</i>, <i>c</i> là nghiệm phương trình 3 2 1 0
4


<i>x</i> <i>x</i>  <i>x</i>
Do đó

; ;

1 1; ;0


2 2
<i>a b c</i>   <sub></sub>



  hoặc các hoán vị.


<b>Bài 89. </b> Cho <i>a</i>, <i>b</i>, <i>c</i> là các số thực dương. Chứng minh rằng:


2 2 2


3
2


<i>a</i> <i>b</i> <i>c</i>


<i>ab b</i> <i>bc c</i> <i>ca</i> <i>a</i>


  


   .


<b>Bài giải: </b>


Ta có


2 2 2


1 1 1


<i>a</i> <i>b</i> <i>c</i>


<i>a</i> <i>b</i> <i>c</i> <i><sub>b</sub></i> <i><sub>c</sub></i> <i><sub>a</sub></i>


<i>a</i> <i>b</i> <i>c</i>



<i>ab b</i> <i>bc c</i> <i>ca</i> <i>a</i>


<i>b</i> <i>c</i> <i>a</i>


    


   <sub></sub> <sub></sub> <sub></sub>


2


1 1 1


<i>a</i> <i>b</i> <i>c</i>


<i>b</i> <i>c</i> <i>a</i>


<i>a</i> <i>b</i> <i>c</i>


<i>b</i> <i>c</i> <i>a</i>


 


 


 


 





    




Đặt <i>x</i> <i>a</i>
<i>b</i>


 , <i>y</i> <i>b</i>
<i>c</i>


 , <i>z</i> <i>c</i>
<i>a</i>


</div>
<span class='text_page_counter'>(45)</span><div class='page_container' data-page=45>

45 DAYHOCTOAN.VN


Ta có



2


2


1 1 1


1 1 1


<i>a</i> <i>b</i> <i>c</i>


<i>x</i> <i>y</i> <i>z</i>



<i>b</i> <i>c</i> <i>a</i>


<i>a</i> <i>b</i> <i>c</i> <i>x</i> <i>y</i> <i>z</i>


<i>b</i> <i>c</i> <i>a</i>


 


 


  <sub></sub> <sub></sub>


  <sub></sub>


    


    






2 <sub>6</sub>


3 3 3 3


<i>x</i> <i>y</i> <i>z</i> <i>xy</i> <i>yz</i> <i>zx</i> <i><sub>x</sub></i> <i><sub>y</sub></i> <i><sub>z</sub></i>


<i>x</i> <i>y</i> <i>z</i> <i>x</i> <i>y</i> <i>z</i>



     <sub>  </sub>


 


     


Suy ra


2 2 2


3
3


<i>a</i> <i>b</i> <i>c</i> <i>S</i>


<i>S</i>


<i>ab b</i> <i>bc c</i> <i>ca</i> <i>a</i>




  


  

<i>S</i>     <i>x</i> <i>y</i> <i>z</i> 3 6



Ta có 3 3 6 2 3 3 3


2 2 2 2 2


<i>S</i> <i>S</i>



<i>S</i>


<i>S</i> <i>S</i>


 


  <sub></sub><sub></sub>  <sub></sub><sub></sub>  


  Suy ra


3 3


3 2


<i>S</i>
<i>S</i>


 . Bất đẳng thức được chứng


minh.


Dấu bằng xảy ra khi <i>a</i> <i>b</i> <i>c</i>.


<b>Bài 90. </b> Cho các số thực , , ,<i>a b c d</i> thỏa mãn :<i>a</i>2<i>b</i>2 <i>c</i>2 <i>d</i>2 1. Tìm giá trị lớn nhất và nhỏ nhất của biểu
thức: 2 2 1





2



<i>E</i> <i>a</i> <i>c</i>  <i>a</i><i>c b</i><i>d</i> .


<b>Bài giải: </b>








2 2 1 2 2 2 2 2 2


2


<i>E</i><i>a</i> <i>c</i>  <i>a</i><i>c b</i><i>d</i> <i>a</i> <i>c</i>  <i>a</i> <i>c</i> <i>b</i> <i>d</i>




2 2 2 2 2 2


2 1 2 1


<i>a</i> <i>c</i>  <i>a</i> <i>c</i>   <i>b</i> <i>d</i> 


   <sub></sub>   <sub> </sub>   <sub></sub>


2 2

2 2



2 2 2 1 2 1


2


<i>a</i> <i>c</i> <i>b</i> <i>d</i>



<i>a</i> <i>c</i>


    


  


2 2 2 2



2 1 2 1


2 <i>a</i> <i>c</i> <i>b</i> <i>d</i> 2


 


    


Đẳng thức xảy ra khi và chỉ khi:

<sub></sub>

<sub></sub>



2 2 2 2


2 2 2 2


;


8 8


2 1


2 2 2 2



;


8 8


1


<i>a</i> <i>c</i>


<i>a</i> <i>c</i>


<i>b</i> <i>d</i>


<i>a</i> <i>b</i>


<i>b</i> <i>d</i>


<i>a</i> <i>b</i> <i>c</i> <i>d</i>


 


 <sub></sub> <sub></sub> <sub></sub>


 <sub> </sub> <sub> </sub> <sub> </sub>


 <sub></sub>


 <sub></sub> <sub></sub> 


   



  


 <sub></sub> <sub> </sub> <sub></sub> 







hoặc


2 2 2 2


;


8 8


2 2 2 2


;


8 8


<i>a</i> <i>c</i>


<i>b</i> <i>d</i>


 <sub></sub> <sub></sub>


   






 <sub> </sub>  <sub></sub> 











2 2 1 2 2 2 2 2 2


2


<i>E</i><i>a</i> <i>c</i>  <i>a</i><i>c b</i><i>d</i> <i>a</i> <i>c</i>  <i>a</i> <i>c</i> <i>b</i> <i>d</i>




2 2 2 2 2 2


2 1 2 1


<i>a</i> <i>c</i>  <i>a</i> <i>c</i>    <i>b</i> <i>d</i>


   <sub></sub>   <sub> </sub>  <sub></sub> 


2 2

2 2



2 2 2 1 2 1



2


<i>a</i> <i>c</i> <i>b</i> <i>d</i>


<i>a</i> <i>c</i>


    


</div>
<span class='text_page_counter'>(46)</span><div class='page_container' data-page=46>

46 DAYHOCTOAN.VN

2 2 2 2



1 2 1 2


2 <i>a</i> <i>c</i> <i>b</i> <i>d</i> 2


 


     .


Đẳng thức xảy ra khi và chỉ khi:

<sub></sub>

<sub></sub>



2 2 2 2


2 1
1


<i>a</i> <i>c</i>


<i>b</i> <i>d</i>



<i>a</i> <i>b</i>


<i>a</i> <i>b</i> <i>c</i> <i>d</i>


 

  


   


    


, hoặc


2 2 2 2


;


8 8


2 2 2 2


;


8 8



<i>a</i> <i>c</i>


<i>b</i> <i>d</i>


 <sub></sub> <sub></sub>


   





  


  





.


<b>Bài 91. </b> Cho các số thực không âm , ,<i>a b c</i> thoả mãn <i>a</i>2<i>b</i>2<i>c</i>2 2. Chứng minh bất đẳng thức






2 2 2 2 2 2


3 3 3



2


8 ; ;


3 <i>max a b b c c a</i>


<i>a</i> <i>b</i> <i>c</i> <i>abc</i>


<i>a b c</i>


   


 


<b>Bài giải: </b>


Do tính đối xứng của bất đẳng thức nên không mất tổng quát giả sử <i>a</i>  <i>b</i> <i>c</i> 0.
Áp dụng bất đẳng thức AM – GM, ta có 3 3 3 3 3

 



3 2 1


<i>a</i>   <i>b</i> <i>c</i> <i>abc</i><i>a</i> <i>b</i>  <i>ab ab</i>


2 2 2 2

 



2 2 2 2 2 4 2


<i>a b c</i>  <i>a</i>   <i>b</i> <i>c</i> <i>ab</i> <i>bc</i> <i>ca</i>  <i>ab</i> <i>ab</i>


Từ

 

1 và

 

2 suy ra

<i>a</i>3  <i>b</i>3 <i>c</i>3 3<i>abc a b c</i>

 

2 8<i>a b</i>2 2 8<i>max a b b c c a</i>

2 2; 2 2; 2 2




Hay





2 2 2 2 2 2


3 3 3


2


8 ; ;


3 <i>max a b b c c a</i>


<i>a</i> <i>b</i> <i>c</i> <i>abc</i>


<i>a b c</i>


   


 
Dấu đẳng thức xảy ra khi <i>a</i> <i>b</i> 1;<i>c</i>0


Tóm lại :





2 2 2 2 2 2



3 3 3


2


8 ; ;


3 <i>max a b b c c a</i>


<i>a</i> <i>b</i> <i>c</i> <i>abc</i>


<i>a b c</i>


   


 
Dấu đẳng thức khi <i>a</i> <i>b</i> 1;<i>c</i>0 và các hoán vị.


<b>Bài 92. </b> Cho các số thực dương <i>a</i>, <i>b</i>, <i>c</i> thỏa mãn 2 2 2

2


4.


<i>a</i> <i>b</i>  <i>c</i> <i>a b c</i>   Chứng minh bất đẳng thức

 

2

 

2

2


1 1 1


3


<i>ab</i> <i>bc</i> <i>ca</i>



<i>a b</i> <i>b c</i> <i>c</i> <i>a</i>


  


  


   .


<b>Bài giải: </b>


Ta có <i>a</i>2<i>b</i>2 <i>c</i>2

<i>a b c</i> 

2 4 <i>a</i>2<i>b</i>2 <i>c</i>2 <i>ab bc ca</i>  2.
Ta chứng minh


 

2

 

2

2


2 2 2 2 2 2


6


<i>ab</i> <i>bc</i> <i>ca</i>


<i>a b</i> <i>b c</i> <i>c</i> <i>a</i>


 <sub></sub>  <sub></sub>  <sub></sub>


   .


Thật vậy, vì 2 2 2


2<i>a</i> <i>b</i>  <i>c</i> <i>ab bc ca</i>  nên ta có :





2 2 2


2 2


2<i>ab</i> 2 2<i>ab</i> <i>a</i> <i>b</i> <i>c</i> <i>ab bc</i> <i>ca</i>


<i>a b</i> <i>a b</i>


 <sub></sub>      


 


 










2


2 1 2


<i>a b</i> <i>c</i> <i>a c b</i> <i>c</i> <i>a c b</i>


<i>a b</i> <i>a b</i>


     



  


 


Suy ra







 



2 2


2 2


1 <i>c</i> <i>a</i> <i>c b</i> 1


<i>ab</i>


<i>a b</i> <i>a b</i>


 




 


  .


Tương tự ta có








 



2 2


2 2


1 <i>a b c</i> <i>a</i> 2


<i>bc</i>


<i>b c</i> <i>b c</i>


 


 <sub> </sub>


  ,






 



2 2


2 2


1 <i>a b b c</i> 3



<i>ca</i>


<i>c</i> <i>a</i> <i>c</i> <i>a</i>


 


 <sub> </sub>


</div>
<span class='text_page_counter'>(47)</span><div class='page_container' data-page=47>

47 DAYHOCTOAN.VN
Cộng các BĐT

 

1 ,

 

2 và

 

3 theo vế với vế ta có :


 

 


















2 2 2 2 2 2


2 2 2 2 2 2


3 <i>c</i> <i>a b c</i> <i>a b c</i> <i>a</i> <i>b c</i> <i>a b</i>


<i>ab</i> <i>bc</i> <i>ca</i>



<i>a b</i> <i>b c</i> <i>c</i> <i>a</i> <i>a b</i> <i>b c</i> <i>c</i> <i>a</i>


     


 <sub></sub>  <sub></sub>  <sub> </sub> <sub></sub> <sub></sub>


      .


Áp dụng BĐT Cauchy cho 3 số dương ta có
















2 2 2 3


<i>c</i> <i>a b c</i> <i>a b c</i> <i>a</i> <i>b c</i> <i>a b</i>


<i>a b</i> <i>b c</i> <i>c</i> <i>a</i>


     


  


   . Suy ra



điều phải chứng minh.


Dấu "" xảy ra khi và chỉ khi 3
3
<i>a</i>  <i>b</i> <i>c</i> .


<b>Bài 93. </b> Chứng tỏ rằng tổng 2014<sub>2</sub> 2014<sub>2</sub> ... 2014<sub>2</sub> ... 2014<sub>2</sub>


2013 1 2013 2 2013 2013 2013


<i>A</i>


<i>n</i>


     


    (2013 số hạng)


không phải là số nguyên dương.


<b>Bài giải: </b>


Trước hết ta giải bài toán tổng quát:


“Chứng minh rằng tổng (<i>n</i> số hạng, <i>n</i>1) <sub>2</sub> 1 <sub>2</sub> 1 ... <sub>2</sub> 1


1 2


<i>n</i> <i>n</i> <i>n</i>



<i>A</i>


<i>n</i> <i>n</i> <i>n</i> <i>n</i>


  


   


   (<i>n</i> số hạng) không phải là số
nguyên dương”.


Ta có <i>A</i> <i>n</i> <sub>2</sub>1 <i>n</i> <sub>2</sub>1 ... <i>n</i> <sub>2</sub>1


<i>n</i> <i>n</i> <i>n</i>


  


    (<i>n</i> số hạng) <i>n</i> <sub>2</sub>1.<i>n</i> 1 1 2.


<i>n</i> <i>n</i>




   


Mặt khác <i>A</i> <i>n</i><sub>2</sub> 1 <i>n</i><sub>2</sub> 1 ... <i>n</i><sub>2</sub> 1


<i>n</i> <i>n</i> <i>n</i> <i>n</i> <i>n</i> <i>n</i>


  



   


   (<i>n</i> số hạng) 2


1


. 1


<i>n</i>
<i>n</i>


<i>n</i> <i>n</i>




 




Do đó 1 <i>A</i> 2.


Vậy <i>A</i> không phải là số nguyên dương.
Với <i>n</i>2013 thì ta có bài tốn đã cho.


<b>Bài 94. </b> Cho , ,<i>a b c</i> là các số thực dương thỏa mãn: <i>a b c</i>  1. Tìm giá trị nhỏ nhất của biểu thức:

<i>ab</i> 1



<i>bc</i> 1



<i>ca</i> 1



<i>P</i>



<i>abc</i>


  


 .


<b>Bài giải: </b>


Ta có: <i>P</i> <i>a</i> 1 <i>b</i> 1 <i>c</i> 1 <i>abc</i> 1 1 1 1 1


<i>b</i> <i>c</i> <i>a</i> <i>abc</i> <i>a</i> <i>b</i> <i>c</i>


   


<sub></sub>  <sub></sub>  <sub></sub>  <sub></sub>     


    .


Vì <i>a b c</i>  1 nên từ BĐT CauChy cho 3 số dương ta có: 1 3 1


3 3 27


<i>a</i> <i>b</i> <i>c</i>


<i>abc</i> <i>abc</i>


 


    .



Do đó: 1 27 1

27



1 27

0


27 27


<i>abc</i> <i>abc</i>


<i>abc</i>


<i>abc</i> <i>abc</i>


 


     .


Suy ra: 1 27 1 730
27 27


<i>abc</i>
<i>abc</i>


    .


Mặt khác ta lại có:

<i>a b c</i>

1 1 1 9 1 1 1 9


<i>a</i> <i>b</i> <i>c</i> <i>a</i> <i>b</i> <i>c</i>


 


  <sub></sub>   <sub></sub>    



</div>
<span class='text_page_counter'>(48)</span><div class='page_container' data-page=48>

48 DAYHOCTOAN.VN
Từ đó suy ra:


2


730 10


9 1


27 3


<i>P</i>      
  .
Vậy


2


10
min


3


<i>P</i>   
 


1
3


<i>x</i> <i>y</i> <i>z</i>



    .


<b>Bài 95. </b> Tìm giá trị lớn nhất của số thực <i>k</i> sao cho bất đẳng thức sau :


2



2



2

2



2



2

2



2



2


2. 1<i>a</i> 1<i>b</i> 1<i>c</i> 2. 1<i>b</i> 1<i>c</i> 1<i>d</i> 2 1<i>c</i> 1<i>d</i> 1<i>a</i>


2



2



2



2. 1 <i>d</i> 1 <i>a</i> 1 <i>b</i> <i>k ab bc cd</i> <i>ac bd</i> <i>da</i> 2


           đúng với mọi số thực <i>a</i>, <i>b</i>, <i>c</i>, <i>d</i> thay đổi
tùy ý.


<b>Bài giải: </b>


Điều kiện cần. Bất đẳng thức đúng với bộ <i>a</i>   <i>b</i> <i>c</i> <i>d</i> 3 thay vào BĐT ta được




4.2.8<i>k</i> 6.3 2  <i>k</i> 4


Điều kiện đủ. Ta chứng minh BĐT đúng với <i>k</i>4


2



2



2



, , ,


2 1 1 1 4 2



<i>a b c d</i>


<i>a</i> <i>b</i> <i>c</i> <i>ab ac</i> <i>ad</i> <i>bc bd</i> <i>cd</i>


         


2



2



2



, , ,


1 1 1 2 2


<i>a b c d</i>


<i>a</i> <i>b</i> <i>c</i> <i>ab</i> <i>ac</i> <i>ad</i> <i>bc bd</i> <i>cd</i>


          .


Ta có

1<i>a</i>2



1<i>b</i>2



1<i>c</i>2

 

 1<i>a</i>2

<sub></sub>

<i>bc</i>1

 

2 <i>b c</i>

2<sub></sub> 

<i>bc</i> 1

 

<i>a b c</i> 

<i>ab bc ca</i>  1.
Tương tự trên ta được


2



2



2



, , ,


1 1 1 2 2


<i>a b c d</i>


<i>a</i> <i>b</i> <i>c</i> <i>ab ac</i> <i>ad</i> <i>bc bd</i> <i>cd</i>



         




Dấu bằng xảy ra khi <i>a</i>    <i>b</i> <i>c</i> <i>d</i> 3


Vậy giá trị lớn nhất của <i>k</i> bằng <i>d</i>. Khi đó BĐT luôn đúng với mọi số thực <i>a</i>, <i>b</i>, <i>c</i>, <i>d</i> thay đổi tùy ý
<b>Bài 96. </b> Giả sử <i>x</i>, <i>y</i> là các số thực dương thỏa mãn <i>x</i> <i>y</i> 1. Chứng minh rằng:


3 3


2 5


3 3


<i>xy</i>  <i>x</i>  <i>y</i> >


36 2 104 5
81


<b>Bài giải: </b>


<sub></sub>

<sub></sub>

3


3 3 3 3 2 3


1 1 1 1 4 4



4


3<i>xy</i><i>x</i> <i>y</i> 3<i>xy x</i><i>y</i>  <i>x</i> <i>y</i>  <i>x</i> 3<i>xy x</i><i>y</i> <i>y</i>  <i><sub>x</sub></i><i><sub>y</sub></i> 

 

1
Mặt khác,




2


2


1 1 1 4


4 1


4 9 9


<i>x</i> <i>y</i> <i>xy</i>


<i>xy</i> <i>x</i> <i>y</i> <i>xy</i>


      


 

2


Từ

 

1 và

 

2 ta có:


3 3


2 5



3 3


<i>xy</i>  <i>x</i>  <i>y</i>




3 3


4 9 2 5


9 2 5 5 1 1 4 5 36 2 104 5


9<i>xy</i> 3 <i>x</i> <i>y</i> 3<i>xy</i> 81 3 81




 


 


  <sub></sub>  <sub></sub>  




 


<b>Bài 97. </b> Cho <i>a</i>, <i>b</i>, <i>c</i> là các số thực dương. Chứng minh rằng 2<i>a</i> 2<i>b</i> 2<i>c</i> 3


<i>a b</i> <i>b c</i> <i>c a</i> 



</div>
<span class='text_page_counter'>(49)</span><div class='page_container' data-page=49>

49 DAYHOCTOAN.VN
Đặt <i>x</i> <i>b</i>


<i>a</i>


 , <i>y</i> <i>c</i>
<i>b</i>


 , <i>z</i> <i>a</i>
<i>c</i>


 , ta có: , , <i>x y z</i>0 và <i>xyz</i>1.


Bất đẳng thức đã cho trở thành: 2 <sub>2</sub> 2 <sub>2</sub> 2 <sub>2</sub> 3
1<i>x</i>  1<i>y</i>  1<i>z</i> 
Giả sử <i>xy</i>  1 <i>z</i> 1.


* Ta chứng minh đẳng thức sau: 1 <sub>2</sub> 1 <sub>2</sub> 2

 

1
1<i>x</i> 1<i>y</i> 1<i>xy</i>
Thật vậy,

 

2 2

2



2



1  2<i>x</i> <i>y</i> 1<i>xy</i> 2 1<i>x</i> 1<i>y</i>
 

1 <i>xy</i>



<i>x</i><i>y</i>

2 0 (đúng) .


Ta có: Theo bất đẳng thức Bunhiacopski
2


2 2 2 2 2 2


2 2 2 2 1 1



2 4


1 <i>x</i> 1 <i>y</i> 1 <i>x</i> 1 <i>y</i> 1 <i>x</i> 1 <i>y</i>


     


    


     


 <sub></sub> <sub></sub>  <sub></sub> <sub></sub> <sub></sub> <sub></sub> <sub></sub> <sub></sub> <sub></sub> <sub></sub>


  .


Theo bất đẳng thức

 

1 suy ra: 4 1 <sub>2</sub> 1 <sub>2</sub> 8 8


1 1 1 1


<i>z</i>


<i>x</i> <i>y</i> <i>xy</i> <i>z</i>


 


  


 <sub></sub> <sub></sub>  <sub></sub> <sub></sub>


  .



Suy ra 2 <sub>2</sub> 2 <sub>2</sub> 2 2


1 1 1


<i>z</i>


<i>x</i>  <i>y</i>  <i>z</i>


   .


Mặt khác, ta lại có 2 <sub>2</sub> 2
1<i>z</i> 1<i>z</i>


Suy ra 2 <sub>2</sub> 2 <sub>2</sub> 2 <sub>2</sub> 2 2 2


1 1 1 1 1


<i>z</i>


<i>x</i>  <i>y</i>  <i>z</i>  <i>z</i>  <i>z</i>


     .


Do vậy, ta sẽ chứng minh : 2 2 2 3


1 1


<i>z</i>



<i>z</i>  <i>z</i>


  .


Thật vậy, ta có: 2 2 2 3 2 2 1

2 3 1



1 1


<i>z</i>


<i>z</i> <i>z</i> <i>z</i>


<i>z</i>  <i>z</i>      


  .


 


2<i>z</i> 2 2 1<i>z</i> <i>z</i> 1 <i>z</i> 0


      

2<i>z</i> 1<i>z</i>

2 0(luôn đúng).
Vậy bất đẳng thức đã được chứng minh.


</div>
<span class='text_page_counter'>(50)</span><div class='page_container' data-page=50>

50 DAYHOCTOAN.VN


<b>Bài 98. </b> Cho các số thực không âm <i>a</i>, <i>b</i>, <i>c</i> thỏa mãn

<i>a</i>

2

  

<i>b</i>

2

<i>c</i>

2

2

. Chứng minh rằng


1

1

1

9



2

<i>ab</i>

2

<i>bc</i>

2

<i>ca</i>

4

.



<b>Bài giải: </b>


BĐT tương đương với


2 2 2 9


1 1 1 3


2 <i>ab</i> 2 <i>bc</i> 2 <i>ca</i> 2


 <sub> </sub>  <sub> </sub>  <sub>  </sub>


 <sub></sub>   <sub></sub>   <sub></sub> 


     


3


2 2 2 2


<i>ab</i> <i>bc</i> <i>ca</i>


<i>ab</i> <i>bc</i> <i>ca</i>


   


  


2 2 2




2


2 2 2


<i>ab</i> <i>ab</i>


<i>ab</i> <i>a</i> <i>b</i> <i>c</i> <i>ab</i>


    2 2 2 2 2 2 2 2 2


2 2


( )


<i>ab</i> <i>ab</i>


<i>a</i> <i>b</i> <i>c</i> <i>c</i> <i>a b</i> <i>a</i> <i>b</i> <i>c</i> <i>c</i>


 


       


2 2 2


2 2 2 2 2 2 2 2


1 ( ) 1


2 ( ) ( ) 2



<i>a b</i> <i>a</i> <i>b</i>


<i>c</i> <i>a</i> <i>c</i> <i>b</i> <i>c</i> <i>a</i> <i>c</i> <i>b</i>


 




  <sub></sub>  <sub></sub>


   <sub></sub>   <sub></sub> (Sử dụng BĐT Cauchy-Schwarz)


Tương tự ta có


2 2 2 2


2 2 2 2 2 2 2 2


1 1


;


2 2 2 2


<i>bc</i> <i>b</i> <i>c</i> <i>ca</i> <i>c</i> <i>a</i>


<i>bc</i> <i>a</i> <i>b</i> <i>a</i> <i>c</i> <i>ca</i> <i>c</i> <i>b</i> <i>a</i> <i>b</i>


   



 <sub></sub>  <sub></sub>  <sub></sub>  <sub></sub>


 <sub></sub>   <sub></sub>  <sub></sub>   <sub></sub>


Cộng vế với vế các BĐT trên ta có đpcm, chỉ ra dấu "" khi 2
3
<i>a</i>  <i>b</i> <i>c</i> .


<b>Bài 99. </b> Cho ba số dương <i>a</i>, <i>b</i>, <i>c</i>. Chứng minh rằng 4<i>a</i> 1 4<i>b</i> 1 4<i>c</i> 1 25


<i>b c</i> <i>c</i> <i>a</i> <i>a b</i>


 <sub></sub>  <sub></sub>  <sub> </sub>


 <sub></sub>  <sub></sub>  <sub></sub> 


   

 

1 .


<b>Bài giải: </b>


Không giảm tổng quát, giả sử <i>c</i> là số lớn nhất trong ba số <i>a</i>, <i>b</i>, <i>c</i>.


Đặt <i>S</i>  <i>a b c</i>, <i>R</i><i>ab bc ca</i>  , <i>P</i><i>abc</i>.

  

1  <i>S</i>3<i>a</i>



<i>S</i>3<i>b S</i>



3<i>c</i>

25

<i>S</i><i>a</i>



<i>S</i><i>b S</i>



<i>c</i>



 



3 2 3 2


3 9 27 25



<i>S</i> <i>S</i> <i>a b c</i> <i>S ab bc ca</i> <i>abc</i> <i>S</i> <i>S</i> <i>a b c</i> <i>S ab bc ca</i> <i>abc</i>


         <sub></sub>        <sub></sub>




3


4<i>S</i> 9<i>SR</i> 27<i>P</i> 25 <i>SR</i> <i>P</i>


     3


4 13 0


<i>S</i> <i>SR</i> <i>P</i>


   

 

2
Ta chứng minh

 

2 đúng. Thật vậy,


  

3





VT 2  <i>a b c</i>  4 <i>a b c</i>  <i>ab bc ca</i>  13

<i>a b c</i> 

 

<sub></sub> <i>a b c</i> 

24

<i>ab bc</i> <i>ca</i>

<sub></sub>13<i>abc</i>


 

2


4 13


<i>a b c</i>  <i>a b c</i> <i>ab</i> <i>abc</i>


   <sub></sub>    <sub></sub>



2




4 13


<i>a b c</i> <i>a b c</i> <i>ab a b c</i> <i>abc</i>


        




2



9 4 4 0


<i>a b c</i> <i>a b c</i> <i>ab</i> <i>c</i> <i>a</i> <i>b</i>


        

do 9<i>c</i>4<i>a</i>4<i>b</i> <i>c</i> 0

.
Vậy ta có điều phải chứng minh.


<b>Bài 100. </b> Cho ba số thực dương <i>a</i>, <i>b</i>, <i>c</i> chứng minh rằng


2 2 2


2 2 2 2 2 2 2 2 2 0


2 2 2


<i>a</i> <i>bc</i> <i>b</i> <i>ca</i> <i>c</i> <i>ab</i>


<i>a</i> <i>b</i> <i>c</i> <i>b</i> <i>c</i> <i>a</i> <i>c</i> <i>a</i> <i>b</i>


  



  


      .


<b>Bài giải: </b>


BĐT tương đương với



2 2 2


2 2 2 2 2 2 2 2 2 3


2 2 2


<i>b c</i> <i>c a</i> <i>a b</i>


<i>a</i> <i>b</i> <i>c</i> <i>b</i> <i>c</i> <i>a</i> <i>c</i> <i>a</i> <i>b</i>


  


  


</div>
<span class='text_page_counter'>(51)</span><div class='page_container' data-page=51>

51 DAYHOCTOAN.VN


Ta có



 



2 2 <sub>2</sub> <sub>2</sub>



2


2 2 2 <sub>2</sub> <sub>2</sub> <sub>2</sub> 2 2 2 2


2


<i>b c</i> <i>b c</i> <i>b</i> <i>c</i>


<i>a</i> <i>b</i> <i>c</i> <i><sub>a</sub></i> <i><sub>b</sub></i> <i><sub>a</sub></i> <i><sub>c</sub></i> <i>a</i> <i>b</i> <i>a</i> <i>c</i>


 


  


  <sub></sub> <sub></sub> <sub></sub>   .


Tương tự với hai BĐT còn lại suy ra đpcm


<b>Bài 101. </b> Cho , , <i>a b c</i>0 thỏa mãn <i>a b c abc</i>   4. Chứng minh rằng




2
2


<i>a</i> <i>b</i> <i>c</i>


<i>a b c</i>
<i>b c</i>  <i>a c</i>  <i>a b</i>    .



<b>Bài giải: </b>


Áp dụng BĐT Bunhiakopxki ta được






2
<i>a b c</i>


<i>a</i> <i>b</i> <i>c</i>


<i>T</i>


<i>b c</i> <i>a</i> <i>c</i> <i>a b</i> <i>a b c</i> <i>b a</i> <i>c</i> <i>c a b</i>


 


   


       


Lại có

<i>a b c</i> <i>b a c</i> <i>c a b</i>

2 

<i>a b c</i> 



2<i>ab</i>2<i>bc</i>2<i>ac</i>


Suy ra:





2



<i>a b c</i> <i>a b c</i>


<i>T</i>


<i>ab bc</i> <i>ac</i>


   




 

 

*


Ta sẽ chứng minh <i>a b c</i>  <i>ab bc ca</i> 

 

1
Đặt <i>a b</i> <i>S</i>, <i>ab</i><i>P</i>,


2
4
<i>S</i>
<i>p</i>


 




 


 .
Từ giả thiết suy ra 4


1



<i>S</i>
<i>c</i>


<i>P</i>



  <i>S</i> 4.
Vậy

 

1 4

4



1 1


<i>S</i> <i>S</i>


<i>S</i>


<i>S</i> <i>P</i>


<i>P</i> <i>P</i>





   


 

 



2



1 2


<i>P P</i> <i>S</i> <i>S</i>


    

 

2


Nếu <i>P</i>  1 <i>S</i> 0VT 0 VP.
Nếu<i>P</i>  1 <i>S</i> 0. Ta có



2 2


1 1


4 4


<i>S</i> <i>S</i>


<i>P P</i> <i>S</i>  <sub></sub>  <i>S</i><sub></sub>
  (vì


2
4
<i>S</i>
<i>P</i> ).


Suy ra

 



2


2 2



1 2 2


16
<i>S</i>


<i>P P</i> <i>S</i>  <i>S</i>  <i>S</i> (vì <i>S</i>4).


Vậy:<i>a b c</i>  <i>ab bc ac</i>  . Từ

 

* suy ra


</div>

<!--links-->

Tài liệu bạn tìm kiếm đã sẵn sàng tải về

Tải bản đầy đủ ngay
×